You are on page 1of 183

Previous 30 Years

Objective & Conventional solved questions of


MADE ERSY
Publications

IAS St IFS
Civil & Mechanical Engineering

ST' NGTH of A RIALS

Useful for

Civil Services Examination Engineering Services Examination

State Engineering Services Examinations Public Sector Examinations

Dr. U.C. Jindal


IAS & IFS
IAS & IFS
(Objective
(Objective &&Conventional)
(Objective & Conventional)
Conventional)
Previous
Previous Solved
Previous Solved
Solved Questions
Questions
Questions

Strength of
Strength
Strength of Materials
ofMaterials
Materials
Previous
Previous 30
Previous 30Years
30 Years
Years Solved Questions
Solved
Solved of of
Questions
Questions of
Civil &
Civil &Mechanical
MechanicalEngineering
Mechanical Engineering
Engineering

Useful for
Useful
Useful for ESE, CSE,
for ESE,
ESE, CSE,
CSE, State
State
State Engg. Services,
Engg.
Engg. PSUs
Services,
Services, PSUs
PSUs
and
and Other
OtherCompetitive
and Other Examinations
CompetitiveExaminations
Competitive Examinations

Dr.
Dr. U.C.
U.C. Jindal
Jindal
Jindal
M.Tech, Ph.D.
M.Tech, Ph.D.
Former Professor
Former Professor &
& Head
Head of
of the
the Department
Department
Department
Department ofofMechanical
Department of Mechanical
MechanicalEngineering
Engineering
Engineering
Delhi
Delhi College
College of
College of Engineering,
of Engineering, Delhi
Engineering,Delhi
Delhi

I
I
MADE EFISH
Publications
Public:mime
Public:mime

MADE EASY
MADE EASY
— Publications
Publications
Preface

I am thankful to Mr. B. Singh, CMD of MADE EASY Group, who is ever ready to help the Student

Community by providing them newest type of books, as in the present book with typical/thought

provoking/mind racking questions asked in IFS and IAS. Prelims and Mains of UPSC, for the

last 30 years for both Civil and Mechanical engineering, in the subject of Strength of Materials.

For the solution of each question a student must be equipped with strong concepts in the

subject, and the students are the beneficiaries of the latest and comprehensive knowledge

of the subject of the qualified and dedicated faculty of MADE EASY.

Further improvements in the text of the book will be made after getting the feedback from the

students.

Any error in printing or calculations pointed out by the reader will be acknowledged with thanks

by the author.

Dr. U.C. Jindal

Author
Contents
Contents
IAS
IAS &
& IFS
IFS(Objective
(Objective &
& Conventional) Previous Solved
Conventional) Previous Solved Questions
Questions
1
Strength
Strength of
of Materials
Materials 1
Sl.
SI. Chapter
Chapter Pages
Pages

1.
1. Simple Stresses
Simple Stresses in
inUniform
Uniform and
and Compound Bars............................................................. 1-19
Compound Bars 1-19

2.
2. Principal Stresses........................................................................................................................20-38
Principal Stresses 20-38

3.
3. Thin and
Thin and Thick
Thick Shells
Shells.................................................................................................................39-51
39-51

4.
4. Shear Force
Shear Forceand
and Bending
Bending Moment Diagrams..................................................................52-69
Moment Diagrams 52-69

5.
5. Theory of
Theory of Simple
Simple Bending.
Bending......................................................................................................70-88
70-88

6.
6. Deflection of Beams
Deflection of Beams............................................................................................................... 88-107
88-107

7.
7. Torsion ......................................................................................................................................108-123
Torsion 108-123

8.
8. Springs ......................................................................................................................................124-130
Springs 124-130

9.
9. Struts and
Struts and Columns..............................................................................................................131-137
Columns 131-137

10.
10. Theories of
Theories of Failure
Failure.................................................................................................................138-148
138-148

11.
11. Strain Energy
Strain Energy Methods........................................................................................................149-157
Methods 149-157

12.
12. Miscellaneous Questions
Miscellaneous Questions....................................................................................................158-165
158-165

13.
13. Rotational Stresses
Rotational Stresses................................................................................................................166-169
166-169

14.
14. Unsymmetrical
Unsymmetrical Bending.....................................................................................................170-173
Bending 170-173

15.
15. General Objective
General Objective Type
Type Questions
Questions..................................................................................174-179
174-179
01
01
CHAPTER
Simple Stresses in Uniform
and Compound BarsBars

Q.1.1 AAsteel
Q.1.1 steelrod
rodofoflength
length300
300mmmmand
anddiameter
diameter3030mm mm isis subjected
subjected to
to a pull
pull P, and the temperature
rise is
is 100°C. IfIf the total extension
extension of the
the rod
rod is
is 0.40
0.40 mm,
mm, calculate the magnitude of P. Take a for
1Cr61°C and E
steel = 12 xx 10-6/°C
steel= 106 N/mm2.
E = 0.215 x 106 N/mm2 .
[CSE-Mains, 2011, CE : 12 Marks]
[CSE-Mains,

Solution:
P = Pull in N
P=

A=
= Area of
of cross-section % 2
cross-section == 4 x 30 2 = 706.86 mm
mm2
2

Extension due to pull,


pull, (assuming pull axial)
~XL P x 300 x -6
xL _ Px300 _
6 -= 1.974
1 974 x 10-6P mm
&11 = AE 706.86x0.215x10
- 706.86 x0.215 x10- · Pmm

612 extension due to


8/2 to temperature
temperature change
change
a.UT
= aLAT
= 12 x 10-6 x 300 x 100 =0.36
100 =
0.36 + 1.974 x 10-6 P = 0.4
1.974 xx 10-6 P = 0.04
1 6
4 x106
P = 0.04
P 0.0 x 0 - = 20263 N = 20.263 kN
1.974

Q.1.2 AAmetallic
metallicbar
bar250
250mm
mmxx100
100mmmmx x5050
mmmmisisloaded
loadedasasshown
shownininthe
thefigure
figure 1.1.
1.1.Work
Work out
out the
the
change in
change should be the change that should be made in
in volume. What should in the 4 MN load in
in order that
should be no change in
there should in the
the volume of the bar.
4 MN
4MN
z

i
50 _ _ _ _ _ _ __ 400 kN
,..x 50
T ,___ __,,,,,_____.,
mm
250 mm
2 MN
2MN

Fig. 1.1
Fig. 1.1
5
E = 2 x 106
Assume E=
2
10 N/mm2,
N/mm , Poisson's =
ratio = 0.25.
Poisson's ratio 0.25.
[IFS 2011, CE : 15 Marks]
Marks]
22 ~= /AS &
IAS & IFS
IFS (Objective & Conventional)
(Objective & Previous Solved
Conventional) Previous Solved Questions EASY
MADE EASY

Solution:

Stresses 0 = +
+ 400,000
400,000 = +80 MPa
= +80 MPa
x 5000

x1066
2x10
2
a
CJy _
= + = +160 MPa
50x250 = +160 MPa
+ 50x250

4x1066
4x10
CJz =
6 160MPa
= 160
250 x100 = -
250x100 MPa

a -Fa +a 1a, +ay +az )


Volumetric strain, Et, - Y z 2v

= 80
= 80 -2
- 2 x0.25x
80 = 40
x 0.25 80
x- =
40
E E
E E E
3
Volume == 250
Volume 250 xx 100
100 x 50 mm3
mm
40
6V = change in
81/ in volume= evv xV
volume . s = Ex250x100x50
xV = — x 250 x100 x 50

50 x1066 50
50x10 x106
= sax
6
10 == 250 mm3
mm 3
E 2x1055
2x10
(b) For Ey
ev = ;;;;; 0
0
+ ay + az - 2v(a, + ay + az)
(Jx (Jy + (Jz - 2v(ax (Jy az> = 0 = 0
+ ay)(1
(ax+ ay) (1-2v}
- 2v) == 2v(az)
2v(az>-az
(80 + 160)(1-2v}
160) (1 - 2v) == (2v - 1) CJz
(2v-1} az
240Xx0.5
240 0.5== (2v-1)(2v -1) = -0.5 az
CJz
120 =
120 ;;;;; -<Jz Xx 0.5
a
az==-240 -240MPa
MPa
where in load, Py'' =; ; ; 240 x 250 x 100
P = 6 MN
100;;;;;
4 MN load should be increased to 6 MN load in same same direction
direction
So that az becomes-240 N/mm 2 .
becomes -240 N/mm2.

Q.1.3 AAcrane
cranechain
chainhaving
having an cm2 carries a load of 15
an area 7.25 cm2 15 kN.
kN. ItIt is
is being
being lowered
lowered at
at aa uniform
uniform
speed of 50
speed of 50 m/minute, the
the chain gets
gets jammed
jammed suddenly,
suddenly, at
at that
that time
time the length of chain unwound
is 12 m. Estimate the stress
stress induced
induced in
in the
the chain
chain due
due to
to sudden
sudden stoppage.
stoppage. Neglect
Neglect weight of the
chain. Assume E.E = 2.1
2.1 x 105 N/mm2 .
Hf N/mm2.
[IFS 2012, CE : 10 Marks]
Marks]

Solution:
W = 15 kN = 15000 N (-; chain negligible)
(·: self weight of chain negligible)
2
A, chain area
area of
ofcross-section
cross-section == 725725 mm2
mm
Length, L=;;;;; 12000 mm
Volumeof
Volume ofchain
chain == 12000
12000 x 725 105 mm3
725 = 87 x 105 mm3
Say, stress
Say, stressdeveloped N/mm22
developed== ai, instantaneous in N/mm

50
Speed, V= = 0.833 m/s
= 60
60

mV 22 15000
mV 0.833 2
15000 0.8332=
energy absorbed
Kinetic energy absorbed by
bychain = - - = 9.81 xx
chain ;;;;; Nm = 530500 Nmm
;;;;; 530.5 Nm;;;;;
2 9.81 2
Strength of Materials Stresses in
Simple Stresses in Uniform and Compound
Uniform and Compound Bars
Bars 3
4 3
<1111

a.x 12
12000
Change in length,
Change in length, 6L = a
SL=
0
a; xi= 1; x ooo
E E
Change in
in potential
potentialenergy ltVlil
energy == WSl
= 15000 x a; x12000 N
15000XO';X12000 Nmm -857143 N
2.1x10s mm =
- 857.143
. aiNmm
a; mm
2.1x 105
2
Gi
Total strainenergy
Total strain energyabsorbed
absorbedby
bychain
chain == 530500
530500 + 857.143 a.=
cri = —
;~ volume
2E

ai2
530500 + 857.14a;
857.14 a, -
= O'i x87x 10 5 == 20.7143
x €3 7 x105 20.7143$52
crf
2 x2.1x10'5
2x2.1x10

or
Or aF - 41.38a1 - 25610.3 == 0
cr~-41.38cr;-25610.3 O

41.38 +V41.3822 +
41.38+.J41.38 + 44x25610.3
x 25610.3
-
2
+ ../1712.30+ 102441.2
41.38+J1712.30+102441.2
41.38
2
41.38++ 322.728
41.38 322. 728
2
= 182.054 N/mm2
182.054 N/mm2

Provethat
Q.1.4 Prove thatPoisson's
Poisson's ratio
ratio cannot
cannot be greater than
than 0.5.
[CSE-Mains, 1988, ME
[CSE-Mains, ME:: 15 Marks]

Solution:
Figure shows a sphere
sphere under uniform
uniform hydrostatic
hydrostatic pressure p.
E = Young's
If E= Young's modulus
modulus and vis
v is Poisson's ratio,
ratio, v, then

3p
Volumetric strain, Ev (1-2v)=0

or ratio == 0.5
Poisson's ratio
v, Poisson's
If vis 0.5, then in place of decrease in
v is greater than 0.5, in volume
volume there will
will be p
increase in volume,
volume, which is not possible.
possible. Fig. 1.2

Q.1.5 A A
Q.1.5 1000
1000 mm mm long
long bar
bar isissubjected
subjectedtotoan
anaxial
axialpull
pullPwhich
Pwhichinduces
inducesaamaximum
maximum stress
stress of 1500 kg/crn2.
kg/cm2.
The area of cross-section of the cm22 over a length of 950 mm and for the central 50 mm
the bar is 2 cm
length, the sectional
sectional area cm2 .
area is equal to 1 cm2.
2
2cm
2 cm2

P~-------------------~--------------------~P
P P
2
1 an
cm2
1- 47.5 cm -P-1l---47.5cm---l
l---47.5cm---l k 47.5 cm -id
5.0 cm
5.0cm

Fig.
Fig. 1.3

Efor
Assuming that E for bar material
material is 20 xx 1a5
is 20 kgf/cm2 , calculate strain energy stored in
105 kgf/cm2, in bar
[CSE-Mains, 1990, ME : 20 Marks]
[CSE-Mains,
44 ~= /AS &
IAS & IFS
IFS (Objective & Conventional)
(Objective & Previous Solved
Conventional) Previous Solved Questions EASY
MADE EASY

Solution:
2
Maximum stress will occur in central portion of area 1 cm
cm2, ,
2
= 1500
1500 kgf/cm2
kgf/cm

Stress inother
Stress in otherportion 1500 xx 1
portion=- 1500 1=-- 750
750 kgf/cm
kgf/cm 22
2
2 2
1500
15002 7502
U, energy =- ~x5x1+2Ex47.5x2
strainenergy
U, strain x 5 x1+ x 47.5 x 2
2E 2E

5625x 103 26718.75


5625 x103 26718.75x 103
x 103
= E + E
E + E
32343750 32343750
32343750 32343750
=
E 20x10 5
20x105
= 16.17
16.17kgf-cm
kgf-cm (Strain
(Strain energy stored)

Q.1.6 AAsteel
steelrod
rodofofsquare
squarecross-section
cross-sectionisisloaded
loadedas
asshown
shown in figure 1.4.
in the figure

A B 10 mm
C D k-
T
T
E
~ 150
E 150 kN
kN 100 kN 75kN
75 kN 25 kN
25kN
-4- 4--------- 10 mm E
II)
U)
N
N N
!k
[4-2 m 2m 2m>1
l----2m----1---2m----1---2m---l
>I<

Fig. 1.4
Fig.

Find the section which is subjected


subjected to maximum stress, its
its magnitude
magnitude and nature. What will
will be the
change in
change in its
its length? E == 200 GPa.
length? Take E

Solution:
A B
B
....-----.---, B c
C cC D

150 kN
150kN E 150 kN 50 kN
50kN E 50 kN
50kN 25 kN 25 kN
► E 4— —110. E 4— 4- I10 mm ►
0
~ ~
N

Hollow
Fig.1.5

Considering compressive stress as (-ve)


(-ve) and vice-versa
150,000
150 000 .
dAB = ; = 240 MPa
= -240 suare
(squaresection
MPa (q section))
6
625 5
5
50 ,000 = -125 MPa
0,000
aBc = 125 MPa
400
25000
25
Oen = +
Gap — 000 = +83.33 MPa
400-100
400 -100
°max =
amax MPa (in
= -240 MPa (inportion
portion AB)
240
240 xx2000 125 x 2000 + 83.33 x 2000
Changeininlength
Change length= = 200,000 2000 _ 125 x 2000 + 83.33 x 2000
200, 000 200, 000
200,000 200, 000
200,000
= -2.4 -1.25 ++0.833
-2.4-1.25 0.833==-2.817
-2.817 mm
mm (contraction)
(contraction)
Strength of Materials Simple Stresses in Uniform and Compound Bars 4 5

Q.1.7 A rigid bar AD is pinned at A and attached to the bars BC


and ED as shown in figure 1.5. The entire system is initially I
E
E
stress free and weight of all bars are negligible. The
0
cv
temperature of the bar BC is lowered by 25°C and that of bar
ED is raised by 25°C. Neglecting any possibility of lateral
buckling, find the normal stress in bars BC and ED.
For BC, of brass, E = 90 GPa, a = 20 x 10-6/°C
For ED, of steel, E = 200 GPa and a = 12 x 10-6/°C
Cross-sectional area of BC is 500 mm2 and that of ED is
250 mm2.
[IFS 2011, CE : 10 Marks]
mm 350 mm-0-I
Solution:
Fig. 1.5
Free expansion in steel bar (if there were no resultant)
8/s = 250x 12x 10-6 x 25 = 0.075 mm
Free contraction in brass bar, SlB = 300 x 20 x 10-6 x 25 = 0.15 mm
If brass bar is free to move down, then end D would move by

600
0.15 x = 0.36 mm
250
(... ABD is rigid, vertical deflection in ABD will be linearly proportional to distance from A)
But free expansion in steel bar is only 0.075 mm.
Therefore steel bar will be pulled down and brass bar will be pulled up, or stretched to prevent, free
contraction
Say as = Stress in steel bar
6B = Stress in brass bar
Ps = Force in steel bar = as x 250 N
PB = Force in brass bar = 08 x 500 N
Taking moments about B
600 x Ps = 250 PB
600 x as x 250 = 250 x 500 x as
150000 as = 1250000'B
6B = 0.833 GB
or aB = 1.2 as
Brass bar
aB
Final contraction = 0.15 - — - x 300 = 81g'
Eg
Steel bar
as
Final extension = 0.075+ Es x 250 = Sls'

, 600 RI R7
But = X (N g' 2.4 viB '
250
Putting the value

0.075 + Es x 250 = 2.4[0.15 - x 300


s EB ]
1.2a
0.075 + Gs x 250 = 0.36 x 720
20Q 000 90,000
6 0- lAS & IFS (Objective & Conventional) Previous Solved Questions MADE EASY

as x 1.25 x 10-3 + as x 9.6 x 10-3 = 0.285


as [10.85] = 285
= 26.26 N/mm2 (tensile)
aB = 31.52 Nimm2 (tensile)
Checking
Steel bar, total extension = 0.075 + 0.032825 = 0.107825 = 8/s
31.52x
Brass bar, final contraction = 0.15 300 = 0.15 — 0.105066
90,000
8/B = 0.04493
8/B x 2.4 = 0.10784 ^ 8/s

Q.1.8 A plate is riveted to a channel section in a 12.5 kN


structure as shown in figure 1.6. An eccentric
load of 12.5 kN acts as shown on the plate. Channel
'V
Determine the rivet diameter so that the
maximum shear stress in any rivet is not to
E
exceed 40 MPa. Diameter of the rivet should E
O
be chosen from preferred series diameter c\I
(in mm)
r = 141.4 mm, /2 = 20000 mm2
12, 14, 16, 18, 20, 22, 24, 27, 30, 33, 36, 39,
42, 48 rivet diameter
I1 II
[CSE-Mains, 2009, ME : 40 Marks] 200 mm 50 mm 400 mm

Solution: Fig. 1.6


Number of rivets = 5
Load = 12.5 kN
12.5
Direct shear load on each rivet — = 2.5 kN
5
If A = Area of rivet
2500
i
d— A N/mm2

Torsional shear stress, is « r


Torsional shear force = krA
4 kA (r2 ) = P.e. = 12500 x (100 + 50 + 400)
4 kA (20000) = 12500 x 550 Nmm
12500 x 550
kA = = 85.9375
80000
85.9375
k—
A
Torsional shear stress
85.9375 x 141.4 12151.5625
"Cs , in outer rivets = kr —
A A
2500
'Cd = A
Fig. 1.7

tir = \('rs x sin45° + td)2 + (Ts x cos4512


Strength of Materials Simple Stresses in Uniform and Compound Bars 7

2 2
I(12151.5625 x 0.707+ 2500) 0 2151.5625
+ x0.707
\ A A A )

(11091.15)2 8591.15)2
A A )

1000 v 1000 x 14.03


-I23.01+ 73.80 =
A A
14030
= 40 MPa (permissible shear stress)
A

2
A = 350.073 mm2 = 4 d

d = 21.11 mm
Rivet diameter from preferred series = 22 mm.

Q.1.9 A compound tube is made by shrinking a thin steel tube on a thin brass tube. The areas of cross-
section of these tubes are As and Ab, while the Young's moduli are Es and Eb respectively. Show
that for any tensile load, the extension of the compound tube is equal to that of a single tube of

E s As + E b Ab
same length and total cross-section area, but having a Young's modulus of
AS + Ab

[CSE-Mains, 2005, ME : 20 Marks]

Solution:
Say P = axial load Steel tube
As, Es
= asAs + abAb

But 5Is = 81b


Brass tube
a Ab, Eb
S1 = ab x L
Es Es

a, Es
Fig.1.8
b Eb
Es Es
P = ab x — x As x ab x Ab = ab[Eb As + At) ]
Eb

[EsAs+AbEb]
P = ob 1 [Es As + AbEb ]
Eb Eb

sib = sis
ab xi
klb = b

81b = a b
1 Eb
P
Strain, fit' = EsAs+EbAb (From eq. (i)
(For single bar having modulus of elasticity E)
P 1
e= x
(As +Ab ) E
8 0. lAS & IFS (Objective & Conventional) Previous Solved Questions MADE EASY

or
Es As + Eb Ab E(A, + Ab )

E= E s As + E b Ab ,
or for a single bar
As + Ab

Q.1.10 A copper tube 22 mm internal diameter, 30 mm outer diameter and 150 mm long is compressed
by a nut tightening over a steel bolt, 20 mm diameter and 1 mm pitch. (i) If the nut is tightened by
a quarter of a turn beyond the just touching position, determine the stress in the bolt. (ii) what
would be the final stress in the bolt if the temperature of the assembly is to increase by 10°C.
Assume E, = 2 x 106 kg/cm2, Ecu = 6 x 106 kg/cm2, as = 12 x 10-6/°C, = 18 x 10-6/°C.
[CSE-Mains, 1999, ME : 30 Marks]

Solution:
2
As area of cross-section of bolt = 4 x 20 = 1 007t mm2
Acu, area of cross-section of copper tube

= .(302 _222 )=1O47c

(i) For equilibrium, compressive force in tube = Tensile force in bolt


ac„ x ,4„ = as x As
acu x 104n = as x 1007c
as = 1.04o
Due to tightening of nut as bolt
Pitch = 1 mm
1
0.25 mm = 4 mm = Extension of bolt + Contraction in tube

6
0.25 mm = sx 150 + a0U x 150
Es E„

0.25 as a„ 1.04a„ a
„ + , (here as and a are in kg/mm2)
150 E, E„ 2x10" 6 x10'

0.25 x103 1.04a„ u„


150 — 20 6
1.6667 = 0.052 a„ + 0.1666 acu= 0.21866 a„
= —7.622 kg/mm2 (comp.)
as = +7.927 kg/mm2 (tensile)
= —762.2 kg/cm2 (comp.)
as +792.7 kg/cm2 (comp.)
=
(ii) Increase in temperature
a >
as
OT= 10°C
Compressive stress will be developed in copper tube due to temperature rate and tensile stress will
be developed in stress bolt
Strain in Cu tube = Strain in steel bolt

Hence, oc„,AT — Er = a. AT GsT


s E
Strength of Materials Simple Stresses in Uniform and Compound Bars 4 9

acur ▪ asr _ (a
or cu - as) AT
E„ E,

acur asT
- (18 -12) x 10-6 x 10 (here asT and Guff are in kg/cm2)
2x106 6x105
a6sT
acur ▪
6 x 10-6 x 105 x 10 = 6
20
But acuT x 104n = sT x 10071
asT = 1.04 acuT
1:5,,,T 1.040,,T
" - 6
20 6
am[0.05 + 0.1733] = 6
acuT -26.866 kg/cm2 (comp.)
a sT = +27.94 kg/cm2 (tensile)
Final stress in steel bolt asT = 792.7 + 27.94 = 820.64 kg/cm2 (tensile)

Objective Questions
Q.1 In a semi-infinite plate shown in the figure 1.9.
The theoretical stress concentration factor kt,
for an elliptical hole of major axis 2a and minor
axis 2b is given by (a)

A
k 2a

(b)
1

Fig. 1.9
0 S
(a) kt = (b) kt = 1+ -a-

(c) = 2b 2a
(d) kt = 1+ — B
kt

[CSE-Prelims, ME : 1998 ] (c)

Q.2 In a simple tensile test, Hooke's law is valid


upto the S
(a) elastic limit
(b) limit of proportionality
(c) ultimate stress
(d) breaking point (d)
[CSE-Prelims, ME: 1998]

Q.3 The stress strain curve for an ideal strain


hardening material will be as in
[CSE-Prelims, ME : 1998]
10 Po. lAS & IFS (Objective & Conventional) Previous Solved Questions MADE EASY

Q.4 The percentage elongation of a material as (c) Potential energy of strain : Body is in a state
obtained from static tension test, depends on of elastic deformation
(a) diameter of the test specimen (d) Hooke's law : Relation between stress and
(b) gauge length of the specimen strain
(c) nature of end grips of the testing machine [CSE-Prelims, ME : 1999]
(d) geometry of the test specimen
0.8 Match the List-I (material properties) with List-II
[CSE-Prelims, ME : 1998]
(technical definition represents) and select the
Q.5 A horizontal force of 200 N is applied at 'A' to correct answer.
lift the load W at C, as shown in figure 1.10. List-I
Value of weight W is A. Hardness
200 N 4 A
B. Toughness
C. Malleability
D. Ductility
E List-I I
1. Percentage elongation
2. Resistance to indentation
3. Ability to absorb energy during plastic
deformation
0.075 m 4. Ability to be rolled into plates
Fig. 1.10 Codes:
(a) 200 N (b) 400 N A BCD
(c) 600 N (d) 800 N (a) 3 2 1 4
[CSE-Prelims, ME : 1998] (b) 2 4 3 1
(c) 2 3 4 1
Q.6 Which one of the following pairs is not correctly (d) 1 3 4 2
matched? [CSE-Prelims, ME : 1999]
If E. Young's modulus
a = coefficient of linear expansion Q.9 A measure of Rockwell hardness is
T= Temperature rise (a) depth of penetration of indentor
A= Area of cross-section (b) surface area of indentation
L= Original length (c) projected area of indentation
(a) Temperature strain with permitted expansion (d) height of rebound
[CSE-Prelims, ME : 1999]
8 is raTL-Sl
L ) Q.10 If a block of material of length 25 cm, breadth
10 cm and height 5 cm undergoes volumetric
(b) Temperature stress — aTE
(c) Temperature thrust — aTEA 1
strain of , then change in volume will be
(d) Temperature stress with permitted 5000
expansion 8 — E(aTL - 8) (a) 0.50 cm3 (b) 0.25 cm3
(c) 0.20 cm3 (d) 0.75 cm3
Q.7 Which one of the following pairs is not correctly
[CSE-Prelims, ME : 2000]
matched?
(a) Uniformly distributed stress: Force passes Q.11 For an isotropic homogeneous and linearly
through centroid of the cross-section elastic material, which obeys Hooke's law, The
(b) Elastic deformation : work done by external number of independent elastic constants are
forces during elastic deformation is fully (a) 1 (b) 2
dissipated as heat (c) 3 (d) 6
Strength of Materials Simple Stresses in Uniform and Compound Bars 1 1 1

Q.12 Assuming E= 180 GPa and G=100 GPa for a


material a strain tensor is
(0.002 0.004 0.006`
0.004 0.003 0 4.
0.006 0 0
Dm-
Shear stress xy is
(a) 400 MPa (b) 500 MPa Codes:
(c) 800 MPa (d) 1000 MPa A BCD
[CSE-Prelims, ME : 2001] (a) 3 1 4 1
(b) 3 2 4 1
Q.13 With the increase of percentage of carbon In
(c) 2 4 3 1
steel, which one of the following properties does
(d) 4 1 3 2
increase
[CSE-Prelims, ME : 2001]
(a) modulus of elasticity
(b) ductility Q.15 Match List-I with List-I I and select the correct
(c) toughness answers.
(d) hardness List-I
[CSE-Prelims, ME : 2001] A. Ultimate strength
B. Natural strain
0.14 Match List-I (material) and List-II (Stress-strain
C. Conventional strain
curve) and select the correct answers.
D. Stress
List-I
List-II
A. Mild steel
1. Internal structure
B. Pure copper
2. Change in length per unit instantaneous
C. Cast iron length
D. Aluminium 3. Change in length per unit gauge length
List-II 4. Load per unit area
Codes:
A BCD
(a) 1 2 3 4
1. (b) 1 3 2 4
(c) 4 3 2 1
(d) 4 2 3 1
[CSE-Prelims, ME : 2002]

Q.16 A steel rod of diameter 1 cm and 1 m long is


a
heated from 20°C to 120°C, its a = 12 x 10-6/K
2. and E= 200 GN/m2. If the rod is free to expand,
thermal stress developed in it is
(a) 12 x 103 N/m2 (b) 240 kN/m2
(c) zero (d) infinite
[CSE-Prelims, ME : 2002]
a Q.17 Consider the following statements:
3. 1. There are only two independent elastic
constants
2. Elastic constants are different in orthogonal
directions
12 . lAS & IFS (Objective & Conventional) Previous Solved Questions MADE EASY

3. Material properties are same everywhere


4. Elastic constants are same in all loading
conditions
5. The material has ability to withstand shock
loading
Which of the above statements are true for a
linearly elastic homogeneous and isotropic
material?
(a) 1, 3, 4 and 5 (b) 2, 3 and 4
(c) 1, 3 and 4 (d) 2 and 5
[CSE-Prelims, ME : 2002]

Q.18 A cast iron specimen in a torsion test gives a Fig. 1.12


(a) cup and cone fracture (a) 70.7 N (b) 100 N
(b) fracture along a plane normal to the axis of (c) 141.4 N (d) 108 N
the specimen [CSE-Prelims, ME : 2003]
(c) fracture along a helix of approximately 45°
Q.22 A mild steel specimen is tested in tension up
(d) fracture along a plane inclined at 60° to the
to fracture in a Universal Testing Machine. Which
axis
of the following mechanical properties of the
[CSE-Prelims, ME : 2002]
material can be evaluated from such a test?
Q.19 The reactions at the rigid support A and B for 1. Modulus of elasticity
the bar loaded as shown in figure 1.11 are 2. Yield stress
respectively 3. Ductility
4. Tensile strength
5. Hardness
6. Modulus of rigidly
Select the correct answer using the code given
below:
(a) 1, 3, 5 and 6 (b) 2, 3, 4 and 6
Fig. 1.11 (c) 1, 2, 5 and 6 (d) 1, 2, 3 and 4
[CSE-Prelims, ME : 2007]
0 kN 10 kN 10 kN 20 kN
(a) (b) Q.23 A heavy uniform rod of length 'L' and material
3 3 3 3
density '8' is hung vertically with its top end
(c) 5 kN, 5 kN (d) 6 kN, 4 kN rigidly fixed. How is the total elongation of the
Q.20 A steel rod 10 mm in diameter and 1 m long is bar under its own weight expressed?
heated from 20°C to 120°C, E = 200 GPa and (a) 28L2g 8L2g
a =12 x 10-6/°C. If the rod is not free to expand, (b)
E E
the thermal stress developed is
8L2g 61_2g
(a) 120 MPa (tensile) (b) 240 MPa (tensile) (d)
(c) 120 MPa (comp.) (d) 240 MPa (comp.) (c) 2E
[CSE-Prelims, ME : 2003] [CSE-Prelims, ME : 2007]

Q.21 The force F is such that the bars AC and BC Q.24 A round bar length 1, elastic modulus E and
(AC and BC are equal to length) as shown in Poisson's ratio ji is subjected to an axial pull
the figure are identically loaded is 'P. What would be the change in volume of the
bar?
Strength of Materials Simple Stresses in Uniform and Compound Bars 41 13

P1(1— 2µ) Q.30 Consider the following statements relating to


Pl
(a) (b) E Rockwell Hardness Testing Method:
(1— 21.0E
1 It is a quick method to determine hardness
P1µ P/ of industrial components.
(c) (d) —
ptE 2. Polishing of test sample is necessary.
[CSE-Prelims, ME : 2007] 3. Test load is applied in two stages.
4. Hardness of relatively softer metals cannot
Q.25 Which one of the following material has the
be determined by this method.
highest ultimate tensile strength
5. Inverted pyramid type of indenter is used.
(a) mild steel (b) cast iron
Which of the statements given above are
(c) spring steel (d) wrought iron
correct?
[CSE-Prelims, ME : 2007] (a) 1, 2, 3 and 4 (b) 3, 4 and 5
Q.26 A straight uniform rod is subjected to axial load. (c) 1, 2 and 3 (d) 2, 4 and 5
Which one of the following is the correct [CSE-Prelims, ME : 2006]
statement? 0.31 If a material has numerically the same value for
(a) It induces maximum shearing stress on a its modulus of rigidity and bulk modulus, then
transverse plane what is its Poisson's ratio?
(b) It induces maximum normal stress on a (a) 0.25 (b) 0.2
plane inclined at 45° to the axis of the rod (c) 0.15 (d) 0.125
(c) It induces maximum shear stress on a plane [CSE-Prelims, ME : 2009]
inclined at 45° to the axis of the rod
(d) It induces zero shear stress on any inclined Q.32 A free bar of length 1 is heated uniformly from
plane to the axis of the rod 0°C to a temperature T°C. a is the coefficient of
linear expansion and E is the modulus of
[CSE-Prelims, ME : 2006]
elasticity. Which one of the following is the stress
Q.27 Which one of the following is rupture stress? induced in the bar?
(a) Breaking stress
aTE aTE
(b) Maximum load/original cross-sectional (a) (b)
4 2
area (A)
(c) Load at breaking point/A (c) aTE (d) Zero
(d) Load at braking point/neck area [CSE-Prelims, ME : 2009]
[CSE-Prelims, ME : 2006] Q.33 What property of a material enables it to be
Q.28 A 2 m long rod of diameter 2 mm is subjected drawn into wires with the application of tensile
to an axial pull of 1 kN. The rod is extended by force?
0.5 cm. What is the approximate value of the (a) Plasticity (b) Elasticity
modulus of elasticity of the material of the rod? (c) Ductility (d) Malleability
(a) 127 G Pa (b) 180 G Pa [CSE-Prelims, ME : 2009]
(c) 125 G Pa (d) 200 G Pa Q.34 ABC is a rigid bar. It is hinged at A and
Q.29 Which one of the following information cannot suspended at B and C by two wires, BD and
CE made of copper and steel respectively, as
be obtained from the static tensile test of a mild
shown in the given figure. The bar carries a load
steel specimen?
of 10 kN at F, midway between Band C. Given
(a) Modulus of elasticity
that
(b) Qualitative determination of toughness
A,-= 4 cm2
(c) Ductility
Aa = 2 cm2
(d) Weldability
= 1 x 105 N/mm2
[CSE-Prelims, ME : 2006]
ES = 2 x 105 N/mm2
14 0. IAS & IFS (Objective & Conventional) Previous Solved Questions MADE EASY

Subscript c and s stands for copper and steel. Q.38 A bar of uniform cross-section of 400 mm2 is
If the extensions in the steel and copper wires loaded as shown in figure. The stress at section
are As and As respectively, the ratio As/Ac would 1-1 is
be
71
40 kN 20 kN 10 kN 30 kN
-0. 4E-- ----0,- 4-
I

I 1
400 mm 300 mm 400 mm

m ,-1 1
. m---1
-- Fig. 1.14
Fig. 1.13 (a) 50 N/mm2 (b) 100 N/mm2
(a) 1/4 (b) 4 (c) 25 N/mm2 (d) 200 N/mm2
(c) 2 (d) 1/2 [CSE-Prelims, CE : 2003]
[CSE-Prelims, CE : 2000] Q.39 As soon as the external forces causing
Q.35 Match List-I (Material) with List-II (Modulus of deformation in a perfectly elastic body are
elasticity N/mm2) and select the correct answer: withdrawn, the elastic deformation disappears
List-I List-II (a) only partially
A. Steel 1. 0.6 x 105 (b) completely over a prolonged period of time
B. Cast iron 2. 1 x 105 (c) completely and instantaneously
C. Aluminium 3. 2 x 105 (d) completely after an initial period of rest
D. Timber 4. 0.1 x 105 [CSE-Prelims, CE : 2003]
Codes: Q.40 Match List-I (Mechanical property) with List-II
A B C D (Feature) and select the correct answer.
(a) 3 2 1 4 List-I
(b) 2 3 1 4 A. Creep
(c) 3 2 4 1 B. Tenacity
(d) 2 3 4 1 C. Ductility
[CSE-Prelims, CE : 2001] D. Brittleness
Q.36 The phenomenon of decreased resistance of a List-II
material due to reversal of stress is called 1. Amenability to go through changes of shape
(a) resilience (b) elasticity without rupture
(c) creep (d) fatigue 2. Susceptibility to deform with time under
[CSE-Prelims, CE : 2002] sustained loading
3. Ability to be drawn into wire
Q.37 Total elongation of a prismatic bar of length L, 4. Susceptibility to fail suddenly without
weight W, cross-sectional area A and modulus warning
of elasticity E, under its own weight while Codes:
hanging vertically, is A BCD
WL 2WL (a) 4 1 3 2
(a) (b)
AE AE (b) 2 3 1 4
WL WL (c) 4 3 1 2
(c) (d) (d) 2 1 3 4
2AE 3AE
[CSE-Prelims, CE : 2002] [CSE-Prelims, CE : 2003]
Strength of Materials Simple Stresses in Uniform and Compound Bars 4 15

Q.41 If P is the direct load, L is the length of the


member, A is the uniform area of cross-section
and E is the Young's modulus; then strain energy
due to direct stress caused by a gradually
applied load is

P 2L P 2L
(a) 2AE (b) 2 A2E

PL PL
(c) 2AE (d) AE
[CSE-Prelims, CE : 2003]
Fig. 1.15
Q.42 Consider the following statements regarding
(a) 16.67 N/mm2 (b) 13.33 N/mm2
tensile test diagrams for carbon steels with
(c) 26.67 N/mm2 (d) 30 N/mm2
varying carbon contents;
[CSE-Prelims, CE : 2007]
As the carbon content increases
Q.47 In a pure tensile member, the normal stress on
1. the ultimate strength of steel decreases
a plane at right angles to the direction of load
2. the elongation before fracture increases
is 100 N/mrn2. What is the normal stress at a
3. the ductility of the metal decreases
plane whose normal is inclined at 60° to the
4. the ultimate strength of steel increases
direction of the load?
Which of the statements given above are
(a) 75 N/mm2 (b) 100 N/mrn2
correct?
(c) 125 N/mm2 (d) 150 N/mm2
(a) 3 and 4 (b) 1 and 3 [CSE-Prelims, CE : 2008]
(c) 1, 2 and 3 (d) 1 and 2
Q.48 How is the maximum strain energy stored per
[CSE-Prelims, CE : 2005]
unit volume in a body without permanent
Q.43 What is the maximum possible value of distortion, termed as?
Poisson's ratio for a non-dilatant material? (a) Modulus of resilience
(a) 0.67 (b) 0.50 (b) Modulus of tenacity
(c) 0.33 (d) 0.25 (c) Modulus of toughness
[CSE-Prelims, CE : 2005] (d) Proof resilience
[CSE-Prelims, CE : 2008]
Q.44 The shear modulus of a material is half of its
Young's modulus. What is the value of its Q.49 A cable is supported at P and Q in which P is
higher than Q. At what point(s) among the
Poisson's ratio?
following is the tension in the cable maximum?
(a) -1 (b) -0.5
(c) zero (d) 0.5
[CSE-Prelims, CE : 2006]

Q.45 What is range of values of Poisson's ratio for


ductile materials?
(a) 0.10 to 0.15 (b) 0.16 to 0.20
(c) 0.21 to 0.24 (d) 0.25 to 0.33
[CSE-Prelims, CE : 2006]

Q.46 What is value of stress at the base CC for the


bar shown in the figure given below?
Fig. 1.16
16 0. lAS & IFS (Objective & Conventional) Previous Solved Questions MADE EASY

(a) Ponly (b) Q only Q.52 At a certain stage under elastic loading, the
(c) R only (d) P and 0 elongation observed was 0.03 mm, the gauge
[CSE-Prelims, CE : 2008] length was 150 mm and the modulus of
elasticity was 2 x 105 N/mm2. What was the
Q.50 In the following pin jointed truss, what is the
stress at that location?
displacement of support B due to the given
(a) 4 N/mm2 (b) 40 N/mm2
load?
(c) 80 N/mm2 (d) 60 N/mm2
5005 kN
[CSE-Prelims, CE : 2009]

Q.53 Modulus of elasticity and Poisson's ratio of a


material are 2.1 x 105 N/mm2 and 0.25
respectively. What is the value of modulus of
rigidity of the same material in 105 N/mm2?
(a) 0.84 (b) 0.70
(c) 1.40 (d) 0.50
[CSE-Prelims, CE : 2010]
Fig. 1.17
Answers
(Cross-sectional area of each member 1. (d) 2. (b) 3. (d) 4. (b) 5. (d)
= 500 mm2, modulus of elasticity E = 2 x 105
6. (d) 7. (b) 8. (c) 9. (a) 10. (b)
N/mm2)
(a) 3.25 mm (b) 2.50 mm 11. (b) 12. (c) 13. (d) 14. (b) 15. (a)
(c) 1.50 mm (d) 0.50 mm 16. (c) 17. (c) 18. (c) 19. (a) 20. (d)
[CSE-Prelims, CE : 2009]
21. (a) 22. (d) 23. (d) 24. (b) 25. (c)
Q.51 What is the normal stress on a plane inclined at 26. (c) 27. (d) 28. (a) 29. (d) 30. (c)
45° to the axis of a square rod of side a
31.(d) 32. (d) 33. (c) 34. (c) 35. (b)
subjected to an axial tensile force of T?
36. (d) 37. (c) 38. (b) 39. (b) 40. (d)
T T
(a) 2 (b) 2a 41.(a) 42. (a) 44. (c) 45. (d)
a 2 43. (b)
T 46. (d) 47. (a) 48. (a) 49. (a) 50. (b)
(c) (d) 2
4a2 8a 51.(b) 52. (b) 53. (a)
[CSE-Prelims, CE : 2009]

Explanations
1. (d) without having holes, acts, shoulders or narrow
2a passes.
kt = 1+---
b 2. (b)
2a is major axis perpendicular to axis of loading.
Stress contraction factor (kt) is a dimensionless but
ae
factor which is used to quantity how concentrated aP
abreaking
the stress is in a material. It is defined as the
ratio of the highest stress in the element to the
ame
reference stress.
°max
k=
r aref
Simple tensile stress
Reference stress is the stress in the part within
an element under the same loading conditions Fig. 1.18
Strength of Materials Simple Stresses in Uniform and Compound Bars 44 17

Hooke's law valid only upto '


limit of 11. (b)
proportionality. Eand [t are two independent elastic constants.

3. (d) 12. (c)

shear strain = 0.004


2
a y = 0.008
G= 100 x 1000 N/mm2
ti~ = yxy x 4
= 0.008 x 100 x 1000 N/mm2
= 800 MPa

13. (d)
Fig. 1.19
With the increase of carbon percentage in steel.
OA = elastic
Hardness of steel does increase.
AB= strain hardening portion
14. (b)
4. (b)
Mild steel - 3, Pure copper - 2, Cast iron - 4,
6L
% elongation - Aluminium - 1.
gauge length
15. (a)
change in length
1. A. internal structure
gauge length
2. B. 6//instantaneous length
5. (d) 3. C. 6L/L
200 x 0.3 = 0.075 W 4. D. load per unit area
W= 800 N (moment about B) 16. (c)
6. (d) Rod is free to expand, no stress will be
is not correctly matched. developed.
Temperature stress with permitted explain is not 17. (c)
E(aTL- 6). 1, 3 and 4 are correct statements.

7. (b) 18. (c)


Elastic deformation work done by external forces A cast iron specimen in a torsion test fails along
during elastic deformation is not dissipated fully a helix approx at 45° to the axis due to tensile
as heat. principal stress p1 = +T(shear)

8. (c) 19. (a)


A. Hardness 2. Resistance to inductation
10 kN
B. Toughness 3. Ability to absorb energy 1
4- P2
during plastic
C. Malleability 4. Ability to be rolled into plates k- 1m 2m

D. Ductility 1. Percentage of elongation Fig. 1.20

9. (a) P1 x1 =P2 x 2
A measure of Rockwell hardness is a depth of AE AE
penetration of indentor.
P1 = 2P2
10. (b) P1 + P2 = 1 0

25 x10x 5 20
6V- - 0.25 cm3
5000 - 3
18 •• IAS & IFS (Objective & Conventional) Previous Solved Questions MADE EASY

20. (d) 29. (d)


TE Weldability cannot be determined by static tensile
= 12 x 10-6 x 100 x 200 x 103 test.
= 240 MPa (comp.) 30. (c)
Because expansion is prevented. 1, 2, 3 are correct statements.
21. (a) 31. (d)
Bars AC and BC are identically loaded.
FAC = FBC 2(1+v) 3(1-2v)
Horizontal component of FAc and FBc are cancelled.
2FAB cos 30° = 100 cos45° 2 + 2v= 3 - v
1 =8v
Horizontal component of 100 N = 70.7 N is
v=0.125
balanced by F
32. (d)
So F= 70.7 N
Bar is free to expand no stress is developed.
22. (d)
33. (c)
1, 2, 3, 4 are evaluated by tension test on MS.
Ductility enables the wire to the drawn.
23. (d)
34. (c)
Since ABC is rigid, deflection will be linear.
(pg) 2
8L due to self weight = L A
2E
8,
24. (b)

2µ) x IA 1-1 m I 1m-►I


8V= evV =
Fig. 1.21

8s
-=2, Similar triangles
Sc

35. (a)
A. Steel 3. 2 x 105 N/mm2
B. Cast iron 2. 1 x 105 N/mm2
25. (c) C. Aluminium 1. 0.6 x 105 N/mm2
Spring steel is a high carbon steel, it has highest D. Timber 4. 0.1 x 105 N/mm2
out amongst the material gives.
36. (d)
26. (c) Fatigue
Axial load on uniform straight bar induces
37. (c)
maximum shear on a plane inclined at 45° to the
axis of the rod.

27. (d)
Rupture load at breaking point/neck area.

28. (a)

1000 2000
E= x
TC 0.5x10
=1.27 x 105 N/mm2
=127 GPa Fig. 1.22
Strength of Materials Simple Stresses in Uniform and Compound Bars 19

Extension due to own weight Ebb = craa x (a)2 = 100 x (0.861)2


wL = 75 N/mm2
8-
2AE 48. (a)
38. (b)
Maximum strain energy per unit volume without
40000
611 = = 100 N/mm2 permanent distortion = Modulus of resilience.
400
39. (b) 49. (a)
Elastic deformation disappears completely over P only, maximum slope of cable at P.
a prolonged period of time. 50. (b)

40. (d) 500 x,FakN

A. Creep 2. Susceptibility to deform


with time under
sustained loading
B. Tenacity 1. Amenability to go
through changes of
A
shape without rupture
250 xsffkN 250 x5- kN
C. Ductility 3. Ability to be drawn into
wire Fig. 1.24
D. Brittleness 4. Susceptibility to fail Extension in AB - Components of contraction
suddenly without in CA and CB
warning 250 x 103 1000 500 x 103 x 1000
8B = x 2 x x cos 60°
500 2 x 105 500 x2 x105
41. (a)
= +2.5 - 5 = -2.5 mm or 2.5 mm
1 PL P2L
Strain energy = -P x = 51. (b)
2 AE 2AE
n
42. (a) \<
7 45°
(3) and (4).
n
43. (b) Fig. 1.25
Maximum possible value of Poisson's ratio for a T
non-dilatant material is 0.5. 6a= a

44. (c) 2 T
abb-
- -7---X(COS45) .
1 E a2 2a 2
G= -E = , so, v = 0
2 2(1+v) 52. (b)
8L= 0.03 mm
45. (d)
L= 150 mm
v = 0.25 - 0.33, for ductile materials.
E= 2 x 105 N/mm2
46. (d)
a= LL xE _ 0.03
150
x2x105
(20+25)1000
0cc - = 30 N/mm2 = 40 N/mm2
1500
53. (a)
47. (a)
E= 2.1x 105 N/mm2, v = 0.25
b
60° 3 E 2.1x105
100 G= 2(1+0.25) - 2.5
b
= 0.84 x 105 N/mm2
Fig. 1.23 11111111•
02
CHAPTER
Principal Stresses

Q.2.1 At a section in a beam the tensile stress due to bending is 50 N/mm2 and there is shear stress of
20 N/mm2. Determine from first principles, the magnitude and direction of principal stress and
calculate the maximum shear stress
[IFS, 2012, ME : 10 Marks]

Solution:

50

20 (F1 + Q2)

F1

Q2
(a) (b) (c)

Fig. 2.1

Figure shows an element ABC unit of thickness subjected to normal and shear stresses as given. On
plane BC there is complementary shear stress of 20 N/mm2 as shown in figure 2.1(a).
Normal force, F1 = 50 x AC
Shear force, Q1 = 20 x AC
Shear force, Q2 = 20 x BC
Normal force on inclined plane AB
Fr,_ (F1 + Q2 ) cos0 + Q1 sin°
On x AB = (F1 + Q2) cos() + Q1 sine
or on x AB = 50 AC cos0 + 20 BC cos° + 20 AC sine
or an = 50 cos2O + 20 sine cos() + 20 cos() sin()
50 cos20 + 20 sin 20
Ft tangential force on plane AB
x AB = (F1 + Q2 ) sin° - 1 cos°
sine 02 sine Q1 cos()
or To =
AB AB AB
Note from figures, that internal resistance is equal and opposite to the applied forces on inclined plane.
Strength of Materials Principal Stresses 4 21

Putting the values of F1, Qi, 02


50 x ACsin0 20 xl3Csin0 20 x ACcos0
tie -
AB AB AB
= 50 sin0 cos° + 20 sin20 - 20 cos20
50
= — x sin 20 - 20 cos 20 = 25 sin 20 - 20 cos 20
2
Principal planes
On principal planes, to, shear stress is zero
So, 25 sin20 - 20 cos20 = 0
tan20 = 0.8
201 = tan-10.8 = 38.66°
01 = 19.33°
Another plane, 02 = 90 + 19.33 = 109.33°
Principal stresses
01 = 19.33, cos01 = 0.943.6
cos201 = 0.890
sin201 = 0.6247
Principal stress, p1 = 50 x 0.8904 + 20 x 0.6247
44.52 + 12.494 = 57.016 MPa
02 = 109.33°, cos02 = -0.3310, cos202 = 0.10956
sin202 = -0.6247
Principal stress, p1 = 50 x 0.10956 + 20 x (-0.6247)
= 5.478 - 12.494 = -7.016 MPa
Principal stresses are + 57.016, - 7.016 MPa
Principal directions are 19.33°, 109.33°

For maximum shear stress, 0, de = 0

25 x 2 cos 20 + 20 x 2 sin 20 = 0
0 = -25.67° or 64.33°
To = 25 x sin (-2 x 25.67°) - 20 x cos (-2 x 25.67°)
= -19.521 - 12.495 = - 32.016 Mpa
It9I = 32.016 MPa

Q.2.2 Draw Mohr's circle for a 2-dimensional stress field subjected to


(i) Pure shear (ii) Pure biaxial tension
(iii) Pure uniaxial compression (iv) Pure uniaxial tension
[CSE-Mains, 1997, ME : 15 Marks]

Solution:

= +T
OB = -T
ti B - ti
22 ► lAS & IFS (Objective & Conventional) Previous Solved Questions MADE EASY

Air - Radius
2
a 6
0
OA =
Pure Bi-axial tension
OB= (72
Cf + 6
oc = '
2 '

R=

Pure uniaxial compression OA =


OC = --cv/2

(iv)

Pure uniaxial tension

Fig. 2.2

Q.2.3 For the following state of stress, show the stresses on two given planes at right angle of an
element. Find the magnitude and directions of principal stress and the maximum shear stresses
in each case.
(i) Simple uniaxial tension
(ii) Pure equal normal stresses on given planes
(iii) Pure shear stresses on given plane
[CSE-Mains, 2002, ME : 20 Marks]

Solution:

(a)
Fig. 2.3 (a)
(i) Simple uniaxial tension.
Principal stress, p1 =
p2 = p3 = 0
p1 along x-axis.

—P1
/91 —P2
Maximum shear stress =
2 2
Strength of Materials Principal Stresses i 23

(b)
Fig. 2.3 (b)
Pure equal normal stresses , p1 = p2 = a (along x and y direction)
p3 = 0 (along z direction)
P1 P2
Maximum shear stress - =0
2
(iii) Mohr's circle diagram will be a point on +x axis with coordiantes (a, 0)

(c)
Fig. 2.3 (c)
Pure shear stresses on given planes
p1 = +T, P2 =
01 = 45°, 02 = —45°
p3 = 0
Maximum shear stress = ti
Note: In case (ii) Mohr circle diagram will be a point on (+ve) x-axis, having coordinate (a, 0)
Q.2.4 At a point in an elastic material the stresses on three mutually perpendicular planes are
First Plane : 50 MN/m2 tensile and 40 MN/m2 shear
Second Plane : 30 MN/m2 compressive and 40 MN/m2 complementary shear
Third Plane : No stress
Find: (i) The position of principal planes and magnitude of principal stresses
(ii) The position of the planes on which maximum shear stress act, calculate normal and
shear stress on them.
[CSE-Mains, 1992, ME : 20 Marks]
Solution:

Fig. 2.4
24 P. lAS & IFS (Objective & Conventional) Previous Solved Questions MADE EASY

(i) BC and AC are two mutually perpendicular planes. Reference plane BC


1 = +50 N/mm2
c = +40 N/mm2
Plane AC
62 = -30 N/mm2
ti = +40 N/mm2

61 10, al 262
-
2 62 - - 40

Principal stresses

P1 = 10-4402 +402 = 10 + 56.56 = 66.56 N/mm2

P2 = 10_J402 + 402 = 10 - 56.56 = -46.56 N/mm2

12 x 40
20 = tan-1 2T = tan- = tan-1(1) = 45°
1 al a2 80

61 = 22.5°, 02 = 112.5°

Tmax = V402 + 402 = 56.56 N/mm2

(ii) Angle of plane of Tam


03 = 0l+ 45° = 67.5° w.r.t plane BC
04 = 67.5 + 90° = 157.5° (wrt plane BC)
Normal stresses on planes of maximum shear stress, 'um,

61 + az
- - 10 N/mm2
2
On both shear planes with +Tricia, and -Tma,

Q.2.5 At a point in a material under stress, the intensity of the resultant stress on a certain plane is
50 MN/m2 (tensile) and inclined at 30° to the normal of that plane (Fig. 2.5). The stress on a plane
at right angle to this plane has a normal tensile component of intensity 30 MN/m2.
Find: (i) resultant stress on second plane
(ii) principal planes and stresses
(iii) maximum shear stress and the plane on which it occurs

Solution:
Shear stress on plane BC = 50 sin 30° = 25 MPa
Normal stress on plane BC = 50 cos 30°
a1= 43.3 MPa
Complementary shear stress on plane AB
ti = 25 N/mm2 (as shown)
50 MPa
Resultant stress on plane AB = V302 + 252
= 39.05 N/mm2 = 39.05 MPa
62, normal stress on plane AB = 30 MPa

(ii) al +62 433+ 3° = 36.65 MPa


2 2 Fig. 2.5
Strength of Materials Principal Stresses 4 25

al — a2 43.3 — 30
= 6.65 MPa
2 2

ai — G2 )2 T_2 _
652 252 25.87 MPa
( 2
2
(31 + G2 + (G1 G2) +T2
Principal stresses P1' 1-'2 2 2
p1 = 36.65 + 25.87 = 62.52 MPa
p2 = 36.65 — 25.87 = 10.78 MPa
Principal angle (wrt plane BC)
1 2T 1 _12 x 25
0 = — tan = tan = 37°33'
1 2 cy1—a2 2 13.3
02 = 90 + 61 = 127°33'
(iii) Maximum shear stress
max = ±25.86 MPa
03 = 01 + 45° = 37°33' + 45° = 82°33'
04 = 03 + 90° = 172°33'
2
20 N/mm
Q.2.6 The figure given below shows the state of stress at a
point namely 30 N/mm2 tensile in x-direction, 20 N/mm2 50
tensile in y-direction and shear stress of 50 N/mm2.
Find the position of principal planes, principal stresses 30 50
2 2
and maximum shear stress graphically or other wise. 30 N/mm 30 N/mm
[CSE-Mains, 2000, ME : 20 Marks]
50
Solution:
Let us take plane AB as reference plane on which shear
D
stress is negative shear stress 50 4
cr / = 30 N/mm2, a2 = 20 N/mm2, ti = 50 N/mm2
20
— 25 Fig. 2.6

— 5 N/mm2

2
+ G2 (01 — 02 ) +ti2
Principal stresses P1 ' P3 = = 25± V52 + 502 = 25± 50.25
2 2
p1 = +75.25 N/mm2
p2 = —25.25 N/mm2
Angle of principal plane w.r.t plane AB (in anti-clockwise direction)
_1 2T = 1 1 2 x 50
0 = -tan _1 2 tan- = 42°8'41"
1 0 G2 10
02 = 01 + 90° = 132°8'41"
2
l'Crinaxl maximum shear stress = ( al — (32 ) T 2 = 50.25 N/mm2
2
Tina, will occur at angle, 03 = 01 + 45° = 87° 8' 41"
and 04 = 03 + 90° = 177° 8' 41"
26 . lAS & IFS (Objective & Conventional) Previous Solved Questions MADE EASY

Q.2.7 A circle of 10 cm diameter is inscribed on a steel plate 2


17 12 kN/cm
before if is stressed. Then the plate is loaded so as to
produce stresses as shown in the figure 2.7 and circle
is deformed into an ellipse. Determine the length of
major and minor axes of the ellipse and their directions.
Take modulus of elasticity of steel as 2.1 x 107 N/cm2
6
and Poisson's ratio = 0.3.
P2
[CSE-Mains, 2004, ME : 20 Marks]
Solution:
E = 21000 kN/cm2
Taking AB as reference plane 2
12 kN/cm
= +12 kN/cm2
a2 = -6 kN/cm2 Fig. 2.7
ai +432 - 3 kN/cm2
2
al - 02 - 9 kN/cm2
2
ti = 4 kN/cm2
2
- 2 2
+tie - V9 + 4 2 --9.849 kN/cm
a2
2
p1 = 3 + 9.849 = 12.849 kN/cm2
p2 = 3 - 9.849 = -6.849 kN/cm2
Principal angle (w.r.t. in plane CD in anti-clockwise direction)
1 1
01 = --tan- = 1 = 12°
2 ai --a2 2 tan-12184
02 = 102°
vp2 12.849 0.3 x 0.849 _ 14.9037
Strains, +
E E 21000 21000 21000
-6.849 0.3 x 12.849 10.7037
E2 - 21000 21000 21000
= 0.70797 x 10-3
e2 = -0.5097 x 10-3
(In the direction of major principal axis)
Maximum axis = 100 + 0.70797 x 10-3 x 100 = 100.0709 mm
(In the direction of minor principal)
Minimum axis = -100 - 5097 x 10-3 + 100 = 99.949 mm

Q.2.8 At a point in a strained material, planes AB and AC pass through the point A as shown in the
figure 2.8(a). Normal and shear stresses on plane AB are 30 MPa and 50 MPa respectively as
shown. Normal and shear stress on plane AC are 60 MPa and 20 MPa respectively as shown. By
graphical or analytical method, determine angle between the planes AB and AC.
[CSE-Mains, 2001, ME : 30 Marks]

Solution:

Using sign convention and


Strength of Materials Principal Stresses 1 27

D
60
(30, +50)
(a)

E
OC = +30 Refer
CD = +50
OA = +60 (60, — 20)
AB = —20

Mohr's stress circle


29 = 90°, 0 = 45°
(b)
Fig. 2.8
Note: In shear stress produce clockwise couple at the center of element then it will be plotted above ax
(+ve) and if shear stress causes anticlockwise couple at the center then it will be plotted below a axis.

Q.2.9 (a) Under which condition of the state of stress at a point in the two dimensions, the Mohr's
circle will be reduced to a point.
(b) How much change in volume would a 100 mm side cube of steel will have when it is kept at
a depth of 2 km in sea water?
Assume specific gravity of sea water equal to 1.02, modulus of elasticity equal to 2.08 GPa and
Poisson's ratio equal to 0.29.
[CSE-Mains, 2007, ME : 20 Marks]
Solution:
(a) Mohr's stress circle will be reduced to a point
If (T = ay =
x
or ax = 0y = Cr as shown

fa

X • rX
, 0)
(—a. (a, 0)

Fig. 2.9
28 0. IAS & IFS (Objective & Conventional) Previous Solved Questions MADE EASY

(b) Depth, h= 2 km
Weight density, y= 1.02 x 1 x 10-5 N/mm2
p, pressure = yh = 1.02 x 10-5 x 2000 x 1000 N/mm2 = 20.4 N/mm2
E 2.08 x105
K, bulk density = -1.65 x105 N/mm2
3(1-2v) 3(1-0.29 x 2)

p 20.4
Volumetric strain, 1.65x105 12.357x 10-5
Ev K=
V = 1003 = 106 mm3
Change in volume, 8V = 12.357 x 10-5 x 106 = 123.57 mm3

Q.2.10 The normal stresses at a point in a strained material across two planes at right angles of each
other are 120 N/mm2 tensile and 60 N/mm2 compressive. The shear stress on these planes is
40 N/mm2. Find
(i) Principal stresses and principal planes
(ii) The direct and shear stresses on a plane inclined at 30° to the vertical
[CSE-Mains, 1998, ME : 30 Marks]

Solution:
Let us take BC as reference plane
al = +120 MPa, a2 = -60 MPa, t = -40 MPa on BC
a' 2 a2 (31 CF2
- 30 MPa - 90 MPa
2 ' 2

Pi, P2 = 30 ± V902 + 402 = 30 ± 98.49


p1 = 128.49 MPa
p2 = -68.49 MPa
1 12T
9 ,_ -tan (w.r.t palne BC in anticlockwise direction)
1 2 al 0,2
12 x 40 1
= 1tan- = x 24° = 12°
2 180 2
02 = 90 + 12° = 102°
Reference plane BC
Inclined plane AB
0 = 60° (angle from plane BC)

al 1- a2 a1 - a2 cos120°+ tsin120° co
Normal, a - 0_
o 2 + 2 m
0
co
= 30 + 90 cos120° + 40 sin120° 1
II
= 30 - 45 + 34.64 = 19.64 MPa T 0"

40 MPa
Shear stress, T = (a1 - a2 )sin120°-tcos120°
o 2

= 90 sin 120°- 40 x cos 120°


= 90 x 0.866 - 40(-0.5) T
C
= 77.94 + 20 = 97.94 MPa al= + 120 MPa
IT' = 97.94 MPa Fig. 2.10
Strength of Materials Principal Stresses 29

Q.2.11 Two planes AB and AC make an angle of 50° at point


A. Plane AB is subjected to tensile stress of 3 kN/cm2
and a shear stress 3 kN/cm2 from B toward A. Plane
AC neglected to a shear stress of magnitude AB — Ref. Plane
2 kN/mm2 (from C towards A) and an normal stress
of unknown magnitude. Determine (i) Normal stress
on plane AC (ii) Principal stresses.
[IFS 2011, ME : 15 Marks]

Solution:
Stress on plane AB
a = +30 N/mm2 = 3 kN/cm2
ti = -30 N/mm2 = -3 kN/cm2
AB is reference plane shear stress on reference plane is
D
negative. Consider a plane AD perpendicular to plane
AB, say normal stress on this plane is a2. Complementary Fig. 2.11
shear stress on this plane is +30 N/mm2.
Plane AC

= 61 - 62 sin20 --tcos20
2
0 = -50°

20 - alsin(-100°)-30cos(-100°)
2

(30 -
a2)( 0 9848)- 30 (-0.17365)
2
= -14.772 + 0.4924 a2 + 5.20945
20 +14.772 - 5.20945
62 - = 60.038 N/mm2
0.4924
(i) Normal stress on plane BC
61
a - 62 4. al a2 cos(-100°) + Tsin(-100°)
2 2

30 + 60.038 30 - 60.038
x cos(-100°)+ 30 x sin(-100°)
2 2
= 45.019 - 15.019 (-0.173648) + 30 (-0.9848)
= 45.019 + 2.6080 - 29.544
= 18.083 N/mm2 (Normal stress on plane AC)
(ii) Principal stresses
a1 = 30 N/mm2
a2 = 60.038 N/mm2

01 + 02 —
+45.019 N/mm2
2

61 2G2 - -15.019 N/mm2

T = 30 N/mm2
30 0. lAS & IFS (Objective & Conventional) Previous Solved Questions MADE EASY

2
(ai - az ) +,r2
= V(-15.019)2 + 302 = ±33.55 N/mm2
2

P1 —
al + az +33.55 = 45.019 + 33.55 = 78.519 N/mm 2
2
01+ 02 + 33.55 = 45.019 - 33.55 = 11.519 N/mm 2
P2 = 2
Q.2.12 A1 m x 1m mild steel shell of 1 mm thickness is stretched in its own plane by stresses, ; = 20 MPa,
ay = 30 MPa as shown in figure 2.12. Point 0 is centre of the plane. OC and OD are two mutually
perpendicular lines inclined at 45° to x and y-direction respectively before application of stresses.
Determine the change in angle (in degrees) between OC and OD after application of stresses.
Take modulus of rigidity of plate material = 80 GPa.

ax = 20 MPa
0

30 MPa
ay = 30 MPa

Fig. 2.12
[CSE-Mains, 2011 , ME : 20 Marks]
Solution:

t, shear stress on plane OC - 30 - 20 sin 90° = +5 MPa


2
Shear stress on plane OD = -5 MPa
Change in angle of 90° between OD and OC
5 5
(I)
xy - = rad=0.0036° 30 kN 1r 500 mm 30 kN
G 80000 ;150

Q.2.13 A steel bracket is bolted a column by three similar


bolts as shown in the figure 2.13. It is subjected to
an eccentric load of 30 kN as indicated. Determine
the suitable minimum diameter for the bolts using 300 mm
a factor of safety of 5.0. Assume the bracket and
the attached plate to be rigid. Take the allowable 1

stresses for the bolt material in tension as 100 mm


600 MN/m2 and in shear as 350 MN/m2.
[CSE-Mains, 1996, ME : 30 Marks]
Fig. 2.13
Strength of Materials Principal Stresses 4 31

Solution:
Load = 30 kN
Load is eccentric about two axes, x and y

10000
Direct shear stress on each bolt — Nimm2 (.0
A
Shear stress due to 30,000 x 150 Nmm moment will be maximum at bolt E, but ab due to moment
30,000 x 500 Nmm will be maximum at both D.
Shear strain in bolts
r1 = 180 mm G of the system bar as shown.
r2 = 200 mm
A = area of cross-section of bolt
Shear stresses in bolts

i d = direct shear stress — 10000


A
N/mm2

'Cs oc r
G of system lies at G', 100 mm for upper bolts
ri = 180 mm, r2 = 200 mm
Ts = kr
Force, fs = krA
Torque, T = EWA
k[2r12 A + r22A] = kA[2 x 1002 + 2002]
kA x 10.40 [ 10.48] = 30,000 x 150
3 x15 x105
Ak —
104 x 10.48
42.939
k—
A
42.939 x 180 7729
To = kri — =
A A 150
10000
TA
,. — A

100
tanO = 0.667
0 = 33.7°
i
s cos 33.7 = 6430
is sin 33.7 = 4288
16430
Td + Ts cos 33.7 —
A

4288 12 11 6 430)2 10A00 270 16980


+ — 18.386 + =
Tr — A A 9
16980
T r' resultant shear stress — Nimm2
A
Bending stress
KA [211 2 + 122] = Re
KA [2 x 4002 + 1002] = P.e = 30000 x 500 = 15 x 106 Nmm
KA [320,000 + 100,00] = 15 x 106
3 2 P. IAS & IFS (Objective & Conventional) Previous Solved Questions MADE EASY

15 x 106 1500
k'A — .
33 x 104 33
45.45
k'A = 45.45, K—
A
45.45 18180
a bi = 0,, — A x 400 =
A

abi 9090
2 A

(Gbi )2 2
For upper bolt (max. principal stress) = + Tr
2

(9090)2 (16980 )2
= 9090 + A ) A )
9090 1000 9090 + 19260
+ V82.628 + 288.32 =
A A A
28350
Pmax — = 120 (since FOS = 5)
A

A= 236.25 mm2 = Ed 2
4
Bolt dia., d = 17.344 mm -=-' 18 mm
For upper bolt (max. shear strain),FOS = 5
19260
'CMaX — = 70
A

A = 275.143 = Ed2
4
Bolt dia., d = 18.72 mm

Objective Questions
Q.1 The complementary shear stresses of intensity (a) 30° and 120° (b) 45° and 135°
T are induced at a point in the material, as shown (c) 60° and 150° (d) 75° and 105°
in the figure 2.14. Which of the following is the [CSE-Prelims, ME : 1998]
correct set of orientation of principal planes with
Q.2 A material element is subjected to a plane state
respect to AB.
of stress such that the maximum shear stress
D T is equal to the maximum tensile stress would
C
compound to
T T
D C

e I
TI (a) 1 0
1
T

A T< B A B
T<---

Flg. 2.14
Strength of Materials Principal Stresses 41 33

(d)
(b)

[CSE-Prelims, ME : 1999]

Q.4 Biaxial stress system is correctly shown in


30

20

a
(c)
(a) 20... ►10
1

20

30
10

30
6
(d) 20

(b) 10 - 10
1
20

[CSE-Prelims, ME : 1998] 30
y
Q.3 For the given stress condition ax = 2 N/mm2, 10
ay = 0, Txy =0, the correct Mohr's circle is 40

(a)
(C) 3 30

40
40
(b)

20

(d) 30 30
1

(c) 20 t

40
[CSE-Prelims, ME : 1999]
34 0. lAS & IFS (Objective & Conventional) Previous Solved Questions MADE EASY

Q.5 Principal stresses at a point in a stressed solid


are 400 MPa and 300 MPa respectively. The
normal stress on plane inclined at ±45° to the
principal planes will be 0
(b)
(a) 200 MPa and 500 MPa
(b) 350 MPa on both planes
(c) 100 MPa and 600 MPa
(d) 150 MPa and 550 MPa
[CSE-Prelims, ME : 2000]

Q.6 A Mohr's stress circle is drawn for a body


subjected to tensile stresses f, and fy in two (c) 0
mutually perpendicular directions such that
fx > fy. Which one of the following statements in
this regard is not correct?
(a) Normal stress on a plane at 45° to fx is equal

fx fy
to
2 B
(d) A C 0
(b) Shear stress an a plane at 45° to f is equal

fx fy
tO
2
(c) Maximum normal stress is fx (0 = origin and C = centre of circle, OA = al ,
OB= 02 )
fx fy
(d) Maximum shear stress is equal to Q.9 When the two principal stresses are equal and
2
like, the resultant stress on any plane is
[CSE-Prelims, ME : 2000]
(a) equal to principal stress
Q.7 The resultant stress on a certain plane makes (b) zero
an angle of 20° with the normal to the plane. On (c) one half of the principal stress
the plane perpendicular to the above plane. The (d) one third of the principal stress
resultant stress makes an angle of 0 with the [CSE-Prelims, ME : 2002]
normal. The value of 0 can be
Q.10 On a plane, resultant stress is inclined at an
(a) 0° to 20°
angle of 45° to the plane. If the normal stress is
(b) any value other than 0° to 90°
100 N/mm2, shear stress on plane will be
(c) any value between 0° to 20°
(a) 71.5 N/mm2 (b) 100 N/mm2
(d) 20° only
(c) 86.6 N/mm2 (d) 120.8 N/mm2
[CSE-Prelims, ME : 2001]
[CSE-Prelims, ME : 2003]
Q.8 The correct Mohr's stress circle drawn for a point
Q.11 The moduli of elasticity and rigidity of a material
in a solid shaft compressed by a shrink fit hub
are 200 GPa and 80 GPa respectively. What is
is as
the value of Poisson's ratio of the material?
(a) 0.30 (b) 0.26
(c) 0.25 (d) 0.24
(a) [CSE-Prelims, ME : 2007]

Q.12 State of stress at a point of a loaded component


is ax = 30 MPa, ay = 18 MPa, 8 MPa. If the
Txy =
Strength of Materials Principal Stresses 4 35

larger principal stress at the point is 34 MPa, Select the correct answer using the codes given
what is the value of smaller principal stress? below:
(a) 12 MPa (b) 14 MPa (a) 1 and 3 (b) 2 and 3
(c) 16 MPa (d) 18 MPa (c) 1 and 4 (d) 2 and 4
[CSE-Prelims, ME : 2008] [CSE-Prelims, CE : 2004]

Q.13 The maximum shear stress occurs on Q.17 If Ex and Ey are the maximum and minimum
(a) principal planes strains, respectively, in the neighbourhood of a
(b) plane at 45° to the principal planes point in a stressed material, then what is the
(c) planes at 90° to the principal planes expression for the maximum principal stress?
(d) planes independent of the inclination to the (a) EE, (b) E(E, + µEy)
principal planes E(E, + µEy ) E(Ey +µE,)
[CSE-Prelims, CE : 2003] (c) (d)
1— µ 2 1-112
Q.14 The ratio of the maximum shear stress to the [CSE-Prelims, CE : 2007]
difference of the two principal stresses is
Q.18 The principal stresses in N/mm2 on a square
(a) 1/2 (b) 1/3
element are shown in the figure below. What is
(c) 1/4 (d) 1/6
the intensity of tangential stress (pi) on the
[CSE-Prelims, CE : 2002]
plane BD?
100
Q.15 For the two dimensional stresses shown in the
figure, what is the normal stress on the 45° oll llll
plane?

100 100

ctfffttB
2 100
10 N/mm
Fig. 2.20

(a) 50 N/mm2 (b) 0


(c) 100 N/mm2 (d) 200 N/mm2
[CSE-Prelims, CE : 2006]
Fig. 2.19 Q.19 A point in an element is stressed as shown in
the figure given below.
(a) 20 N/mm2 (b) 12 N/mm2
(c) 4 N/mm2 (d) 8 N/mm2 100 N/mm2
[CSE-Prelims, CE : 2004] t E t A

Q.16 A rectangular block is subjected to shear stress


intensity of T. The stresses induced on a plane 100 N/mm2 c„, —.-100 N/mm2
inclined at 135° to the horizontal axis shall be O.,

1. ax = 4-T (tensile)
2. crx = —T (compressive) 1 1

3. atangential = -FT 100 N/mm2

4. atangential = Fig. 2.21


36 ► IAS & IFS (Objective & Conventional) Previous Solved Questions MADE EASY

What is the value of normal stress on the oblique


400 N/mm2
plane BE?
(a) 200 N/mm2 (b) 175 N/mm2
(c) 150 N/mm2 (d) 100 N/mm2
[CSE-Prelims, CE : 2007] 400 N/mm2

Q.20 A specimen is subjected to pure shear, the shear


stress being q. Tensile and compressive
stresses of intensity a, occur on planes inclined 400 N/mm2
at 45° to the shear stress. What is the value of
Fig. 2.22
the ratio a/q ?
(a) 200 (b) 400,
(a) 2 (b) 1.5
(c) 1.25 (d) 1 (c) 400/V- (d) 0
[CSE-Prelims, CE : 2008] [CSE-Prelims, CE : 2009]

Q.21 The square element in the figure is subjected Answers


to a biaxial stress of 400 N/mm2 as shown. What 1. (b) 2. (d) 3. (d) 4. (d) 5. (b)
is the intensity of normal stress pn on the plane 6. (d) 7. (b) 8. (d) 9. (a) 10. (b)
BD? 11. (c) 12. (b) 13. (b) 14. (a) 15. (d)
16. (d) 17. (c) 18. (b) 19. (d) 20. (d)
21. (d)

Explanations

1. (b) 3. (d)
45° and 135°
p1 = +r, P2 =

Fig. 2.23
Cannot Mohr's stress circle.

5. (d)
40
20

Fig. 2.22 30 30

20
2. (d)

al -(-G1)
max
2 40

= a = maximum tensile stress Fig. 2.24


Strength of Materials Principal Stresses 4 37

Complementary shear stresses are balanced 10. (b)


Normal stresses is balanced in x and y-direction

5. (b) 100 mpa

D1 + 10
, ,2 ± P1 — P-2 cos 90°
an — 45°
2 2

400 + 300
= 350 MPa
2
On both planes.

6. (d)

fx fy
Amax
2 Fig. 2.26
= 100 MPa
7. (b)
11. (c)
E = 2G(1 +v)
200 = 2 x 80 (1 + v)
ar cos 20° 1.25 = 1 + v
v = 0.25

12. (b)
ax + ay = p1 + P2
30 + 18 = 34 + p2
p2 = 14 MPa

13. (b)
The maximum shear stress occurs on planes at
Fig. 2.25 45° to the principal planes.

4) can be other than 0 or 90° 14. (a)


where a = 0 P1 — P2
— max
2
8. (d)
P2
For a point in a solid shaft, compressed by a shrink =2
ti max
fit hub is
= ao = p' (junction pressure) ti max
= 0.5
OA = OB = OC = p', — P2
circle will be point 15. (d)
9. (a)
_ Pi P2 sin 20
2

ti = +45°, sin29 = 1
— P2
a 2—
0 2
Pi= P2 = P
= 10 N/mm2, p2= 6 N/mm2

a on any plane = p. P1 + P2 P1 P2
+ cos 20
Pn — 2 2
38 ► lAS & IFS (Objective & Conventional) Previous Solved Questions MADE EASY

cos 20 = cos 90° = 0 18. (b)


= p2 = —100 N/mm2
10+6 2
p_ = 8 N/mm
n 2 pt _ Pi — P2 cos 90° = 0
2
16. (d) Mohr's stress circle is a point.

19. (d)
P1 = P2 = +100 N/mm2
+ P2
— 100
2
45°
. 913° — P2 "
—u
2
+ P2 P1 P2 cos 60°
a _
a= sin (-2 x 45°) e 2 2
= —t (compressive) = 100
0 = 450 20. (d)
atangential = 't COS 90° = 0
—=1
Shear

—t

Fig. 2.27 Fig. 2.29

17. (c) 21. (d)

e _ Pi — 1-IP2 p1 = +400 N/mm2


x E p2 = —400 N/mm2

P2 — 11,131 Pi + P2
v.
2
— o
E

Eex = — 1-1P2 •• .(i) P1 — P2


— 400 N/mm2
2
ay = P2 — PP1 0 _ 450

EVEy = µP2—µp1
p1 + P2
+Pi — P2
(1 —1,1,2) P1 = E(ex + 1.tev) p —
n
cos90°
2 2
= 0+400 x 0=0
P1 E
— (1— µ2 ) [ex +"Y]
03
CHAPTER
Thin and Thick Shells

0.3.1 Derive a formula for increase in volume of a thin metallic sphere when it is subjected to an
internal pressure p. A thin spherical shell of copper has a diameter of 400 mm and a wall thickness
of 2 mm and just full of water at atmospheric pressure. Calculate the volume of water pumped in
to raise the inside pressure to 1.5 N/mm2. The modulus of elasticity of copper is 1 x 105 N/mm2.
K (Bulk modulus) is 2.5 x 103 N/mm2 and Poisson's ratio v is 0.25)
[CSE-Mains, 2000, ME : 20 Marks]

Solution:
(a) When a thin spherical shell is subjected to internal pressure p, circumferential stress, sac is developed in
wall of the shell as shown in figure 3.1.

pD
a=
e 4t
where p = pressure, D = diameter, t = wall thickness, Ec = circumferential strain
a, va, pD
_ 4tE(1- v)
ec - E E

3pD
Volumetric strain, Ev = 3E, = (1 v)
4tE
(Neglecting the effect of p)

703
V, volume of shell -
6

3pD 703
8V, change in volume = evV = — (1 v) x
4tE 6
(b) D = 400 mm
t = 2 mm
p = 1.5 N/mm2

pD 1.5 x 400
a_ = = 75 N/mm 2
4t 4x2

pD 3 x 75
(1 0.25) Fig. 3.1
EV = 3 -4 (1-v) = E
40 0. IAS & IFS (Objective & Conventional) Previous Solved Questions MADE EASY

225 x 0.75
= 1.6875 x 10-3
1x105

it133 TE
V = Volume of shell - X 40 03 = 33.51x 106 mm3
6 6
Expansion in volume of shell = ev x V= 1.6875 x 10-3 x 33.51 x 106
= 56.548 x 103 mm3

Decreases in volume of water = xV = 1.5 , (33.51 x 106 )


k 2.5 x 10'
= 20.106 x 103 mm3
Volume of water pumped in = 56.548 x 103 + 20.106 x 103 mm3
= 76.654 x 103 mm3
= 76.654 cc

Q.3.2 A thin cylinder 150 mm internal diameter and 2.5 mm thick has its ends closed by rigid plates and
is then filled with water under pressure. When an axial pull of 37 kN is applied to the ends, water
pressure is observed to fall by 0.1 N/mm2. Determine the value of Poisson's ratio. Assume
E. 140000 N/mm2, K for water = 2200 N/mm2.
[CSE-Mains, 2008, ME : 20 Marks]

Solution:
Op = 0.1 N/mm2
x
Reduction in volumetric strain of cylinder =
4tE (5 4v)

Op 0.1
Reduction in volumetric strain of water =
K 2200
Axial force = 37000 N

37000
Axial stress - 37000 = 31.406 N/mm2
701- x 150 x 2.5

31.406
Increase in volumetric strain due to a = T
a (1- 2v) = E x (1- 2v)

0.1x 0.1 31.406


or 150(5-4v)+ (1 2v)
4 x 2.5E 2200 = E

0.1x 140000
or 1.5(5 - 4v) + 31.406 (1 - 2v)
2200
7.5 - 6v + 6.3636 = 31.406 - 62.812v
or 56.812v = 31.406 - 6.3636 - 7.5 = 17.5424
17.5424
Poisson's ratio, v - = 0.308
56.812

Q.3.3 A tyre is shrunk on a wheel of 12 meter diameter. Assuming the wheel to be rigid, calculate the
internal diameter of the tyre if after shrinking, hoop stress in the tyre is 1200 kgf/cm2, a for the tyre
= 11.7 x 10-6/°C and E = 2 x 106 kgf/cm2. Find the least temperature to which the tyre must be
heated above that of the wheel before it could be fitted.
[CSE-Mains, 1986, ME : 30 Marks]
Strength of Materials Thin and Thick Shells 41

Solution:
Hoop stress, csc = 1200 kgf/cm2, a = 11.7 x 10-6 /°C
E = 2 x 106 kgf/cm2, D= 12m

1200 _ a,
Hoop strain, E = = 600 x 10-6 = 0.6 x 10-3
2 x 106 E
= A Ta = AT x 11.7x 10-6 = 0.6 x 10-3

0.6 x 1000
AT - = 51.28°C
11.7

Q.3.4 Distinguish between thin and thick cylinders. A thin cylindrical shell having hemispherical ends is
subjected to internal pressure p. The internal diameter is 'd and thickness of cylinder and
hemisphere are t1 and t2 respectively. Assuming Poisson's ratio, v = 0.3, prove that.
t2 7
1. For no distortion at junction, -6- = 17

2. For equal maximum hoop stress in cylinder and hemisphere, ratio it = 0.5 .
ti
[CSE-Mains, 2002, ME : 20 marks]

Solution:

D
In a thick shell or a thin shell, D is internal diameter and t is wall thickness, if — > 20 , then it is a thin

shell. When the shell is subjected to internal pressure, the hoop stress developed in the shell does not

vary much across the thickness. If — < 20 , i.e., thickness of shell is considerable in comparison to
f
diameter, then there is variation of hoop and radial stresses
across the thickness of thick shell.
Figure 3.2 shows a thin cylindrical shell with hemispherical ends,
subjected to internal pressure p.
In cylindrical portion:
pD
a hoop stress = 2t1
Fig. 3.2
pD
Axial stress, as = 4t
1
pD v pD (2 v)
c hoop strain =
1 E ptDE
In hemispherical portion at junction
pD
a = hoop stress = 4t
2

c / hoop
E, strain = PD (1 v)
4t 2E
For no distortion
Sc' = EC"
4 2 0. lAS & IFS (Objective & Conventional) Previous Solved Questions MADE EASY,

v) = (1— v)
4t0 (2 4t2 E
t2 1—V 1— 0.3 0.7 = 7
or
t1 2—v 2 — .07 1.7 17

pD
Maximum stress in cylindrical portion =
2t1

pD a
Maximum stress in hemispherical portion = =
4t2
a
= ac

pD = pD
2t1 4t2
t2 =
0.5
tl

Q.3.5 A steel sleeve is pressed onto a solid steel shaft which has 5 cm diameter. The radial pressure
between shaft and sleeve is 1800 N/cm2 and hoop stress at the inner surface of sleeve is
4500 N/cm2. If an axial compressive load of 50 kN is applied to the shaft, determine change in
radial pressure at the interface of shaft and sleeve. Assume, v = 0.3.
[IFS 2011, ME : 15 Marks]

Solution:
Shaft diameter, d = 50 mm
Radial stress, a = 18 N/mm2 = p' at inner radius
Hoop stress, a = 45 N/mm2
Poisson's ratio, v = 0.3
Compressive load, Pc = 50,000 N
50000 4
Axial compressive stress, CYa = x , 25.46 N/mm2
lc 50'
Say = additional radial stress developed in shaft due to axial
compressive stress of 25.46 N/mm2
Additional stresses in shaft
p,, = radial compressive stress
ID" = hoop stress compressive
aa" = axial compressive stress
cs additional circumferential strain in shaft
—p" vp" va," —p" + 0.3p" + 0.3 x 25.46
+ +
E E
—0.7p" + 7.638

Sleeve
Additional radial stress = p" at inner surface
45
additional hoop stress = — p" = 2.5pItensile)
18
Strength of Materials Thin and Thick Shells 4 43

2.5p" + 0.3p" 2.8p"


esi, strain (hoop direction) -
E E E
Strain compatibility
Es/ = Ecs
7.638 - 0.7p" 2.8p"
E - E
7.638
p" = = 2.18 N/mm2
3.5
= Increase in radial pressure

Q.3.6 (a) How will you distinguish between a thin walled and a thick walled pressure vessel.
(b) What advantages you obtain by wire winding a thin cylinder?
(c) What largest internal pressure can be applied to a cylindrical tank 1.8 meter in diameter
and 14 mm wall thickness, if the ultimate tensile strength of steel is 467 MPa and factor of
safety of 7 is desired?

Solution:
(a) In a pressure vessel if D is diameter and t is wall thickness and internal pressure is p, then if D/t
ratio is greater than 20 then it is termed as thin walled pressure vessel. Hoop stress and axial
stresses developed in thin pressure vessel are assumed to be constant along radial thickness
of pressure vessel. If Dlt ratio is much less then 20, then it is termed a thick walled pressure
vessel. Hoop stress and radial stress developed in thick pressure vessel vary along its thickness.
(b) When a thin pressure vessel is subjected to internal pressure, then axial and hoop stresses
developed in thin shell are

pD
as = - and ac=
2t
ac = aya
Material of the shell is not gainfully used in axial direction. By wire winding, a thin wire under
tensile stress crw is wound tightly along the outer circumference of shell introducing initial
compressive stress ac' in the hoop direction in shell.
When the thin cylinder with wire wound on it is subjected to internal pressure then hoop stress
developed in shell due to internal pressure is ac" resultant hoop stress will be ad, = 0:- ac'.
Pressure bearing capacity of the cylinder is increased by wire winding.
(c) aut = 467 MPa
FOS = 7

467
Cr a I = allowable stress = = 66.71 N/mm 2
7

pD
= , hoop stress
2t
D= 1800 mm, t = 14 mm
2taa 2x14 x66.71
P- .
D 1800
= 1.038 MPa, maximum internal pressure
44 1. IAS & IFS (Objective & Conventional) Previous Solved Questions MADE EASY

Q.3.7 A compound cylinder is formed by shrinking a tube 16 cm external diameter and 12 cm internal
diameter on to another which has an internal diameter of 8 cm. If after shrinking the radial
compressive stress at common surface is 300 kg/cm2, find the circumferential stress at the inner
and outer surfaces and at the common surface.

[CSE-Mains, 1987, ME : 30 Marks]


Solution:
Inner cylinder
p' = 300 kg/cm2 Ri = 4 cm (junction pressure)
R2 = 8 cm R3 = 6 cm (junction)

2pRE
acRi
-Rf

6
= —2 x 300 1080 kgf/cm2
x36 -16=
2-
8 cm
RE +R-f Compass cylinder

acR3 = 3 — R21
P x R2 Fig. 3.3
36 +16
. -300 x = 780 kg/cm2
36 -16 + 1071.43 kgf/cm2
Outer cylinder
+ 771.43 kgf/cm2

RE +RE
a cR3 +13' X 2 2
R2 - R3

64 + 36
. +300 x = +1071.43 kgf/cm2
64 - 36

2R —1080 kgf/cm2
6 cR2 = x De
,2 Dp2
112 - 13 Hoop Stress distribution

Fig. 3.4
= +300 x 2 x 36 = 771.43 kgf/cm2
64 - 36

Q.3.8 On the outer surface of a closed thick cylinder of diameter ratio 2.5, were fixed strain gauges to
measure the longitudinal and circumferential strains. At an internal pressure of 230 MN/m2 the
strains were recorded as 91.8 x 10-6, and 369 x 10-6 respectively. Determine the value of Young's
modulus and, modulus of rigidity and Poisson's ratio.
[CSE-Mains, 1993, ME : 20 Marks]
Solution:
R2
= 2.5
Ri
p = 230 N/mm2
Outer surface
2/312 2 x 6.25
Hoop stress, ac P X 2 - 2 = 230 x 6.25 -1 = +547.62 N/mm2
Strength of Materials Thin and Thick Shells 4 45

Axial stress, aa = +PX 2 2 = 273.81 N/mm2


R2 —
= 0, radial stress is zero at outer surface

547.62 v x 273.81 273.81


E strain = = (2 v)
E E

273.81 v x 547.62 273.81


c axial strain — (1 2v)
E E E
273.81
(2 v) = 369 x 10-6
E
273.81
(1 2v) = 91.8 x 10-6
E
2—v 369
or = 4.0196
1— 2v 91.8
4.0196-4.0196 x 2v = 2 —v
2.0196 = 7.0392v
v = 0.287
273.81(2 — 0.287) 6
E— x 10 = 1.271 x 106 N/mm2
369

E 1.271 X 1 06
Modulus of rigidity, 4.94 x 105 N/mm2
G= 2(1+v) 2 x1.287 =

Q.3.9 Find the ratio of thickness of internal diameter of a thick tube subjected to internal pressure when
the pressure is 5/8 of the value of maximum permissible circumferential stress.
Find the increase in internal diameter of such a tube of 100 mm internal diameter, when the
internal pressure is 100 MN/m2, E. 200 x 109 N/m2, Poisson's ratio = 0.286.
[CSE-Mains, 1992, ME : 20 Marks]
Solution:
Say, R1 = Internal radius, R2 = External radius, p= Internal pressure

R 22 +
R2
1
ac max = P — 2
132 —1312

5
But, P = 8 6 cmax

5 R 2 +R12
—XpX 22 — p
R2 —131 2

or 5F2 + 513 = 8/3 — 8R1

3F2 = 13131
R2 = 2.08 R1
R1 + t = 2.08 R1
Di
t = 1.08 R1 = = 0.54D1
46 0. IAS & IFS (Objective & Conventional) Previous Solved Questions MADE EASY

= 0.54

Now, = 50 mm
p = 100 N/mm2
R2 = 104 mm

1042 + 502 _ 50 x 10816+2500 13316


ac max = 50 x = 50x
1042 - 50' 10816 - 2500 8316
= 1.6 x 50 = 80 N/mm2
R2 502
Axial stress, as = p x 2 - 50 x
R2
"2 "1 1042 - 502

502
= 50 x = 15.03 N/mm2
8.316
Principal stresses at inner radius
ac max = +80, a,. = 15.03 N/mm2

p = -50 N/mm2
80 0.286 x15.03 0.286 x 50
Strain, Ec E

80 +10 90
= 4.5 x10-4
E 200,000
Change in internal diameter
SDI = 4.5 x 10-4 x 100 = 0.045 mm

Q.3.10 A compound cylinder is made by shrinking a jacket with outer diameter of 20 cm on a hollow
cylinder with diameters 10 and 15 cm. When the compound cylinder is subjected to an internal
pressure of 350 kgf/cm2, the maximum circumferential stress in both the cylinder is the same.
Calculate the maximum stress developed and internal diameter of the jacket. Take the value of
E = 2 x 106 kg/cm2.
[CSE-Mains, 1988, ME : 30 Marks]

Solution:
Inner radius, R1 = 5 cm
Junction radius, R3 = 7.5 cm
Outer radius, R2 = 10 CM
Internal pressure, p = 350 kgf/cm2

+
Inner cylinder, acRi = 350 x R2
1=6 -13-F

100+25
= 350x
100 - 25 Fig. 3.5
= 583.33 kgf/cm2
FifF? +RfFIE .= 52 X 102 ± 52 X 7.52
Outer cylinder, acR3 = Px 02 (R 2 R2)
350 x
/13 2 - 1 / 7.5 2 (10 2 -5 2 )
Strength of Materials Thin and Thick Shells 4 47

2500 +1406.25 3906.25


= 350 x = 350 x
56.25(100 - 25) 75 x 56.25
= 324.074 kgf/cm2
Junction pressure p',
2R2 , 2 x7.5 2
aCRi (inner cylinder) = -p' x 3 = p x
RE -Fq 56.25 - 25

acR, = -p' x 3.6

+Fq , 102 + 7.52


-+P x 2
acR3 = +13' X R22-R32 10 - 7.5 2
156.25
6CR3 = p' X = +p' x 3.5714
43.75
Finally
583.33 - 3.6p' = 324.074 + 3.5714p'
_ 259.256
= 36.15 kgf/cm2
7.1714
amax = 583.33 - 3.6 x 36.15
= 583.33 - 130.145 = 453.185 kgf/cm2
Shrinkage allowance
D,
8D3 -=[3.6p'+ 3.5714p1

= 7.1714x 36.15
E
7.5x
2 x 10'
= 1.944 x 10-3 cm
D3, inner diameter of jacket = 15 -1.944 x 10-3 = 14.998 cm

0.3.11 A compound cylinder is made by shrinking an outer tube of outside diameter 200 mm and inside
diameter 150 mm onto an inner tube, internal diameter 100 mm with a radial interference of
0.2 mm.
Both the tubes are made of steel with elastic modulus E = 2 x 106 kg/cm2 and Poisson's ratio
v = 0.3.
Calculate the value of pressure at the interface and values of hoop stress in the two tubes at the
interface.
Work from the first principle assuming the basic Lame's equation.

ar = A- ae A+ 2
r2 r
[CSE-Mains, 1999, ME 30 Marks]

Solution:
= 5 cm, R2 = 10 CM, R3 = 7.5 cm, 8R3 = 0.02 cm

p'R3 [R5 + R3 R3 +Rf]


8R3 - +
E R2 -RE 14 -13?
48 ► IAS & IFS (Objective & Conventional) Previous Solved Questions MADE EASY

102 +7.52 7.52 +52


0.02 - P' x 7.5 +
2 x 106 102 - 7.52 7.52
52

5333.3 _ ,[156.25 81.25]


P +
43.75 31.25

p'[3.571+ 2.6]= 6.171 p'


p' = 864.2 kg/cm2
= 8.64 kg/mm2, Pressure at interface
Hoop stress at interface Fig. 3.6

R3 +Ri2 7 52 4. 52
Inner tube a' CR3 = = 864.2 x ' 2 ' 2246.9 kg/cm2
7.5 - 5 2 =

, 3 10 2 +7.52
Outer tube 864 2 x „
'IcR3 +P X Fq+R 10` -7.5'
= 864.2 x 3.571 = 3086.06 kg/cm2

Objective Questions

Q.1 Match List-I (Terms used in thin cylinder stress Q.2 A thin cylinder of diameter d, thickness t is
analysis) with List-II (Mathematical expression) subjected to an internal pressure' p'. Change in
and select the correct answer. diameter is (where E is the modulus of elasticity
List-I andµ is the Poisson's ratio.)
A. Hoop stress, a
pd2 pd2
B. Maximum in plane shear stress (a) (2 µ) (b) (1+µ)
4tE 2tE
C. Longitudinal stress
D. Cylinder thickness
pd 2 pd2
List-II (d) (2+ it)
(c) 7E-(2+11) 4tE
pd
1. [CSE-Prelims, ME :1998]
4t
pd Q.3 The percentage change in volume of a thin
2. cylinder under internal pressure having hoop
2t
stress = 200 MPa, E. 200 GPa and Poisson's
pd
3. ratio = 0.25
2o
(a) 0.40 (b) 0.30
pd (c) 0.25 (d) 0.20
4.
8t [CSE-Prelims, ME : 2002]
Codes:
Q.4 A thin cylinder shell of mean diameter 750 mm
A BCD
and wall thickness 10 mm has its ends rigidly
(a) 2 3 1 4
closed by flat steel plates. The shell is subjected
(b) 2 4 3 1
to internal fluid pressure of 10 N/mm2 and an
(c) 2 3 4 1
external pressure p1 . If the longitudinal stress
(d) 2 4 1 3
in the shell is to be zero, what should be the
[CSE-Prelims, ME : 1998]
approximate value of p1?
Strength of Materials Thin and Thick Shells 4 49

(a) 8 N/mm2 (b) 9 N/mm2 2. Circumferential stress is twice the


(c) 10 N/mm2 (d) 12 N/rnm2 magnitude of longitudinal stress.
[CSE-Prelims, ME : 2007] 3. Hoop stress and longitudinal stress remain
constant along the vessel thickness.
Q.5 A thin walled water pipe carries water under a
Which of these statements are correct?
pressure of 2 N/mm2 and discharges water into
(a) 1 and 2 only (b) 2 and 3 only
a tank. Diameter of the pipe is 25 mm and
(c) 1, 2 and 3 (d) 2, 3 and 4
thickness is 2.5 mm. What is the longitudinal
[CSE-Prelims, ME : 2009]
stress induced in the pipe?
(a) 0 (b) 2 N/mm2 Q.10 In a hollow thick cylinder the radial stress or
(c) 5 N/mm2 (d) 10 N/mm2 under an internal pressure 'p'
[CSE-Prelims, ME : 2007] (a) increase from a minimum at the innermost
surface to a maximum value at the
Q.6 What is the value of volumetric strain in a thin
outermost surface
walled cylindrical vessel with modulus of
(b) decreases from a maximum at the
elasticity E, Poisson's ratio µ and of diameter
innermost surface to a minimum value at
'd, wall thickness 't' and subjected to internal
the outermost surface
fluid pressure p?
(c) increase from zero at the innermost surface
pd pd to a value or = p at the outermost surface
(a) (5 - 4µ) (b) - 3µ)
4tE 3tE (d) decreases from a value ar. = p at the
pd pd innermost surface to zero at the outermost
(c) (3 20 (d) — (2 - )
2tE tE surface
[CSE-Prelims, ME : 2006] [CSE-Prelims, ME : 2001]

Q.7 A thin cylindrical shell 500 mm in diameter and Q.11 A seamless pipe with 80 cm diameter carries a
25 mm thick having E. 2.0 x 105 N/mm2 and fluid under a pressure of 2 N/mm2. If the
= 0.25, is subjected to internal fluid pressure permissible tensile stress is 100 N/mm2, the
of 20 N/mm2. What is unit change in volume of minimum required thickness of the pipe is
the shell? (a) 2 mm (b) 4 mm
(a) 4 x 10-3 (b) 2.5 x 10-3 (c) 8 mm (d) 10 mm
(c) 2 x 10-3 (d) 1.5 x 10-3 [CSE-Prelims, ME : 2003]
[CSE-Prelims, ME : 2008]
Q.12 Note: 'id is the internal fluid pressure
Q.8 Which one of the following is the correct A thin cylindrical shell is subjected to the loads
expression for hoop stress, if p is internal as shown in figure. The element marked 'A' will
pressure in a thin walled cylinder of diameter be subjected to
'd and thickness t?
pd pd
(a) (b)
2t
pd
(c) pd
— (d)
4t 8t
[CSE-Prelims, ME : 2009]

Q.9 Consider the following statements in respect of


thin cylindrical pressure vessel subjected to Fig. 3.7
internal fluid pressure.
1. State of stress along the vessel thickness (a) biaxial compressive stresses
is biaxial except on the inside surface. (b) biaxial tensile stresses
50 0. IAS & IFS (Objective & Conventional) Previous Solved Questions MADE EASY

(c) uniaxial compressive stresses pD


(d) tensile stress along the longitudinal (b) tE-(1- ")
direction only
pD2
[CSE-Prelims, ME : 2004] (c) (d) (1 21.0
4tE
Q.13 The maximum hoop stress in a thick cylinder [CSE-Prelims, ME : 2010]
under internal pressure would occur at which of
Q.16 A thin cylindrical shell is subjected to internal
the following locations?
pressure, such that hoop strain is approximately
(a) At the inner surface
five times the axial strain, what is the material
(b) At the outer surface
of the shell (depending on Poisson's ratio)
(c) At mid thickness
(a) Steel (b) Cast iron
(d) Between the inner surface and mid
(c) Aluminium (d) Wrought iron
thickness
[CSE-Prelims, ME : 2008] Q.17 A thin cylindrical shell made of steel is subjected
to internal pressure such that hoop stress
Q.14 A thin cylindrical pressure pipe with both ends
developed in shell is 120 MPa. This shell is
closed has diameter 1000 mm. The pipe is
subjected to axial compressive stress such that
subjected to an internal pressure of 4 N/mm2.
there is no change in the length of the shell.
The permissible tensile stress in the material is
(There are end plates on the shell). If E= 200
100 N/mm2. What is the minimum required
GPa, Poisson's ratio is 0.3. What is axial
thickness of the pipe?
compressive stress applied on cylinder
(a) 5 mm (b) 10 mm
(a) 90 MPa (b) 60 MPa
(c) 40 mm (d) 20 mm
(c) 36 MPa (d) 24 MPa
[CSE-Prelims, ME : 2008]
Answers
Q.15 What is the change in diameter D of a thin
spherical shell of wall thickness t when subjected 2. (a) 3. (d) 4. (c) 5. (c)
to an internal fluid pressure p? (E = Young's 7. (c) 8. (b) 9. (c) 10. (d)
modulus of la = Poisson's ratio) 12. (b) 13. (a) 14. (d) 15. (c)
17. (d)

Explanations
1. (d)
d
od = xd = (2 v)
pD c 4E
al, Hoop stress -- 2
2t
3. (d)
Maximum shear stress, 13) - 4 c = 200
8t
as = 100
Longitudinal Stress, ±
15 1 - 1 200 - 0.25 x 100 175
4t

Cylinder thickness t , PD -3 100 - 0.25 x 200 50
2a Ea =
2. (a) 400 400
2ec + ea =
pd - vpd _ pd E 200 x 1000
E= - (2-v)
2tE 4tE 4tE e=2x 0.2%
Strength of Materials Thin and Thick Shells 4 51

4. (c) 13. (a)


10 x 750 pi x750 Maximum hoop stress in a thick cylinder under
-0
4 x 10 4 x 10 internal pressure would occur at the inner surface.
p1 = 10
14. (d)
5. (c) p = 4 N/mm2
pD 2 x 25 D = 1000 mm
a_ = = 5 N/mm2
a 4t 2.5 x 4
D
= 100 =
6. (a) 2t
pd - 4 x 1000
Volumetric strain = — (5-4g) t = .20 mm
4tE 2 x 100
7. (c)
15. (c)
E - PD (5 - 4v) , as v = 0.025 Thin spherical shell
v 4tE t = wall thickness
pD x 4 pD = Poisson's ratio
4tE tE p = internal pressure
20 x 500 D = diameter
25 x 2 x105 pD
= 200 x = 2 x 10-3 6 11)
G 4tE (
8. (b) ED=Ec xD
pd pD 2 (1-v)
Hoop stress =
4tE
9. (c) 16. (c)
1, 2, 3 are correct statements. = Poisson's ratio
10. (d) hoop strain 2- 5
ar decreases from maximum value of ar = pat the
axial strain - 1- 2g
inner most surface to zero at the outermost surface.
or 2-g= 5- 10 p.
11. (c) 9g=3
D = 800 m, p = 2 N/mm2
= 100 N/mm2 µ= 1 = 0.333
pD
= is for aluminium i.e., 0.33
2t
pD 2 x 800 17. (d)
= = 8 mm
t = 2a, 2 x 100 a = 120 MPa
as = 60 MPa
12. (b)
as Via c as'
E E
60 - 0.3 x 120 = cr;
as
as = 24 MPa
Axial compressive stress.
1' a,
Fig. 3.8
04
CHAPTER
Shear Force and Bending
Moment Diagrams

Q.1 A beam ABCD, 10 m long is supported at B, 1 m from end A and at C, x is from end D. The beam
carries a point load of 10 kN at end A and a UDL of 4 kN/m throughout its length fig. 4.1. Determine
the value of x if centre of the beam is the point of contraflexure. Draw the BM diagram.
10 kN

4 kN/m
Al
B
R8 Rc
-'---1m (9—x) x -°1
Fig. 4.1

[CSE-Mains, 2004, ME : 30 Marks]

Solution:
Total load on beam = 10 + 40 = 50 kN
Reaction
Take moments about B
(9 - x)Rc + 10 x 1 = 40 x 4
150
Rc -
9 -x
150 450 - 50x -150 300 - 50x
RB = 50
9-x 9-x 9-x
Moments about centre of the beam
-10 x 5 + R8 x 4 - 4 x 5 x 2.5 = 0

4 (300 - 50x)
-50+ 50 = 0
9 -x

4 (300 - 50x)
100
9 -x --
300 - 50x = 225 - 25x
75 = 25x
x = 3m
300 - 50x 150
RB - = 25 kN
9 -x 6
Strength of Materials Shear Force and Bending Moment Diagrams 11 53

BMD
MA = 0
MB = -10 x 1 - 4 x 0.5 = -12 kNm

M3 =-10 x 3 + 25 x 2-4 x 3 x 1.5


= -30 + 50 - 18 = +2 kNm + 2kN/m

M5 =-10 x 5 +25 x 4-5 x 4 x 2.5


= -50 + 100 - 50 = 0
—12 kN/m
Mc = M7 = -3 x 4 x 1.5 = -18 kNm — 18 kN/m
MD= 0 BMD
Fig. 4.2
Q.4.2 On a simply supported beam (10 m span), a concentrated load (10 kN) and a moment of 40 kNm
act a section 7 m from one of the ends. Draw the shear force and bending moment diagrams.
Indicate the points of contraflexure, if any 10 kN
[ICSE-Mains, 2012, ME : 10 Marks]
Solution:
Fig. shows a beam AB, 10 m long, simply supported at
ends carrying a load of 10 kN at Cand moment 40 kNm at RA = —1 kN
RB = +11 kN
C, 7 m from A
Reaction 3 —0-
Taking moment about A
70 + 40 = 10 RB
Reactions, RB = 11 kN
(b) — 11 kN
Reaction, Ftc = 10 - 11 = -1 kN
SF diagrams
FAC 1 kN
FCB = 1 - 10 = -11 kN
Fig. (b) shows SFD
BM diagram A
MA
Mc = -1 x 7 = -7 kNm
Mc = -7 + 40 = +33 kNm
— 7 kNm
MB = 0, Fig. (C) shows BMD BMD
Fig. 4.3
Q.4.3 A beam ABCD is loaded as shown in figure 4.4. The beam is of rectangular section 50 mm x
100 mm.

600 N
60 N/m

RA = 393.33 N t 5000 Nmm Rc = 386.67 N


1-*-3 m 3m 5m
Fig. 4.4

(i) Sketch the SF and BM diagrams of the beam


(ii) Determine the maximum bending stress at section B of the beam
[CSE-Mains, 2011, ME : 25 Marks]
54 1. lAS & IFS (Objective & Conventional) Previous Solved Questions MADE EASY

Solution:
Reaction
Moment about A

60 x 6
x 4 + 600 x 11 = 5000 + 6/3c (Note that CG of triangular load lies at 4 m from A)
2
720 + 6600 = 5000 + 6/3c
2320 = 6/qc
= 386.67 N
Total load on beam = 600 + 180 = 780 N
Reaction, RA = 780 - 388.67 = 393.33 N
SFD
FA = +393.33 N
386.67 N
30 x 3 A
F3 = 393.33 = 348.33 N 600N
2 393.33
213.33 N
(Just left to C) F6 = 393.33-180=213.33N
(Just right to C) F6 = 213.33 + 386.67 = 600.00 N •
A D
BMD Fig. 4.5
MA = 0
30 x 3
(Just left to B) MB = 393.33 x 3 x 1= 1180 - 45 = 1135 Nm 1135 Nm

(Just right to B)M3'= 1135 - 5000 = -3865 Nm


x
M6 = 393.33x6-5000 6026 x2

= 2360 - 5000 - 360 = -3000 Nm


MD =
Jump moment at B, 5000 Nm
bxd2 50x1002
z- = 8,33 x104 mm3
6 6 — 3000 Nm
M 5000 x 10 3 — 3865 Nm
a - = = 60 N/mm2
max z 8.33 x 104 Fig. 4.6
Q.4.4 Draw the bending moment and shear force diagram of the beam as shown in fig. 4.7(a).
0= Constant.
[CSE-Mains, 2012, CE : 15 Marks]
Solution:
BM diagram (Let us consider this as a combination of to cantilevers AEB and CFD and a simply supported
beam BC)
Portion BC
Carries UDL = 4 kN/m
Total load = 4 x 4 = 16 kN
Reaction at end = 8 kN
x 42
Maximum BM = w/2 = 4 8 = 8 kNm
8
Strength of Materials Shear Force and Bending Moment Diagrams 4 55

2 kN 2 kN

4 kN/m
A 1 Mt-VW\ 1
E B C F D
--1-1 1 I 2 .(-- 4 m—•-1 2 I 1 [4--
(a)
8kN

(b)
BMD
—26 —26 kNm
10 kN

— 10 kN

Fig. 4.7
Cantilever AB
MB =
ME = -16 kNm
MA = -8 x 3 - 2 = -26 kNm
BM diagram for CD cantilever will be similar
SF diagram,
BC, FB = +8 kN, Fcenter = 0

FBE = -8 kN
Fa4 = -10 kN
SFD for portion CFD portion will be similar to that of portion BEA
Figure shows BMD and SFD for the beam AB, BC, CD.

Q.4.5 The bending moment diagram of a simply supported beam AB is shown in figure 4.8(a). Sketch
the loading, diagram and SF diagram of the beam.
[CSE-Mains, 2013, ME : 10 Marks]
Solution:
BM diagram is of a beam subjected to a building couple Mat C, at a distance of 3 m from A. Length of
beam is 5 m.
M = 12 + 8 = 20 kNm (anticlockwise)
Moment at B, RB x5 = 20
56 P. lAS & IFS (Objective & Conventional) Previous Solved Questions MADE EASY

RB = 4 kN1..
RA = 4 kNT (to balance RB)
Mc = 4 x 3 = 12 kNm
Mc = 12 - 20 = -8 kNm

SFD

Fig 4.8

SF diagram
FAB = +4 kN (constant)

Q.4.6 The shear force diagram of a beam is shown in the figure. Draw bending moment and load
diagrams.

2P
B P/3
+ D
E
2P/3

2a .1- a•-•-1

Fig. 4.9
[CSE-Mains, 2002, CE : 12 Marks]

Solution:
Beam is supported at B and D
Reaction
P
RB = 2P —
+ 7P
3 3
2P 8P
RD = 2P + — =
3 3
Strength of Materials Shear Force and Bending Moment Diagrams 1 57

7P 8P
Total load on beam = — + — = 5P
3 3

Rate of loading between A and B = 2P sl,


a

2P P 2P
P 2P ,
Vertical load at C = - + — = P 4, 1". a
3 3 .
A.,,rem.„, B 1, C
11 1 E
Vertical load at E = 2P1 H'— a a -044- 2 a
BMD RB RD
(a)
A4A = 0

2P a
-- x ax- = -P.a
MB= a 2 — 2Pa
— Pa 3
ME= ° — 2Pa
MD = -2Pa BMD

Fig. 4.10
8P 2Pa
Mc= -2P x 3a + — x 2a = -
3 3
Verification of reaction
Moment about B

2P a
ax3RD+- xax- = Pa + 8Pa
a 2
3RD = 8P

8
RD = iP

Q.4.7 A beam ABCD, 8 m long hinged at end A and roller


supported at end D carries transverse loads as
20 kN
shown in the figure 4.11(a). Determine support
reactions and draw BM diagram for the beam.
Solution:
A load at end of the lever equal to 20 kN can be 2m
replaced by a load of 20 kN and a CWmoment of (a)
40 kNm at point C. 20 kN 20 kN

Reaction
Taking moments at A
2 x 20 + 20 x 4 + 40 = 8 RD
RD = 20 kN
RA = 20 + 20 - 20 = 20 kN
BMD MA =O
MB = +20 x 2 = +40 kN
Mc = 20 x 4 - 20 x 2
= 40 kNm
Mc = 40 + 40 = 80 kNm
Fig. 4.11
58 0. IAS & IFS (Objective & Conventional) Previous Solved Questions MADE EASY

Q.4.8 Construct the SF and BM diagrams for the beam shown in the figure 4.12.
50 kN 40 kN
20 kNm 100 kNm

A D E F G

-id 0.5 1-4-- 3 m 1-1.4 1.5 m--1.-1 0.5 1 0.5


RB =45kN RE = 55 kN

Fig. 4.12
[CSE-Mains, 2006, ME : 20 Marks]

Solution:
Taking moments about B
20 x0.5 x 0.25+100+4.5 xRE = 50 x 3 +40 x5
2.5+ 100+4.5 RE = 150+200

0, 350 -102.5
RE - = 55 kN
4.5
RB = 20 x 0.5 + 50 + 40 - 55 = 45 kN
SF diagram
AB Portion = x x20
= 0 at 0
= -10 kN, at x = 0.5
BCD Portion
Fx = -10 + 45 = +35 kN (remains constant)
DE Portion = -10 + 45 - 50 = -15 kN (remains constant are DE)
EF Portion = -10 + 45 - 50 + 55 = +40 kNm

+ 40 kN

Fig. 4.13

BMD + 32.5 kNm

MA
MB = -20 x 0.5 x 0.25 = -2.5 kNm
Mc = -20 x 0.5 x 1.25 + 45 x 1 = +32.5 kNm
Mc = 32.5 - 100 = - 67.5 kNm
MF =
ME = -4 x 0.5 = -20 kNm - 67.5 kNm
MD =-40x 2+55 x 1.5 BMD
= -80 + 82.5 = +2.5 kNm Fig. 4.14
Strength of Materials Shear Force and Bending Moment Diagrams 4 59

Q.4.9 A beam ABCD, 6 m long hinged at both the ends A and


6 kNm 3 kNm
D, is subjected to moment 6 kNm (CW) at B and 3 kNm
(CCW) at C as: shown in figure 4.15(a). Sketch the
bending moment diagram of the beam. How many point AD
of contraflexure are then in the beam)
[CSE-Mains, 2009, ME : 5 Marks] 0.5 kN t

Solution: ho— 2m 2m 2m —0-1

Reaction: Moment about A, RA = 0.5 kN (a)

RD x 6 + 3 = 6 kNm +5 kNm
RD = 0.5 kN T
+4 kNm
RA = 0.5 kN
BMD MA = + 1 kNm
MB = -0.5 x 2 = -1 kNm
MB = -1 + 6 = 5 kNm
Mc = -0.5 x 4 + 6 = +4 kNm BMD
— 1 kNm
(b)
Mc = 4 - 3 = +1 kNm
Mo = Fig. 4.15
There is only one point of contraflexure in the beam.

Q.4.10 For the semicircular simply supported member shown


in the figure 4.16, find the value of bending moment at
a function of 0 and also draw its bending moment
diagram.
[CSE-Mains, 2013, CE : 15 Marks]
Solution:
Consider a section xx at angular
displacement 8 as shown
Taking moment about A, for reaction
Px 1.5R = 2R x RB
RB = 0.75P and so RA = 0.25P

Bending moment, Me = (R - Rcos0)


= 0 at x = 0 Fig. 4.16
= 0.0335 PR, at 0 = 30°
= 0.07325 PR, at 8 = 45°
= 0.125 PR, at 0= 60°
= 0.25 PR, at 0 = 90° 0.375 PR
Portion BC (0 = 0 - 60°)
3P
Me = — (R - R cos0)= 0.75PR (1- cos())

= 0 at 0 = 0
= 0.1005 PR, at 0= 30°
= 0.21975 PR, at 0 = 45°
1.5 R 0.5 R-0-1
= 0.375 PR (at point C), at 0 = 60°
BMD
At 90° M90 = 0.75 P(R)- Px 0.5 R
= 0.25 PR Fig. 4.17
60 0. lAS & IFS (Objective & Conventional) Previous Solved Questions MADE EASY

Objective Questions
Q.1 Which one of the following figure represents the Q.3 The beam is loaded as shown in the fig. 4.19.
correct shear force diagram for the loaded beam Select the correct BM diagram
shown in the figure 4.18.

a I.- 3a a 1.4
Fig. 4.19
Fig. 4.18

(a)
(a)

(b)
(b)

(c)

(d) (c)

[CSE-Prelims, ME : 1998]

Q.2 A cantilever carrying an uniformly distributed


load throughout its length. Select the correct
(d)
bending moment diagram of the cantilever

[CSE-Prelims, ME : 1999]
(a)
Q.4 If the beam shown in the fig. 4.20 is to have
zero bending moment at its middle point, the
overhang x should be

(b)
r w

(c)
Fig. 4.20
wL2 wL2
(a) (b)
4P 6P
(d)
wL2 wL2
(c) (d)
8P 12P
[CSE-Prelims, ME : 1999] [CSE-Prelims, ME : 2000]
Strength of Materials Shear Force and Bending Moment Diagrams 41 61

Q.5 Match List-I with List-II and select the correct Q.6 If the SF diagram for a beam is a triangle with
answer. length of the beam as its base, the beam is
List-I (a) A cantilever with a concentrated load at its
free end
(b) A cantilever with UDL over its whole span
A.
f Q SI tT (c) Simply supported with a concentrated load
at its mid point
(d) Simply supported with a UDL over its whole
span
B.
[CSE-Prelims, ME : 2000]

Q.7 The bending moment diagram for a simply


P Q supported beam is a rectangle over a larger
portion of the span except near the supports.
What type of load does the beam carry?
(a) A uniformly distributed symmetrical load
Q
over a larger portion of the span except near
D.
F F
t
F F
the supports
(b) A concentrated load at mid span
List-II (c) Two identical concentrated loads equidistant
from the supports and close to midpoint of
P T
the beam
(d) Two identical concentrated loads equidistant
1. from the mid span and close to supports
[CSE-Prelims, ME : 2007]

Q.8 Which one of the following is the correct


statement?

dM
If for a beam, — 0 for its whole length, the
dx
beam is a cantilever
(a) free from any load
R S T (b) subjected to a concentrated load at its free
4. p Q end
(c) subjected to an end moment
(d) subjected to a UDL over its whole span
S T
[CSE-Prelims, ME : 2007]
R
5. Q.9 Which one of the following is the correct answer?
P Q The point of contra flexure in a beam is a point
Codes: on its length where
A BCD (a) the shear force is zero
(a) 4 2 5 3 (b) the bending moment is maximum
(b) 1 4 5 3 (c) the bending moment changes its algebraic
(c) 1 4 3 5 sign and is zero at that point
(d) 4 2 3 5 (d) the shear force changes its algebraic sign
[CSE-Prelims, ME : 2001] [CSE-Prelims, ME : 2006]
62 I,. lAS & IFS (Objective & Conventional) Previous Solved Questions MADE EASY

Q.10 A overhanging beam AB has simple supports


at D and E as shown in the figure below. It
carries concentrated loads P at its free ends
and a uniformly distributed load of intensity
w (N/m) over its length DE. If bending moment
at the middle point C of the beam is to be zero,
then what is the value of P?

P (N) P (N)
Fig. 4.22

(a) 8(W + 2 (b) 4(2W+ w)


J

(c) 212W + —
w (d) 4(W + ;)
2 11
Fig. 4.21 [CSE-Prelims, ME : 2008]

w/ Q.14 A cantilever beam is subjected to moments as


(b) shown in the given figure. The BM diagram for
the beam will be
w/
(d) 16 30 kNm 50 kNm
[CSE-Prelims, ME : 2006]

dM
Q.11 For a cantilever = a , constant for its whole 2m 2m
dx
length. What is the shape of the SF diagram for Fig. 4.23
the beam A
(a) rectangle (b) triangle (a) 20 kNm
50 kNm
(c) a parabola (d) a hyperbola
[CSE-Prelims, ME : 2008]

Q.12 SF diagram for a simply supported beam is a


rectangle with its longer side equal to beam (b) 80 kNm 50 kNm

length. What type of load is acting on the beam?


(a) Concentrated load at its mid span A
(b) UDL over its whole span 20 kNm
(c) 50 kNm
(c) Concentrated load alongwith a couple at a
point on beam length
(d) Couple at a point on the beam length
[CSE-Prelims, ME : 2008]
50 kNm
Q.13 A beam AB of length 1 and weight wsupports a (d) 80 kNm

load Wat its free end Band is hinged at its end


A. A wire CD supports the beam at the mid [CSE-Prelims, CE : 2001]
point C making an angle of 30° with the
horizontal. What is the value of tension T in the Q.15 A cantilever is loaded as shown in the below
wire CD? figure. The bending moment along the length is
Strength of Materials Shear Force and Bending Moment Diagrams 63

p Q.18 Consider the following statements in regard to


a the shear force diagram for an overhanging
beam supported at A and C
P
Fig. 4.24 6 kN

(a) uniform 2 kN 2 kN 2 kN
(b) uniformly varying
(c) zero A 1-4-- 2m 2m I.- .1 m
B C
(d) concentrated at the free end
3 kN
Q.16 Consider the following statements:
7 kN
1. Point of contraflexure is the point where the
bending moment is maximum Fig. 4.26
2. Point of contraflexure is the point where the
1. The beam is carrying a uniformly distributed
bending moment changes sign
load of 2 kN/m throughout
3. Point of contraflexure is the point where the
2. The beam is carrying a uniformly distributed
shear force is zero
load of 2 kN/m over the supported span
Which of these statements is/are correct?
AC and concentrated load of 2 kN at the
(a) 1, 2 and 3 (b) 2 and 3
free end B
(c) 2 only (d) 1 only
3. The beam is carrying a uniformly distributed
[CSE-Prelims, CE : 2002]
load of 2 kN/m over the supported span
Q.17 A cantilever AB carries loading as shown in the AC, and concentrated load of 5 kN at the
figure. The BM diagram for the beam is centre of supported span BC and also a
point load of 2 kN at the free end B.
4. The points of contraflexure occurs between
the supported region AC and nearer to
4 kNm support C.
F- 1m M Which of the above statements is/are correct?
(a) 1, 2, 3 and 4 (b) 4 only
Fig. 4.25
(c) 2 and 3 (d) 3 and 4
6kN-m [CSE-Prelims, CE : 2004]

Q.19 A propped cantilever beam shown in the figure


(a)
given above is having internal hinge at its mid
4kN-rn span. Which one of the following is the shape
6kN-m of bending moment diagram for the given
(-) 4kN-m loading?
(b)

6kN-m eWslY=NPWY-1e-
l

(c)
Fig. 4.27
4kN-m
Parabolic

6kN-m
(d) 4kN-m 4kN-m (a)
A
B
[CSE-Prelims, CE : 2003]
64 ► IAS & IFS (Objective & Conventional) Previous Solved Questions MADE EASY

Straight line Q.21 A cantilever beam of span 1 carries a uniformly


varying load of zero intensity at the free end
(b) A and w per metre length at the fixed end. What
does the integration of the ordinate of the load
Parabolic diagram between the limits of free and fixed
ends of the beam give?
(a) Bending moment at the fixed end
(c)
— (b) Shear force at the fixed end
A
(c) Bending moment at the free end
(d) Shear force at the free end
[CSE-Prelims, CE : 2006]
(d)
A Q.22 In the cantilever beam shown below, what is
the percentage of bending moment at 1/2 with
[CSE-Prelims, CE : 2005]
respect to the maximum bending moment at
Q.20 A simply supported beam AB is loaded as the fixed support?
shown in the figure given below. CDE is a rigid
w/unit length
member. A load 4 kN is applied at E. Which
one of the following is the SFD for the beam?
C1.4— 1/2
4 kN
.1
D E 0.5m Fig. 4.29
C t (a) 15% (b) 20%
k- (c) 25% (d) 30%
0.5 m
2m —141 2m [CSE-Prelims, CE : 2006]
Fig. 4.28 Q.23 Match List-I with List-II and select the correct
answer using the code given below the lists:
2 kN
List-I
A. Cantilever beam
(a) 2 kN
B. Overhanging beam
(b) C. Fixed beam
D. Simply supported beam
1.5 kN List-II
1. At the supports, shear force exists, BM is
2.5 kN
zero
(c) 2. Deflection is zero, but BM exists at the
supports
2.5 kN 3. Shear force and BM exist at the supports
4. BM exists, deflection is zero at the supports
1.5 kN
Codes:
A BCD
1 kN (a) 4 1 2 3
(b) 2 3 4 1
(d) 3 kN (c) 4 3 2 1
(d) 2 1 4 3
[CSE-Prelims, CE : 2005] [CSE-Prelims, CE : 2006]
Strength of Materials Shear Force and Bending Moment Diagrams 65

Q.24 What is the maximum bending moment at mid Codes:


span of the beam given below? A B C
r W (Rate of loading)
(a) 3
(b) 4
2
1
4
3
(c) 3 1 4
(d) 4 2 3
[CSE-Prelims, CE : 2007]

Q.26 The figure shows the shear force diagram for


an overhanging beam ACDB.
55 kN

Fig. 4.30

(a)
wi2
(b)
1/4/12
2 6 25 kN
3 m —0,40— 3 m
w/2 Wi t
(c) (d) Fig. 4.31
12
[CSE-Prelims, CE : 2007] Consider the following statements with respect
to the above beam:
Q.25 Match List-I (Loaded Beam) with List-II 1. The beam has supports at A and D
(Maximum bending moment) and select the 2. The beam carries a concentrated load at C
correct answer using the code given below the of 25 kN
lists: 3. Bending moment at D is 15 kNm
List-I 4. The beam carries a uniformly distributed

r w/m load of 80 kN over the portion AC.


Which of the statements given above is/are are
A. correct?
(a) 1, 2 and 4 (b) 1 only
L H
(c) 2, 3 and 4 (d) 1 and 4 only
w/m [CSE-Prelims, CE : 2009]

Q.27 A simple supported beam AB of span 2 m is


B. f loaded as shown in the figure. Which one of the
L
following pairs corresponds to SFD and BMD
for the beam?
1 kN-m 3 kN-m

C. G E)
2m
'Alm

List-II Fig. 4.32

wL2 wL2 2 kN (+) 2 kN


1. 2.
8 12

wL2 wL2 (a)


3. 4. (—) 3 kN-m
2 kN-m
2 6
66 0. 1AS & IFS (Objective & Conventional) Previous Solved Questions MADE EASY

2 kN 3 kN-m 1 kN (-) 1 kN

(b) (d)
(-) 2 kN-m (-) 2 kN-m
1 kN-m 1 kN-m

[CSE-Prelims, CE : 2010]
1 kN (-) 1 kN Answers
1. (a) 2. (c) 3. (d) 4. (c) 5. (d)
(c)
(-) 3 kN-m 6. (b) 7. (d) 8. (c) 9. (c) 10. (c)
1 kN-rn
11. (a) 12. (d) 13. (d) 14. (a) 15. (a)
16. (c) 17. (b) 18. (d) 19. (d) 20. (a)
21. (b) 22. (c) 23. (c) 24. (c) 25. (c)
26. (d) 27. (c)

Explanations
1. (a) 4. (c)
Reactions = Weach.

C IW

L/2-0-1-4- LJ2
wt tw
H.- a -1. 3a -I- a Fig. 4.37
) (p 4. wL2
-PI x
wic - 2 2 )2 2x4

SFD PL wL2 wL2


Fig. 4.34
- - Px +-+ -
2M 2 4 8
2. (c) wL2
-Px + 0
-w 8
wL2
x=
8P
5. (d)
ABCD
4 2 3 5
Fig. 4.35
6. (b)
3. (d)

BMD SFD

Fig. 4.36 Fig. 4.38


Strength of Materials Shear Force and Bending Moment Diagrams 41 67

7. (d) 12. (d)

a .1. (L — 2a)
M
L

SFD
Fig. 4.39 Fig. 4.43
8. (c) 13. (d)
dM
=0
dx

Fig. 4.40
dM
0 end moment M
dx ,

9. (c)
At point of contraflexure, bending moment
changes its algebraic sign and is zero at that point.

10. (c) 14. (a)


30kNm

cvmeme_v_w, 1 50 kNm

L L
2 2
Fig. 4.41
A
wL) L wL2
- P x 1.5L +(P+— - - 0 20
2 2 8
50 kNm
L wL2 wL2 30
- 1.5PL+ + - = 0
2 4 8 Fig. 4.45
wL2
PL = 15. (a)
8
Uniform
WL 2 kN
P=
8

11. (a) 4 kNm

dM
= a constant = SF is constant Ii M .1. 1m
dx

(-)
4 kNm
BMD
SFD 6 kNm
Fig. 4.42 Fig. 4.46
68 0. lAS & IFS (Objective & Conventional) Previous Solved Questions MADE EASY

16. (c) 20. (a)


2 only
2. Point of contraflexure is the point where 4 kN
bending moment changes sign. DrIE
A 0.5 m
17. (b)
11114111
0.5 m
RA = 2 kNt RB = 2 kN
2m

Fig. 4.50

(-) 10
4 kNm R8= = 2.5 kA

6 kNm RA = 1.5 kN
BMD

Fig. 4.47 z
(+)
18. (d)
3 and 4 are correct statements ko-- 1.5 m
SFD
5 kN 2 kN Fig. 4.51

21. (b)
A
SF at fixed end.
6 kN
}-0— 2 m --144— 2 m —1±1- 1 m-0-1

BMD -2 kNm

Fig. 4.48 SFD

19. (d)
Fig. 4.55
Parabolic
22. (c)
25%

I-0— 2 m 2m

Fig. 4.56
2
w/
MA — 2
BMD
wit
Fig. 4.49 M, - 8
Strength of Materials Shear Force and Bending Moment Diagrams 4 69

23. (c) 25. (c)


List-I w/ 2
A. Cantilever beam A. 3.
2
B. Overhanging beam 2
w/
C. Fixed beam B. 1.
8
D. Simply supported beam
2
List-II w/
C. 4.
4. BM exists, deflection is zero at the supports 6
3. Shear force and BM exist at the supports
26. (d)
2. Deflection is zero, but BM exists at the
1. Beam has supports at A and D.
supports
4. Beam caries a UDL of 80 kN over portion AC.
1. At the supports, shear force exists, BM is
zero 27. (c)

24. (c) 1 kNm 3 kNm

2m ,
RA = 1 kN /4 4 RB = 1 kN

1 kNm (-) 1 kNm

SFD
Fig. 4.57
wi i wi i wit wg 3 kNm
(-)
c 4 X 2 4 x 6 8 24 1 kNm

wit BMD
= 12
Fig. 4.58

NM MI
05
CHAPTER
Theory of Simple Bending

Q.5.1 State the restrictions or assumptions made in deriving the formula for theory of bending
M a E
I y R
[CSE-Mains, 1998, ME : 15 Marks]

Solution:
Following assumptions are taken while deriving the flexure formula:
1. The material of the beam is homogeneous and isotropic.
2. Young's modulus of the material in tension = Young's modulus in compression.
3. Material is considered as consisting of layers.
4. Each layer is independent to extend or contract irrespective of the layers above or below it.
5. Beam is initially straight.
6. Elastic limit of the material i.e., ae is not exceeded.
7. Beam is symmetrical about plane of bending.
8. Transverse sections which are plane before bending remain plane after bending.

Q.5.2 A wire of diameter d is wound round a cylinder of diameter D. Determine the bending stress
produced in the cross-section of the wire. Hence or other wise find the minimum radius to which
a 1 cm diameter circular rod of high tensile steel can be bent without undergoing permanent
deformation. Take yield stress = 17000 kg/cm2 and E. 2 x 106 kg/cm2. What is the magnitude of
BM necessary for this?
[CSE-Mains,1991, ME : 20 Marks]
Solution:
Wire diameter = d

Drum radius, R=
2
Bending stress, ab
_ E _ 2E
b
d/2 R D
20 b 2E
d

Gb E-5 Bending stress in wire


Strength of Materials Theory of Simple Bending 71

Now, ab = 17000 kg/cm2 E= 2 x 106


d= 1 cm
Ed 2x106 x1
= =117.647 cm
ab 17000
It X13
M = ab Z = b X 1°3- =17000 x
32 32
M = 1668.97 kg-cm

Q.5.3 Compare the moments of resistance for a given maximum bending stress of a beam of square
section placed (i) with two sides vertical and (ii) with a diagonal vertical. The bending in each
case is parallel to vertical plane.
[CSE-Mains, 2012, CE : 12 Marks]

Solution:
M = Zab
Section-I (with two sides vertical)
a3
1 --
z1 = 6
Section-II (with a diagonal vertical)
a 4 2 a3
Z2 - ,___
12 X ,/2a 6J
Say ab = Maximum bending stress
a3
M1 = bX 6

a3
M2 = ab x 6,//- a —h
2
II

= =1.414 Fig. 5.1


M2

Q.5.4 Compare the bending strengths of three beams of same material, same weight and same depth,
if one of them has solid rectangular area 6 x 20 cm, second beam is a hollow rectangular section
having a wall thickness of 2 cm. The third beam has I-section of equal flanges have web and
flange thickness equal to 2 cm.
[IFS 2011, ME : 10 Marks]
Solution:
For solid rectangular area 6 x 20 cm

6 x 202 [-4-- 6 mm
Z1 = = 400 cm3
6
Hollow rectangular section (assume width 8)
208 - (8- 4) x 16 = 6 x 20 = 120 cm2 (area same) - 20 mm
20B- 16B + 64 = 120
4B = 56 = 14 cm
[14x203 10x163 ]
Fig. 5.2
4- 10 12 12
1 1
= — x — [112000 - 40960] = 592 cm3 ...(ii)
10 12
7 2 a. 1AS & IFS (Objective & Conventional) Previous Solved Questions MADE EASY

I B

2 cm

20 cm
I
16 cm 20 cm

I
2 cm
B
TT
Fig. 5.3 Fig. 5.4
I-section
28 + 2B + 16 x 2 = 120 cm2
4B= 88
= 22 cm

203 63]
= = [176000 - 81920] = 784
- 10 [2212 2012 120
Beam strength is proportional to section modulus
Zi : Z2 : Z3 = 400: 592: 784
= 1 : 1.48 : 1.96

Q.5.5 Compute second moment of area of the plane lamina shown


in the figure 5.5 about an axis parallel to the base and
passing through the centroid.
[CSE-Mains, 2012, ME : 20 Marks]

Solution:
Area, Al = 22 x 16 = 352 cm2
22 x 9
A2 — = 99 cm2
2
[4-16 cm --+-- 9 cm
TcR2 n x 82
A3 = = = 100.531 cm2
2 2 Fig. 5.5
4R 4 x 8
7 for semi circle _ = = 3.395 cm (from diameter)
3n 3 x7c
Location of centroid (from bottom)
99 x22
16 x 22 x11+ +100.531(22+3.395)
3
111
352+99+100.531
3872 + 726 + 25529.85
551.531
7150.985
- 12.966 cm
551.531
Moment of inertia of semi circular section about its own centroidal axis.
irR 4 itR2 (412
1x/x =
8 2 37r
Strength of Materials Theory of Simple Bending 73

nR 4 nR 2 16R2 ER 2 8R 4
x
8 2 9112 8 911
= R4 [0.3927 - 0.2829]
= 0.1098 R4
= 0.1098 x 84 = 449.741 cm4

1182 - 100.531 cm2


2

16 x 223 2 x 223 22 2
+352(12.966 11) + 36 + 99112.966 - -3--
/xx = 12 )
+449.741 + 25.1327 (22 + 3.395 - 12.966)2
= 14197.333 + 1360.535 + 2662 + 3141.338 + 449.741+ 15530.03
= 37340.98 cm4
= 3.7341 x 10 cm4

Q.5.6 The shear force diagram for rectangular cross-section beam


is shown in the figure 5.6. Width of the beam in 100 mm and
depth is 250 mm, determine the maximum bending stress in
the beam.
[CSE-Mains, 1995, ME : 30 Marks]

Solution: 5 kN
SF diagram is only on one side. This is the case of a cantilever
3 m long AB, fixed at A and free at end B carrying UDL of 5 kN/m 5 kN T 5 kN 5 kN
over BC= 1 m, a point load of 5 kN at Cand a point load of 5 kN LW-4/
at D. Loading diagram is shown in the figure as above.
Maximum bending moment occurs at fixed end A 1-0-1 m -1+-1 m -44-1 m
MA = 5(2.5) + 5 x 2 + 5 x 1
Fig. 5.6
= 12.5 + 10 + 5 = 27.5 kNm
= 27.5 x 106 Nmm = ob x Z
Z = Section modulus
2 100 x 2502
- bd = 1.04166 x106 mm3
6 6
27.5 x106
as - = 26.4 N/mm 2 (Max. bending strtess in beam)
1.04166 x 10'

Q.5.7 A cantilever beam of cross-section 50 mm x 100 mm in subjected to a compressive force parallel


to the longitudinal axis of the beam at the free end. The point of application of the load is at 10 mm
above the bottom surface of the vertical centroidal axis. Span of the beam is 4 m. Calculate the
maximum stress in the beam.
[CSE-Mains, 2012, CE : 15 Marks]
Solution:
b = 50 mm, d = 100 mm
bd 3 = 50 x 1003
= 4.1667 x106 mm4
12 12
Eccentricity of loading, e= 50 - 10 = 40 mm
74 ► lAS & IFS (Objective & Conventional) Previous Solved Questions MADE EASY

Pin Newton, A —0-150


50
Bending moment
M = P x 40 Nmm 8 P 10 mm 4°
4m
My + 50 x 40P
Maximum bending stress, Fig. 5.7
= / 4.1667 x 106
= +480 x 106 P = ± 4.8 x 10-4 P NImm2

Direct shear, a = = 2 x 10-4 P N/mm 2 (Comp.)


d area 50 x 100
amax at edge B = —(4.8 + 2) x 10-4 P. —6.8 x 10-4 P N/mm2 (Comp.)
6max at edge A = +4.8 x 10-4 — 2 x 10-4 = 2.8 x 10-4 P N/mm2 (Tensile)

Q.5.8 Compare the flexure strength of the following three beams of equal weight per unit length.
1. I-section 300 mm x 150 mm with flanges 20 mm thick and web 12 mm thick.
2. Rectangular section having depth twice the width
3. Solid circular section
[CSE-Mains, 2012, ME : 15 Marks]

Solution:
8
Having same material and equal weight per unit length means same area
of cross-section for all three beams. 20

I-section
Area of cross-sections = 2 x 150 x 20 + 260 x 12
260
= 6000 + 3120 = 9120 mm2

150 x 3003 138 x 2603


zr 20
12 12
—150 --xd
337.5 x 106 — 202.124 x 106
= 135.376 x 106 mm4 Fig. 5.8

135.376 106
= 0.9025 x 106 mm3
150
Rectangular section
Area = 282 = 9120 (same area of cross-section)
B = 67.53 mm
D = 135.055 m D = 28
BD2 67.53 x 135.0552
4 6
6 =
= 0.20529 x 106 mm3
Circular section
Fig. 5.9
Tcd2
— 9120
4
Diameter, d = 107.758 mm

nd3 x (107.7583 )
Z3 — — 0.122845 x 106 mm4
32 32
: Z2 : Z3 = 0.9025: 0.20529: 0.122845
= 7.346 : 1.671 : 1
Strength of Materials Theory of Simple Bending 4 75

Q.5.9 A beam is of square section with diagonals 60 mm long, vertical


and horizontal, as shown in figure 5.10. Shear force at a particular
section is 5 kN. What is the shear stress at layer AB.
[CSE-Mains, 2009, ME : 15 Marks]

Solution:
Square section
Diagonal, d = 60 mm

d4 60 4 60 mm '-J
= 27 x 104 mm4
= 48 48
Fig. 5.10
Layer, AB, breadth = 30 mm = b = d/2
1
Area, OAB = - x 30 x 15 = 225 mm2
2
15
y = 15+ — = 20 mm
3
ay = 225 x 20 = 4500 mm3
F= 5 kN = 5000 N

Fay 5000 x 4500


= 2.778 N/mm2
TAB = lb 27 x104 x 30

Q.5.10 Two long wooden planks form a T-section of a beam as shown in


200
figure 5.11. If this beam transmits a constant vertical shear of 3000 N,
find the necessary spacing of the nails between the two planks to 50

make the beam act as a unit. Assume that allowable shear force per # a a
r•-•
co
nail is 700 N.
4G x 250
Solution:
Location of G of section = 162.5

200 x 50 x100+200 x50 x 225


2 x200 x 50
I
= 50 + 112.5 = 162.5 mm
y2 = 250 - 162.5 = 87.5 mm Fig. 5.11
Moment of inertia, I

50 x 2003 2 200 x 503


= + 50 x 200(162.5 100) + +200 x 50 (87.5 - 25)2
12 12
= 33.33x 106 + 78.125 x 106 + 2.083 x 106 = 113.538x 106 mm4
Shear stress along a-a, nail section

Fay
ti
aa = Ib
b= 50 mm, I =113.538 x 106 mm4, F= 3000 N
ay about neutral axis = 200 x 50 x (87.5 - 25) = 200 x 50 x 62.5 = 6.25 x 105 mm3

3000 x6.25 x10 5


as x 50 - 0.33 N/mm2
113.538 x106
Allowable shear force = 700 N
76 ► 1AS & IFS (Objective & Conventional) Previous Solved Questions MADE EASY

Spacing between nails = S mm (say)


700 = S x 0.33 x 50
S, spacing = 42.42 mm
Nails are fixed at spacing of 40 mm

Q.5.11 An I beam with the following dimensions is subjected to a shearing force I


5.5 mm
of 20 kN.
Flange: breadth = 50 mm, web thickness = 3.5 mm
Thickness = 5.5 mm, web depth = 189 mm
x
I
189 mm
Area of cross-section = 9.4 x 102 mm2
Ad/ = = 220 x 104 mm4
Calculate the value of the transverse shear stress at the neutral axis x-x
I
5.5 mm
and at the top of the web. 1-4— 50 I
[CSE-Mains, 2012, ME : 12 Marks] Fig. 5.12
Solution:
Shear force, F= 20 kN
= 220 x 104 mm6
b = breadth = 3.5 mm
Shear stress at neutral axis
b= 3.5 mm

a)7 = 50x 5.5 x(100 - 2.25)+ 3.5 x 189(189)


= 26743.75 + 1562.9375 = 42371.69 mm3
20000 x 42371.69
= 110.056 N/mm2
220 x104 x3.5
20000)26743.75
Shear stress at top edge of web= = 69.46 N/mm2
220x104 x3.5

Q.5.12 A horizontal beam of square cross-section is so placed that the loading in the transverse plane is
along one of the diagonals of length d. If the shear A
force at a section of the beam is S. Draw shear 2s
stress distribution diagram for the section and T a 2
a d
2.25s
indicate the position and magnitude of maximum T 2
d -t d
shear stress on it.
ir
[CSE-Mains, 1998, ME : 30 Marks] I. j M8X
Solution:
Square section Shear stress
Diagonal = d distribation
d Fig. 5.13
Side =
4
(d ) 1 d4
x
= 12 - 48
Shear force = S
Take a section at a distance y. from NA, xx
Area Aaa (distance of centroid) from xx
Strength of Materials Theory of Simple Bending 4 77

(d -y )
Height of the triangle Aaa =
2

Base of the triangle = 21d -y)


2

id )1 (d ) (d )2
Area = y 2 x2 2 y 2 y
2

d 2 2y d
Ay =
(2 - Y ) (V d

Breadth, b=
2( 2-y )
Say
Transverse shear stress, T=
Tu b

s(d _ )(2yd ) 48
y + x
3 6)

d4 x2(2-y)

[ d 2 4_ dY 2 v 2
=
d 4 ' 2 - '

d 8S[ d2d d d2] 8S [d2 d 2 d 2 2S


At- = Y T= — +— x— -2 x
4 ' d4 4 2 4 16 d4 4 8 8 = d2

8S [d2 d2 d2]
At — = y Outer edge, =o
2 ' TA =
d4 4 4 2

8S d 2 2S
At y = 0, TNA =
d 4 X 4 —d2
tmax
d'r _
For Tma,„ 0 —
d - 4y) = 0 Refer equation (i)
dy - = (2
d
Y =
8
8S [d2 d d d2] 9S s
Tom _ — +— x— -2x— = = 2.25x—,
d" 4 2 8 64 4d2 d'

Q.5.13 An I-beam has flanges 10 cm wide and 1 cm thick, and the web 12 cm high and 1 cm thick. At a
section of this beam act a bending moment of 1000 kg-m and a shear force of 10,000 kg, find the
normal and shear stresses at the following points on the vertical center line.
(i) Top of the flange
(ii) In the web at the junction with the flange
(iii) At the neutral axis
[CSE-Mains, 1999, ME : 20 Marks]
78 0. lAS & IFS (Objective & Conventional) Previous Solved Questions MADE EASY

Solution:
I-section shown in the figure 5.14 I
1cm

About neutral axis,


= 10 x 143 9 x 123
12 12 I
12 cm

(i) Bending normal stress


= 2286.667 - 1296 = 990.667 cm4
M = 1000 kg-m = 1000,00 kg-cm
I
1 cm

GA =
1000,00 x 7
= 706.6 kg/cm2'
I-4— 10 —0-1 I
990.667
Fig. 5.14
1000,00 x 6
• = = 605.65 kg/cm2
990.667
ac = 6NA=Q
(ii) Shear stresses
TA = 0
For TB, ay = 10 x 1 x 6.5 = 65 cm3
Breadth, b = 1 cm
F = 10,000 kg
Fay _ 10,000 x 65
656.12 kg/cm2
Ib 990.667 x 1

tic ay = 10 x 1 x 6.5 + 1 x 6 x 3=65 + 18 = 8.3 cm3


b= 1 cm
10000 x 8.3
T - = 837.82 kg/cm2
C 990.667 x 1

Q.5.14 Simply supported beam of 3 m span is subjected to loads


as shown in figure 5.15(a) the beam is of I-section and all
of the dimensions are in mm. Determine the principal
stresses at point D in the web. The section is located at a
distance of 1 m for right hand support. 2m 1m
-4
[CSE-Mains, 2013, ME : 25 Marks] 4.333 kN 5.667 kN
(a)
Solution:
Reactions, 2 x 3x 1.5 +4 x 2= 3RB Fig. 5.15 (a)
17 = 3RB
Reaction,
Reaction,
RB = 5.667 kN
RA = 2 x 3 + 4 - 5.667 I
10 mm

Mc = Bending moment at C
= 10 kN - 5.667 = 4.333 kN

= 4.333 x 2-2 x 2 x 1
40 mm
50 mm
I
180 mm
= 8.666 - 4 = 4.666 kN-m 15m
= 4.666 x 106 Nmm (Sagging)
10 mm
Shear force at C
Fc = 4.333 - 2 x 2 = 0.333 (just left to C) F°—
(b)
Fc = 0.333 - 4 = -3.667 kN (just right to C)
Jump shear force at C = 4 kNI Fig. 5.15(b)
Strength of Materials Theory of Simple Bending 4 79

c x Y E) Mc x 50
ab, bending stress - M
l x„

75 x 2003 60 x 1803
xr = 50 x106 -29.16 x106
12 12
= 20.84 x 106 mm4

4.666 x 106 x 50
(ado -
20.84 x 106
= -11.20 N/mm2 (sagging moment) compressive
Shear stress at D
Fay
'co = when, b = 15 mm
lb
ay (about neutral axis) = 15 x 40 x 70 + 75 x 10 x 95 = 113250 mm2
F = 4000 N

113250 x 4000
= 1.45 N/mm2
- 20.84x106 x15
Principal stresses

11.20 44 11.20)2

P1 , P2 = 2
+0.45)2
l 2
11.385 N/mm
= -5.6 ± V31.56 + 2.1025 2
1.45 N/mm
= -5.6 ± 5.79 2
1.45 N/mm
p1 = -11.385 N/mm2
p2 = +0.19 N/mm2

Objective Questions
Q.1 A cantilever beam of rectangular cross-section
is 1 m deep and 0.6 m thick. If the beam were
to be 0.6 m deep and 1 m thick then the beam
would
(a) be weakened 0.5 time
(b) be weakened 0.6 time
(c) be strengthed 0.6 time
(d) have the same strength on the original beam
because the cross-sectional area remains
the same
Fig. 5.16
[CSE-Prelims, ME : 1999]
(a) zero (b) -30 MPa
Q.2 If the T-beam cross-section shown in the given (c) -80 MPa (d) 0.50 MPa
figure 5.16 has bending stress of 30 MPa in the
Q.3 The distribution of shear stress of a beam in
top fibre, then the stress in the bottom fibre
shown in the figure 5.17. The cross-section of
would be (G is centroid)
the beam is
80 0. IAS & IFS (Objective & Conventional) Previous Solved Questions MADE EASY

Q.7 By conjugate beam method, the slope at any


section of actual beam is equal to
(a) El times the SF of the conjugate beam
(b) EI times the BM of the conjugate beam
(c) SF of the conjugate beam
(d) BM of the conjugate beam
[CSE-Prelims, ME : 2002]
Fig. 5.17
Q.8 Select the correct shear stress distribution
(b) T
diagram for a square beam with a diagonal in a
(d) /\ vertical position

Q.4 A uniform bar lying in x-direction is subjected


to pure bending. Which one of the following
tensors represents the strain variation when
(a) (b)
bending moment is about z-axis (p, q, and x
constant)
py 0 0 py 0 0)
(a) 0 qy 0 (b) 0 qy 0
0 0 ry 0 0 0,
(c) (d)
py 0 0 py 0 0
(c) 0 py 0 (d) 0 qy 0
0 0 py, 0 0 qy [CSE-Prelims, ME : 2002]
[CSE-Prelims, ME : 2001] Q.9 A pipe of external diameter 3 cm and internal
Q.5 Two identical planks of wood are connected by diameter 2 cm and of length 4 m supported at
bolts at a pitch distance of 20 mm. The beam its ends. If carries a point load of 65 N at its
is subjected to a bending moment of 12 kNm center-section, modulus of pipe will be
and the shear force in bolts will be 657E 3 65/r 3
(b) 32 cm
(a) 64 cm
I
10 657t 3 657t 3
(c) cm (d) cm
10 96 124
I [CSE-Prelims, ME : 2002]

Q.10 In a loaded beam under bending


Fig. 5.18 (a) both the maximum normal and maximum
(a) zero (b) 0.1 kN shear stresses occur at the skin fibres
(c) 0.2 kN (d) 4 kN (b) both the maximum normal and maximum
[CSE-Prelims, ME : 2001] shear stresses occur at the neutral axis
(c) the maximum normal stress occurs at the
Q.6 A straight bimetallic strip of copper and steel
skin fibres and the maximum shear stress
is heated. It is free at ends. The strip will
occurs at the neutral axis
(a) expand and remain straight
(d) the maximum normal stress occurs at the
(b) not expand but bend
neutral axis while the maximum shear stress
(c) expand and bend also
occurs at the stress fibres
(d) first only
[CSE-Prelims, ME : 2003]
[CSE Prelims, ME : 2002]
Strength of Materials Theory of Simple Bending 81

Q.11 Consider the following statements: Q.15 Three beams of 100 mm x 100 mm section are
Two beams of identical cross-section but of made from RCC, aluminium and timber
different materials carry same bending moment respectively. These three beams are loaded
at a particular section, then identically with similar supports. Maximum
1. the maximum bending stress at that section bending stress will occur in which beam?
in the two beams will be same (a) Timber beam
2. the maximum shearing stress at that section (b) Aluminium beam
in the two beams will be same (c) RCC beam
3. maximum bending stress at that section
(d) The three beams will have same maximum
will depend upon the elastic modulus of the
bending stress
beam material
[CSE-Prelims, ME : 2008]
4. curvature of the beam having greater value
of E will be larger Q.16 A simply supported beam of span Land flexural
Which of the statements given above are rigidity EI carries a unit point load at its centre.
correct? What is the strain energy in the bema due to
(a) 1 and 2 only (b) 1, 3 and 4 bending?
(c) 1, 2 and 3 (d) 2, 3 and 4
[CSE-Prelims, ME : 2002] L3 L3
(a) (b)
16E/ 48 E1
Q.12 For the shear force to be uniform throughout the
span of a simply supported beam, it should L3 L3
(d)
carry which one of the following loadings? (c) 96E1 192 E1
(a) A concentrated load at mid span [CSE-Prelims, ME : 2009]
(b) UDL over the entire span
(c) A couple anywhere within its span Q.17 For a triangular section with base 'b' and height
(d) Two concentrated loads equal in magnitude ' h' , the ratio of the moment of inertia about an
and placed at equal distance from each axis passing through its vertex and parallel to
support its base to the moment of inertia about an axis
[CSE-Prelims, ME :2007] passing though its centre of gravity and also
parallel to its base would be
Q.13 Which one of the following is the preferable
(a) twelve to one (b) nine to one
cross-section of a beam for bending loads?
(c) six to one (d) four to one
(a) Circular (b) Annular circular
(d) I-section [CSE-Prelims, CE : 2001]
(c) Rectangular
[CSE-Prelims, ME : 2006] 0.18 A beam has rectangular section 100 mm x 200
Q.14 Consider the following statements for simple mm. If it is subjected to a maximum BM of
bending of beams: 4 x 107 Nmm, then the maximum bending stress
1. Neutral axis always passes through the developed would be
centroid of the beam cross-section (a) 30 N/mm2 (b) 60 N/mm2
2. Bending stress in a fibre is a longitudinal (c) 90 N/mm2 (d) 120 N/mm2
stress [CSE-Prelims, CE : 2001]
3. Bending stress is zero at the neutral axis
Q.19 Consider the following statements:
4. Shearing stress is always zero on the neutral
For each component in a flitched beam under
axis
the action of a transverse load.
Which of the statements given above are
correct? 1. the radius of curvature will be different.
(a) 1, 2 and 3 only (b) 1, 2, 3 and 4 2. the radius of curvature will be the same.
(c) 2, 3 and 4 only (d) 1 and 4 only 3. the maximum bending stress will be the
[CSE-Prelims, ME : 2006] same.
82 ► lAS & IFS (Objective & Conventional) Previous Solved Questions MADE EASY

4. the maximum bending stress will be Select the correct answer from the codes given
dependent upon the modulus of elasticity below:
of the material of the component. (a) Fig. (i) (b) Fig. (iii)
Which of these statements are correct? (c) Figl. (ii) and (iv) (d) Fig. (iii) and (iv)
(a) 1 and 3 (b) 1 and 4 [CSE-Prelims, CE : 2014]
(c) 2 and 3 (d) 2 and 4
[CSE-Prelims, CE : 2001] Q.24 In a thin uniform lamina having symmetrical
central axis as shown below, the distance of
Q.20 A timber beam is simply supported at the ends
centre of gravity from AD is
and carries a concentrated load at mid span.
The maximum longitudinal stress 'f is 12 N/mm2 A B

and the maximum shear stress 'q' is 1.2 N/mm2.


The ratio of span to depth would be
(a) 10 (b) 6
o- 1.5 m
(c) 5 (d) 4 co
[CSE-Prelims, CE : 2001] K T
0.21 The moment of inertia of a given rectangular
area is minimum about D C
(a) its longer centroidal axis
—►] 2 cm I-4— 8 cm ►H
(b) its polar axis
(c) its axis along the diagonal Fig. 5.21
(d) its shorter centroidal axis (a) 3 cm (b) 22/7 cm
[CSE-Prelims, CE : 2003] (c) 23/7 cm (d) 24/7 cm
Q.22 For a beam of rectangular section under bending [CSE-Prelims, CE : 2004]
the shear stress across the depth varies
Q.25 In the cross-section of a rectangular beam, what
(a) Linearly (b) Exponentially
(c) Hyperbolically (d) Parabolically is the ratio of the average shear stress to the
[CSE-Prelims, CE : 2003] maximum shear stress?
(a) 3/2 (b) 2/3
Q.23 A cantilever beam is loaded with a uniformly
(c) 4/3 (d) 3/4
distributed load of intensity w along its entire
[CSE-Prelims, CE : 2006]
length. The span of the beam is L. Which of the
following Mohr's circle diagram correctly Q.26 Which of the following are implied in the
represent(s) the state of stress above the assumption of plane sections remaining plane
neutral axis of the beam? (in simple bending)?
Shearing stress Shearing stress
1. Stress is proportional to the distance from
neutral axis.
2. Displacement is proportional to the distance
Normal Normal from neutral axis.
Tensile Tensile
stress stress 3. Strain is zero across the cross-section.
(i) (ii) 4. Strain is directly proportional to the distance
Shearing stress from neutral axis.
Shearing stress
Select the correct answer using the codes given
below:
(a) 1 and 4 only (b) 2 and 3 only
Normal Normal
Tensile Tensile (c) 3 and 4 only (d) 1, 2 and 4
stress stress
( v) Q.27 A beam of rectangular cross-section is to be
Fig. 5.20 cut from a circular beam of diameter D. What is
Strength of Materials Theory of Simple Bending -41 83

the ratio of the depth of the beam to its width (a) top edge of the flange
for maximum moment of resistance? (b) centre of the web
(b) (c) junction of the flange and web
(d) bottom edge of the flange
3 [CSE-Prelims, CE : 2010]
(d)
Q.32 Two tubes of outer diameter 20 mm and inner
[CSE-Prelims, CE : 2009] diameter 16 mm each are combined with a solid
bar of diameter 20 mm. Their centres make an
Q.28 A cantilever beam as shown in figure is
equilateral triangle of side 20 mm. The section
subjected to end moment M0. What is the
is of a cantilever, with solid tube on top,
deflection at the free end?
subjected to load at free end. If the maximum
tensile stress developed in section is 100 MPa.
21 1
What is the maximum compressive stress
developed.

-4m -IA 4

Fig. 5.22

10 M0 15 M0
(b)
(a) EI El

20 M0 30 M0
(d)
(c) El El
[CSE-Prelims, CE : 2010] Fig. 5.23
(a) 136 MPa (b) 116 MPa
Q.29 A mild steel beam is simply supported. It has
(c) 86 MPa (d) 73.2 MPa
constant moment of inertia =106 mm4. The entire
length of the beam is subjected to a constant Q.33 A beam of uniform strength refers which one of
BM of 107 Nmm, E = 2 x 105 N/mm2. What is the following?
the radius curvature of the bent beam in meters? (a) A beam in which extreme fibre stresses are
(a) 200 (b) 2 same at all cross-section along the length
(c) 20 (d) 0.2 of the beam
[CSE-Prelims, CE : 2010] (b) A beam in which the moment of inertia about
the axis of bending is constant at all cross-
Q.30 In a beam of solid circular cross-section, what
section of the beam
is the ratio of maximum shear stress to the
(c) A beam in which the distribution of bending
average shear stress?
stress across the depth of cross-section is
3 4 uniform at all cross-sections of the beam
(a) 7
1 (b)
(d) A beam in which the bending stress is
uniform at the maximum bending moment
3 2 cross-section.
(c) (d) 3
Q.34 The cross-section of a beam in bending is as
[CSE-Prelims, CE : 2010]
shown in the figure given below. It is subjected
Q.31 In case of a beam of I-section subjected to to a shear force acting in the plane of cross-
transverse shear force 'F, the maximum shear section. Which among the following figures
stress occurs a the shows the correct shear stress distribution
84 0. IAS & IFS (Objective & Conventional) Previous Solved Questions MADE EASY

across the depth of the cross-section of the (a) 3.2 (b) 3.3
beam? (c) 3.4 (d) 3.5
[CSE-Prelims, CE : 2006]
T
Q.37 The below diagrams show the details of two
2
simply supported beams B1 and 82. EI is
constant throughout the length and same for
both the beams. The beams have the same area
of cross-section and the same depth. What is
the ratio of maximum bending stress in 82 to
that in Bi?
Fig. 5.24
4 kN

A1y-v-v-v-
kN/m X3

111100.11
(b)
m -+- 2 m-od 1-4— 4 m
B1 B2

Fig. 5.26
(a) 4 (b) 1
(c) 2 (d) 1/2
(d) [CSE-Prelims, CE : 2009]

Q.38 Cross-sections of two beams A (600 mm x 200


[CSE-Prelims, CE : 2005] mm) and 8(800 mm x 60 mm) are shown in the
figure given below. Both the beams have the
Q.35 A mild steel plate is subjected to a moment M same material. By how many times is the beam
each at its ends such that it bends into an arc A stronger than the beam B in resisting
of a circle of radius 10 m. The plate has width bending?
60 mm and thickness 10 mm. E = 2 x 105
N/mm2. 200 mm-1-1 1
What is the maximum bending stress produced E
in the plate? E
0
co
(a) 100 MPa (b) 200 MPa
(c) 300 MPa (d) 400 MPa A 800 mm
T i
[CSE-Prelims, CE : 2005]

Q.36 What is the y-coordinate of the centroid of the


area ABCDEshown in the figure given below?

Fig. 5.27

(a) 80 (b) 60
(c) 50 (d) 25
[CSE-Prelims, CE : 2007]

Q.39 A mild steel structural beam has cross-section,


which is an unsymmetrical I-section. The overall
Fig. 5.25 depth of the beam is 250 mm. The flange
Strength of Materials Theory of Simple Bending 1 85

stresses at the top and bottom are 200 N/mm2 Answers


and 50 N/mm2, respectively. What is the depth 1. (b) 2. (c) 3. (b) 4. (d) 5. (a)
of the neutral axis from the top of the beam? 6. (c) 7. (c) 8. (d) 9. (c) 10. (c)
(a) 50 mm (b) 100 mm 11. (a) 12. (c) 13. (d) 14. (a) 15. (d)
(c) 150 mm (d) 200 mm 16. (c) 17. (b) 18. (b) 19. (d) 20. (c)
[CSE-Prelims, CE : 2008] 21. (a) 22. (d) 23. (a) 24. (b) 25. (b)
26. (d) 27. (b) 28. (c) 29. (c) 30. (b)
31. (b) 32. (b) 33. (a) 34. (c) 35. (a)
36. (c) 37. (d) 38. (d) 39. (d)

Explanations

1. (b) 6. (c)
0.6 x13
Z 0.1 m3
1- 6
1 x 0.63
Z2 = 0.036 m3
6
Z2
= 0.36 weakened by 0.64 times Fig. 5.29
Z
Strip will bend and expand.
2. (c)
7. (c)
Y2 = 3°
Slope = SF of conjugate beam.
Y1 = 80
f2 = 30 8. (d)
= -80 MPa Square with diagonal vertical
linearly proportional to distance y (d) is correct shear stress distribution
3. (b) d
Shear stress distributed is of T-section. max 8 from NA.
at -

4. (d)
9. (c)

Fig. 5.28
G oc y
a
Stress Fig. 5.30
= PY = E x
65n
= _ vpy = qy
VG TC
Ey= = / = 6 £31- 16) =
4 64

5. (a) 657C 1 657C 3


Z- X = - cm
Zero, as BM is constant and shear force is zero. 64 1.5 96
86 ► IAS & IFS (Objective & Conventional) Previous Solved Questions MADE EASY

10. (c)
bh3 2bh3 bh3
=
36 9 4
Ixx
=9
IGG
- ;Mx

I2h
3
Fig. 5.31 1

11. (a) 5
1 and 2 are correct statements.
b
12. (c) Fig. 5.33

18. (b)
b = 100, h = 200 mm
bh 2 100 x 2002 4
Z= = = x 106
6 6 6
M= 4 x 107 Nmm
RA 4 x 107 6
a — x = 60 MPa
b 4 106
SFD
Fig. 5.32 19. (d)
Flitched beam (Each component)
13. (d)
5. Radius of curvature will be the same
I-section has maximum moment of inertia for same 4. Maximum crb dependent on Eof component
area of cross-section.
20. (c)
14. (a)
w
1, 2, 3 are correct statements. at = —Lx 12
4 bd` —
15. (d)
1
Stress depends on BM and section modulus tia = 1.5 1
— =1.2
2
2 bd
independent of material properties.
12 10 = 1.5wL 2bd 2L
16. (c) or x
1.2 = bd 2 1.5W d
Strain energy,
3
L
1 wL W 2 L3
=—WX d 5
2 48E1 96EI
W= 1 21. (a)
L3 MI minimum about the longer centroidal axis.
U=
96E1
D
17. (b)
x
bh 3
GG
I = 36
Fig. 5.34
bh3 bh 14h2 )
BD 3
Ixx = 36 ± 2 9
x 12
Strength of Materials Theory of Simple Bending 1 $7

22. (a) 2
av
Parabolically 3
"c max -

26. (d)
1, 2 and 4 are correct statements.

27. (b)

Fig 5.35

23. (a)

Fig. 5.36
At a +ve normal stress Fig. 5.39
shear stress
b2 +d2 =
b = D sing, d =D cos()
bd 2 D3 sinOcos2 0
Z — —
6 6
dz D3
= = — sinO(1—sin2
ti O 6
Fig. 5.37 3
= dt ( 0 3 0))
sin — sin
d_1 6 k
24. (b)
1-1— 7 —""i 3
= [cos() — 3 sin2 cos 0] = 0
6
1
sin() = core =

d_
T b

8 cm —01 28. (c)

Fig. 5.38 L L

16x1+12x6 • G • G
x
28
16 + 72 88 22 21
= = —cm
28 28 7 Fig. 5.40
25. (b) MnL L M L 3L
8— x —+ x
Rectangular beam section D. 2 2E1 2
max
— 1.5 M0L2 3M0L2 5 M0 42 M0 x 20
'ray 2E1 4E1 • 4 El El
88 P. IAS & IFS (Objective & Conventional) Previous Solved Questions MADE EASY

29. (c) 35. (a)


M E R = 10 mm, b = 60 mm, t = 10 mm
I R E = 2 x 105 N/mm2
107 2 x 105 a E
_20m R 2 x 104 mm — -
106 R Y R

30. (b) 2 =5mm


Beam of circular section y=
'c max 4 2x105 x5
_ =100 MPa
Tav 3 10000
31. (b) 36. (c)
I-section beam, Tmax occurs at centre of the web. 24 x 3+6 x 5 102
Y= = 3.4
32. (a) 30 28
Ai .7c x 100, solid
37. (d)
A2 = n(100 — 82) = 367c (hollow) M1 = 4 kNm
1x16
IV12 — 8 = 2 kNm

Same area, same depth


as M
02
= 0.5
al
38. (d)

Fig. 5.41 200 x 6002


ZA - =12x106 mm3
6
1007cx17.32+36nx0 x2
Y- 10.07 800 x 602
172n ZB = 48 x 104 mm3
y1 = 17.32 — 10.07 + 10 = 17.25 mm 6
y2 = 10.07 + 10 = 20.07 ZA
20.07 ZB = 25
x 100 =116 MPa
6 "p 17.25
39. (d)
33. (a)
Beam of uniform strength — in which extreme fibre 200 MPa

stress are same at all cross-section along the


length of the beam. I
200 mm
34. (c)
For cross N A

50 mm

500 MPa T
Fig. 5.44

Depth of neutral axis from top.

•• • •
Shear stress distribution
Fig. 5.42
06
CHAPTER
Deflection of Beams

Q.6.1 A simply supported beam of length L carries a concentrated load Wat a distance 'a' from one end
and 'b' from the other end (a > b). Find the position and magnitudes of the maximum deflection

L
and show that the position is always within — approx from the center of the beam.
13
[CSE-Mains, 2003, ME : 30 Marks]

Solution:
Wa = RB X L
Wa x
Reaction, RB = L

Wa Wb Wb
Reaction, RA = W RA = y b R _ Wa
L L B
a>b
Take a section x-x at a distance x from A, in portion CB
Fig. 6.1
2y
El = RAx /- W(x - a)
dx 2
Wbx
L W(x - a)

,r dy 2 / W(x-a)2
Li — = Wbx ±ci
dx 2L 2
Wbx 3
Ely = + Ci x +C2 / W (x-a)3
6L 6
y = 0 at x = 0, therefore constant, C2 = 0 (second term not used)

,
Moreover, y = 0, x = L (second term -- v - a)3also used)
6

WbL3 W
0- - (L - a)3 + C1 x L
6L 6

WL2
b
0= (L-a)3 + Ci L
6 6

0= bL2 --
W x b3 + Ci L
6 6
90 ► lAS & IFS (Objective & Conventional) Previous Solved Questions MADE EASY

WbL Wb3 Wb r h2 ,21


Constant,
6 ± 6L = 6L

= 6b [b2 - a2 - b 2 -2ab]= Wb
— [-a 2 - 2a b]
6L 6L
Wab
_ [a + 2b]
6L
dy Wb x2 W 2 Wa b
El = (x a) (a + 2b)
dx 2L 2 6L
dy
ymax will occur, where — = 0
dx

n = Wbx2 W ( 2 Wab
x2 -zax) —6L (a + 2b)
2L 2 k +a

b 2 2 ab
= L ( + a 2 -2ax)-- (a + 2b)
3L

0 = 3bx2 -3L(x2 + a 2 -2ax)-ab(a+2b)

0 = 3bx2 - 3Lx2 -3La 2 + 6axL-a2b-2ab2

0 = -3x2 (L -b)-3La2 +6axL-a 2b-2ab 2


or 0 = -3x2 -3La+6xL-ab-2b 2

3x 2 -6xL+3La+ ab+2b 2 = 0
3x2 -6xL+3a(a + b)+ab+2b 2 = 0

3x2 - 6xL+3a2 +3ab+ ab+2b 2 = 0


3x2 -6xL+3a 2 +4ab+2b 2 = 0

6L + \1(602 - 4 x 3(3a2 +4ab+2b 2 )


x=
6

6L + 6 1L2 --
1(3a2 +4ab+2b2 )
x= 1, 3
6

L±/ L2 -(3a2 +4ab+2b 2 )


3

, 2
X = L±,\IL2 - - [a- ( + 2ab + b2 )]
3

, IL2 2L2 + a 2
x L
3

i3L2 - 2L2 - a 2
X = L+ \
3

L2 - a 2

\1 3
Strength of Materials Deflection of Beams 91

, 2ab + b 2
x= L±
3
Let us take a= 0.6L, b = 0.4L

.12x.6x0.4L2 +(0.4) L
x= L±
3
= L±LV0.16+0.0533
= L±LV0.21333
x = L±Lx 0.4619
x = -L x 0.4619
= 0.538 L= 0.5 L+ 0.038 L

x = 0.5L + 0.038L -_-_ — from the center


26 '
L L
-_) _ L , total
26 26 ) 13
II 2 _ a2
or x= I-
3

Q.6.2 A cantilever beam of length L, is subjected to two concentrated loads of 2P and Pat its mid length

L
and free end respectively. If the deflection at its free end is limited to — . What should be the
500
value of P? Take flexural rigidity of the beam as El.

[CSE-Mains, 2011, ME : 15 Marks]

Solution:
Flexural rigidity = El
Deflection at free end,

PL3 2P ( 13 2P ( L
5 - —+- x - + x x
3E/ 3E/ 2 2E/ U) 2
Fig. 6.2
PL3 PL3 P L3
+ + x-
3E112E1 El 8

PL3 [1 1 + 1] PL3 r8+ 2 +3]


= +
EI 3 12 8 3E1 1_ 24

13 PL3 L
x— =
24 E/ 500
El 24 1 24E1
P= X X = value of P
L2 500 13 6500 L2

Q.6.3 A cantilever 3 m long, and of symmetrical section 250 mm deep carries a uniformly distributed
load of 30 kN/m run throughout together with a point of 80 kN at a section 1.2 m from fixed end.
Find the deflection at the free end. E = 200 GPa, I = 54000 cm4
[CSE-Mains, 2012, ME : 15 Marks]
92 P IAS & IFS (Objective & Conventional) Previous Solved Questions MADE EASY

Solution: 80 kN = W
Fig. 6.3 shows a cantilever,
w = 30 kN/m
AB fixed at end B, A
Free at end A carrying a point load W at L1 from B end UDL of w
L
throughout to length
Fig. 6.3
= flexural rigidity
8 = deflection at free end
WO1 wL4
WL3 + x L2 +
3E/ 2E1 8E1
where, L1 = 1.2 m, L2 = 1.8 m, L = 3 m, W= 80 kN and w = 30 kN/m
EI = 200 x 106 kN/m2 x 54000 x 10-8 1114
= 108000 kNm2

80 x 1.23 80 x 1 22 30 x 34
8- + 1.8 +
3 x 108000 2 x 108000 x 8 x 108000
= 0.42666 x 10-3 + 0.96 x 10-3 + 2.8125 x 10-3
= 4.2 x 10-3 m = 4.2 mm

Q.6.4 A cantilever beam of length L and uniform flexural rigidity El is subjected to continuously distributed
externally applied moment m kg-cm per cm of length of the beam. Using the area moment method
show that deflection of the free end of the beam is 8 = mL3/3 El and explain why this is the same
as that obtained for the case of a concentrated force P = is applied at the end of the beam.
[CSE-Mains, 1991, ME : 20 Marks]

Solution: m kg-nn/m

L mL2
Area of BM diagram = mL x
= 2 A

2L
Ditance of CG of BMD from A =

2L mL3
First moment of BM area from A = mL2 X =
2 3 3

mL3 BMD
Deflection at the free end =
3E1 Fig. 6.4
(since deflection at fixed end is zero)
Because in BM diagrams = mL = PL
m= P
Therefore same deflection.
1. x'
Q.6.5 Use Macaulay's method or area moment method to show
that the deflection of a simply supported beam at point B
with an off centre load at point A as shown in fig. 6.5 is R L, *1. L2 R2
given by L
pL2 x (L2 _ x2 _ L2 Fig. 6.5
2
YB =
6 EL
[CSE-Mains, 1989, ME : 30 Marks]
Strength of Materials1 Deflection of Beams 93

Solution:
PL2
Reactions, R1 = L

PLi
R2 = L
Consider a section yy at a distance x' from point 1
Bending moment, Mx' = P(x - Li )
d y PL2
EI = P(x' - L ) - x' P(x' - Li )
dx 2 1 L

dy PL2 'x 2 P 2
= x—-(x' - L1) + C1
dx L 2 2
PL2 x' 3 P 3
Ely = — x- --6(x'-Li) + + C2
L 6
C2 = O,x = 0, y = 0
x' = L, y = 0

PL2 2 PL2 L2
El x0 = P (1_32 )+4
L xL P (1--1-1)3±C11- - 6 L

P PL2L
C1 =
6xL 6

IL2 x px PL21-1,c,
So, Ely = x'3 (x' 1-1)3 + 16
6 6 L 6
Deflection at B, (at distance x) x < x'

ElyB =
PL
2 x3 al
P
x-- omitted term+ x —
6
6 L 6 L

PL2 x 3 PL32 x PL2Lx Px1-2r 2


Lx +L22 -L2 ]
6L 6L 6 6L

PL 2 x 112 x 2 _ L21
Y 2
s 6 EI L L

Q.6.6 A beam ABCD, 5 m long is supported at A and C as


H x
shown in figure 6.6. It carries a point load of 2 kN at
2 kN-m x J, 2kN
end D, and a moment of 2 kNm (CW) at B. What is the
flexural rigidity of the beam (El), if the deflection at D
:1410 B/
x
is not to exceed 1 mm.
[CSE-Mains, 2009, ME : 20 Marks] 1•-- 2 m 2 m--4-1 m-R
RA = 1kN Rc = + 3kN
Solution:
Reaction Fig. 6.6
Taking moment about A
5 x 2 + 2 = 4Rc
Rc = 3 kNT
RA = 2- 3 = -1 kNL
94 0. lAS & IFS (Objective & Conventional) Previous Solved Questions MADE EASY

Take a section x-x at a distance x from A, in portion CD (Fig. 6.6)


d 2y
El — = -1x x + 2(x - 2)° + 3(x - 4))
dx 2
x3 / nN2 / A
Integrating two times, Ely= +

y = 0, x = 0, C2 = 0, constant
x = 4 m, support C, y = 0

64
0 = -- +4+0+4C1
6
0 = -10.6667 + 4 + 4C1
C1 = 1.666
3 2 3
Ely = + (X — 2) -I- (x — 4) +1.666x
6 2
At end D, x = 5 m, Y = YD
125 1
E/YD = - — + 9 + - + 1.666 x 5
6 2
= -20.833 + 9.5 + 8.33 = -3.003
3.003
YD = El m

3.003 = 0.001 m
EI
3.003
El = 3003 kNm2 (Flexural rigidity)
0.001

Q.6.7 A cantilever of length L, is loaded with uniformly increasing load, starting from zero at the free
end and to a maximum of Wo at the fixed end. The free end is propped to the level of the fixed end.
Determine the reaction at the prop and equation of elastic curve along with the slope at the
propped end. EI is assumed constant.
[CSE-Mains, 2006, ME : 30 Marks]
Solution:
Say prop reaction = R
Rate of loading at a distance,

Wx = wo-
L
x x
Load uptox, = wo- X — wo
L 2 w
x
( x2 x)
BM at section x-x = WO — x -
2L 3
Fig. 6.7
X3
BM at section x-x = Rx - wo -E

d2y x3
EI = Rx -wo -6-E
(12
Strength: Of Materials Deflection of Beams 41 95

x 2 ,.,
vv ox4
El - Rx +c
dx - 2 24L 1
dy
0, x = L
dx
RL2 woL3
0=
2 24 +1
woL3 RL2
Cl
24 2 -
Rx 3 wox 5 ( w oL3 RL21 r
Ely =
6 120E + 24 2 )x + -2
x = 0, y = 0, C2 = 0
Rx 3 wox 5 ( w oL3 RL2 )
So, Ely =
6 120L + 24 2 )x
x = L, y = 0

0=
RL3 woo woo RL3
6 120 + 24 2
_RL3 4.woL4
0 =
3 30

R= wol-
10

Ely = w°1-x3 w°x5 1w°1-3 w°L x L2 x


60 120L + 24 20

woi-x3 wox5 wo0


60 120L 120
Slope at free end
x= 0
woL3 w01-3 w0L-3
EliA - Cl = 24 20 120
woL3
Slope at A, iA =
120E1 16 kN
0.3 m 10 kN
Q.6.8 A simply supported beam is subjected to the
.._ 20 kN/m

loading as shown in the figure 6.8(a). Calculate


the deflection at a section 2 m from end A. 101(N 11i1:3
Assume E= 70 GN/m2, I= 830 cm4) 141---- 1.5 m --144-- 0.9 m -414-0.6 m -44
[CSE-Mains, 1996, ME : 30 Marks]
3 kNm 20 kN/m 16
Solution: A AB
Reaction D x
3xRB + 3 = 16 x 2.4 + 20 x 0.9 x(1.5 + 0.45)
3 + 3/38 = 38.4 + 18 x 1.95 =38.4 +35.1 t
4-- 1.5 m
RA = 10.5 kN
'14 0.9.m --4-}4- 0.6 m

RB = 23.5 kN T
Fig. 6.8
Total load = 20 x 0.9 + 16 = 34 kN
9 6 0. lAS & IFS (Objective & Conventional) Previous Solved Questions MADE EASY

Reaction, RA = 34 - 23.5 = 10.5 kN


Figure shows the simplified loading diagram UDL extended to portion is portion DB upto x and negative
UDL applied beyond D.

10.5x - 3(x -1.5)° - 2 (x - 1.5)2 +14-(x


/ - 2.4)2 -16(x - 2.4)
2
w = 20 kN/m

d2y
El _ 10.5x - 3(x - 1.5)° -10(x - 1.5)2 + 10(x - 2.4)2 - 16(x - 2.4)
dx 2

dy 10 1 .5)3 +
EI 5.25x2 - 3(x -1.5)- (x _ (x - 2.4)3 - 8(x - 2.4)2 +
dx 3 3

10
— (x -1.5)4 + 10 (x 2.4)4 8 (x 2.4.)3
Ely = 1.75x3 -1.5(x -1.5)2 CiX + C2
12 12 3
x = 0, y = 0, C2 = 0, Omitting II, Ill, IV and V terms
x = 3 m, y = 0

10 4 10 \ 4 8 3
0 = 1.75 x 27 - 1.5 x 2.25 - (1.5) + (0.6) - -(0.6) + 3C1
2 2 3

0 = 47.25 - 3.375 - 4.21875 + 0.108 - 0.576 + 3C1


= 39.188 + 3C1
C1 = -13.063

+ 10
2 (x 1 4
Ely = 1.75 -1.5(x -1.5)2 5) ;(x - 2.4)4 - 3 (X - 2.4)3 - 1 3.063x
1
x = 2m

Sy2 . 1.75 x; - 1.5 x 0.25 + ;(0.5)4 - Omitted - Omitted + 13.063 x 2

= 2.333 - 0.375 - 0.05208 - 26.126 = -24.22


y2EI = 24.22

= 70 x 106 kN/m2 x 830 x 10-8 kNm 2 = 581 kNm2

24.22
Y2 = = 0.04168 m = -41.08 mm
581

Q.6.9 A simply supported beam carries a uniformly varying load with zero intensity at left support and
an intensity of w at the right support. Calculate the maximum deflection and maximum slope and
mention the position on the beam when these occur.
[CSE-Mains, 1988, ME : 30 Marks]

Solution:
Figure 6.9 shows a beam AB of span length L, carrying uniformly varying load with maximum intensity
w/m at right hand B.
wL
Total load on beam = 2
Strength of Materials Deflection of Beams 41 97

wL 2L
Moment about A, -T x 3 = RB x L

wL
Reaction, R8 =
3

wL wL wL
Reaction,
RA = 2 — 3 6 wL wL
RA = 6 F2,-= 3
Consider a section xx at a distance x from A
Fig. 6.9
wx
= rate of loading = L

wx x = wx2
Total load upto x= x
L 2 2L

CG of load ties at — from x-x


3 x
2
Wx x
Bending moment, = RAx x
2L 3
wL Wx 3
-x x
6 6L

wLx2 wx4
or ci — = + C1 (constant of integration)
dx 12 24L

wLx3 wx 5
Ely = 36 120L + Ci x+ C2 (constant of integration)

y= 0, x = 0, C2 = 0
y= 0 at x = L

wLx 3 wx 5 r ,
0= + + C2
36 120E
0 — 3) 4
— (1 wL A- CiL = 7 x wL4 +Ci L
360 360

3
Content, wL
C1 360 x

r.,dy wLx 2 wx 4 7 o
", — = w
dx 12 24L 360
dy
Maximum deflection occurs, where — = 0
dx
WL 2
,,
vv X
4 7
0 = 12 24L 360 w1-3
4 3)
W y2 x
0
= 12 2L — 30 "'
4 7
or LX2 -
X =0
2L 30
98 0. IAS & IFS (Objective & Conventional) Previous Solved Questions MADE

4 7
4
I-2X2 - X - =0
2 30
30L2x2 -15x4 - 7L4 = 0
15x4 - 30L2x2 + 7L4 = 0

2 + V900L4 - 420L4
x2 - 30L
30

2= 300 ± NFINTIL71 30L2 ± L2 x 4.156


x
30 30
4 r-
x2 301: 2 - , x- N/30
30
= L2 - 0.730L2 = 0.27L2
x = 0.5196L
ymax at x = 0.5196L

wLx3 7 ,3
= WL x +1.1x
EIy 36 360
wL 3 5 7 3
syma, -k(0.5196L) x (0.51960 - — wL x 0.5196L
-120L 360
= wL4 [0.003897 - 10003156 -0.0101] (Putting the value of C1)
= 6.522 x 10-3 wL4

6.522 x10-3wL4
Ymax = at x = 0.5196 from A
EI

7wL3 1
Slope at A 360 xt aend A
EI
Slope at B
wL3 wL3 7
EBB = wL3
12 - 24 - 360 x
(30 -15 - 7)
wL3 = +
360 360 wL3

wL3
ja = 45E1 , (Hence, maximum slope at B)

Q.6.10 Draw SF and BM diagram of the beam ACDB, where two couples are acting. Find the ratio of
deflection at location C and D where couples are acting.
[CSE-Mains, 2002, ME : 30 Marks]

Solution:
Reaction:
Moment about A, 10 + 20 = 3RB
RB = 10 kN
RA = -10 kN
Strength of Materials Deflection of Beams 4 99

= 10 kNm M2 = 20 kNm

-10 kNm -10 kNm


(c)
BM diagram

Fig. 6.10
SF diagram
A vertically, SF is -10 kN constant
BM diagram
MA =
Mc = -10 x 1 = -10 kNm (Hogging) (Just left of C)
Mc = -10 + 10 = 0 kNm (Just right of C)
MD = -10 x 2 + 10 = -10 kNm (Hogging) (Just left of D)
Mc = -10 + 20 = +10 kNm (Sagging) (Just right of D)
Me =
Deflection
Taking a section in the last portion DB of the beam
Mx = -10x + 10(x -1)° + 20(x - 2)°

d2
El Y = -10x + 10(x-1)0 + 20 (x - 2)°
dx 2

dy
EI— -5x2 +10(x-1)+20(x-2)+C1
dx

5x 3
Ely = - +5(x-1)2 +10(x-2)2 + Cpc + C2
3
y = 0, x = 0, constant, C2 = 0
y= 0, x = 3

3
= — — x3 +5x4+10x1+3Ci
3

0 = -45 + 20 +10 + 3C1


C1 = +5
100 10- lAS & IFS (Objective & Conventional) Previous Solved Questions MADE EASY

Ely = -5c+ 5(x -1)2 +10(x-2)2 +5x

5 10
Elyc = 0 + Omitted+ 5 x 1= 3 = +3.33

- 3 x 8+ 5(1)+100)+5 x 2

= -13.33+5+0+ 10 = +1.667

Yc 3.33
=2
Yo 1.667

Q.6.11 A beam of flexural rigidity 20 MN m2 is simply supported over a span of 6 m as shown in figure
6.11. It carries a concentrated load of 24 kN, 2m from the left hand support and a UDL of 6 kN/m
on central 2 m part. The beam has vertical member welded onto it 2 m from right hand support
which carry two horizontal loads of 24 kN as shown in the figure. Distance between these loads
is 1 m.

24 kN
24 kN
6 kN/m m D
WyTheThrY\
C
I18 kN
(a) 24 kN
2m ► 1.1 2m '14 2m

(b)

Fig. 6.11
(i) Draw BM diagram of the beam.
(ii) Calculate the vertical deflection at the central points of the beam.

Solution:
Reaction:
Moment about A
24 x 2 + 24 + 6 x 2 x 3 = 6130
48 + 24 + 36 = 6R0
Reaction, 130 = 18 kN
Reaction, RA = 24 + 12 - 18 = 18 kN
BM diagram
MA = °
MB = 18 x 2 = 36 kNm
M3 = 18 x 3 - 24 x 1- 6 x 1 x 0.5 = 54 - 24 - 3 = 27 kNm
Just left to C, M4 = 18 x 4 - 24 x 2 - 6 x 2 x 1 = 72 - 48- 12 = 12 kNm
Strength of Materials Deflection of Beams 4 101

M4' = 12 + 24 = 36 kNm (Just right to C)


MD = °

36 kNm 36 kNm
27

A B
BMD

Fig. 6.12
Deflection at central point
Extend UDL to portion CD and apply -w from C to Das shown.

MC = 18x - 24(x - 2)- --t4 (x - 2)2 + 1.;1-(x - 4)2 + 24(x - 4)°


2
W = 6 kN/m

d y
El x2 = 18x - 24(x - 2)- 3(x- 2)2 + 3(x - 4)2 + 24 (x - 4)°
d

dy
EI — = 9x2 - 12(x - 2)2 -(x- 2)2 + (x - 4)3 + 24(x - 4) +
dx

3 (x - 2) 4 (x-44)4
Ely = 3x3 - 4(x -2) + +12(x -4)2 +Cix+ C2
4
x = 0, y = 0 Constant C2 = 0
y= Oatx= 6 m

0 = 3 x 63 - 4(4)3 --•-• -+-


24 +12x22 +6Ci
4 4
0 = 648 - 256 - 64 + 4 + 48 + 6C1
0 = 380 + 6C1
Constant, C1 = -63.33

3 (x - 2) 4 (x - 4) 4
Ely = 3x3 - 4(x -2) + +12(x -4)2 - 63.33x
4 4
4
0y3 = 3 x 33 - 4 - + omitted + omitted term - 63.33x
4
= 81-4-O.25-190=-113.25
81 -4-
El = 20000 kNm2

113.25
= 5.66x10-3 m= - 5.66 mm
Y3 - 20000

Q.6.12 A cantilever beam AC of span '3a' and carrying a concentrated load 'P at distance '2a' from the
fixed end A is strengthened by supporting its free end on the free end of another cantilever BC as
shown in the figure 6.13. Determine percentage reduction in the moment at the support A.
10 2 IP. IAS & IFS (Objective & Conventional) Previous Solved Questions MADE EASY

IC
AI

2a —.4.-- 2a --id
Fig. 6.13

[CSE-Mains, 2002, CE : 20 Marks]

Solution:
MA = -2Pa (without strengthening)
Say reaction at C = RT
Take a section at a distances of x from C

d2y
2c1— = R xx- P(x- a)
dx 2

dy Rx 2 P(x -
2E1— = +Ci
dx 2 2 Fig, 6.14

dy
= Oatx= 3a
dx

R9a2 P
0= (4a2)+Ci
Ci 2 2
Constant, = 2Pa2 - 4.5Ra2

dy 2
2E1— = Rx P (x -a)2 .1- 2Pa2 - 4.5a2R
dx 2 2

Rx3 P
2Ely = — --(x - a)3 +2Pa 2 x - 4.5Ra2x + C2
6 6
y= Oatx= 3a
3 3 3
0 = 4.5Ra -- Pa +6Pa -13.5Re + C2
6

3 14 3
C2 = 9Ra - 3Pa

x3 P 3 14
2
2Ely = --(x - a) 3 + 2Pa x — 4.5Ra2x +9Ra - — Pa
6 6 3
But y= 0 at x = 0, from C

14
or = 9Ra3 - -5-Pa- , Omitting Ilnd term

14P
or R=
27
Strength of Materials Deflection of Beams 1 03

14 14 4
Bending moment, - —
MA = 27 x P x 3a - 2Pa = + 9 Pa - 2Pa = -- Pa

4 Pa
--
Reduction in BM = 12Pal H--Pa
9 149
14 1 7
% reduction = — Pax — x 100 = - x100 = 77.78%
9 2Pa 9
0.6.13 Compare the elastic deformations at the points of loading produced in a simply supported beam
of length L., carrying a concentrated load Pet its mid-point to that of a cantilever of the length L,
carrying a load P at its free end. The moment of inertia of the section of the simply supported
beam is J in the left half and 0.5 J in the right half, For the cantilever, the moment of inertia of the
section Is 2J upto a distance of 112 from the fixed end and J for the remaining portion. Assume
that both the beams are of the same material.
[CSE-Mains, 1990, ME : 20 Marks]
Solution:

BMD
Fig, 0,15 Fig. 6.16

d2y P P L.) Px _2P( L


For S,S, Beam E =
dx2 2J 0.5J = 2J J
2 p( 2
dy
c— = 4j - -j- x - +C
dx

Ey = 3 213 Cix + C2
P2J 3J k
Y= 0, x = 0, C2 = 0 (Omitting lind term)
Y= 0, X = L
P L3 P L3 r, PL3 PL
3 r,
0= X — - — X + = — - — + iL
12 J 3J 8 12J 24J
PL3 1 _
• =
- 24J x — 24J

Px 3 P L)3 PL2
Ey =
- 24J
104 10. IAS & IFS (Objective & Conventional) Previous Solved Questions

Deflection at centre (x= L/2)


PL3 PL3 PL3
EYc 96J 48J = 96J

PL3
Y0 96EJ
For Cantilever Beam
d2 y Px 2dx i L Px2dx
E — xdx
dx 2 JI-I2 2J
px3 L/2 px3 L
d - yi B -
E[xJ
dx A 3J o 6J
L/2

PL3 PL3 PL3


E[(L.iB - y B L) - (0 x i A - y + , where iB= 0, YB = 0
24J 6J 48J

2 -4 3 PL3
+Ey A =
:J+ l a = x J

3 PL3
YA = x
16 EJ

Q.6.14 For the beam shown in the figure 6.17 compute


the deflection of end B by force method.
Fixed
[CSE-Mains, 2013, CE : 15 Marks] end
Solution:
Consider a section at a distance x from A
wx 2 w x2 L
M = bending moment = Rx- - Rx
2 2L
where W= wL, total load, Fig. 6.17
d2y W 2
El _ Rx-- x
dx 2 2L

dy = Rx 2 Wx 3 r ,
El —
dx 2 6L
dy =
0 at x = L
dx
RL2 WL3 c
0=
2 6L 1
WL2 RL2
Constant, C1 -
6 2
Rx2 Wx3 RL2
dy
El — =
dx 2 6L 6 2

Rx3 Wx4 WL2 RL2 ,-,


EIy = x 2 x+u2
6 24L + 6
Y= 0, x = L
Strength of Materials Deflection of Beams 10 5

RL3 WL3 WL3 RL3


0= +L.2
6 24 6 2

RL3 WL3
+ + C2
3 8
_ WL3 RL3
Constant, C2
8 3
Rx3 Wx 4 WL2 RL2 WL3 RL3
So, EIy + x
6 24L 6 2 8 3
At end A, y = —8, x = 0
WL3 RL3
2(-5) +
8 3
R = k8
WL3 k8L3
—E18 =
8 4- 3

k WL3
—81E/ + =—
3 8

WL3
8=
81E/ + 1(12
3

Objective Questions
Q.1 The ratio of area under the bending moment M/ 2
diagram to the flexural rigidity between two (a) (b)
2E/ EI
points along a beam gives the change in
(c) 3M/2 (d) 2M/2
(a) deflection
(b) slope 2E1 El
[CSE-Prelims, CE : 2003]
(c) shear force
(d) bending moment Q.3 In a simply supported beam AB of span L, the
[CSE-Prelims, ME : 1998] mid point is C. In case-1, the beam is loaded
by a concentrated load W. In case-2, the beam
Q.2 For cantilever beam shown in the given figure
is subjected to a UDL of intensity w such the
6.18, the deflection at C due to a couple M
wL = W. The ratio of central deflection in case 1
applied at B is equal to
to that in case-2 is
5
M (a) — (b) 3
3
5
A (c) — (d) 8
8
[CSE-Prelims, CE : 2003]
El = Constant
Q.4 A simply supported beam AB of span L is
Fig. 6.18 subjected to a concentrated load W at the
centre C of the span. According to Mohr's
10 6 0. IAS & IFS (Objective & Conventional) Previous Solved Questions MADE EASY

moment area method, which of the following 12EIA 6EIA


gives the deflection under the load? (a) (b) 3
/3
(a) Moment of the area of M/E1 diagram
4E1A 3E/A
between A and C taken about C (C) 3 (d) 3
(b) Moment of the area of M1E1 diagram /
between A and B taken about B [CSE-Prelims, CE : 2006]
(c) Moment of the area of M/E1 diagram Q.7 A propped cantilever is acted upon by a
between A and B taken about A moment M0 at the propped end. What is the
(d) Moment of the area of M1E1 diagram prop reaction?
between A and C taken about A
[CSE-Prelims, CE : 2005]

Q.5 A cantilever has rectangular cross-section and


supports concentrated load at its free end
initially. If depth and width of the beam section
are doubled, the deflection at free end of the
cantilever will reduce to what percentage of the 0.5M0 (b) Mo
initial deflection? (a) L
(a) 20% (b) 15.72%
(c) 1.5M0 (d) 2.0Mo
(c) 9.57% (d) 6.25%
L
[CSE-Prelims, CE : 2005]
[CSE-Prelims, CE : 2007]
Q.6 Due to settlement of support at A of propped
Q.8 A simply supported beam AB of span L carries
cantilever shown in the figure 6.19 given below,
a concentrated load Wat the centre of the beam
what is the vertical reaction at B?
with constant El throughout. What does the
B moment of the area of the M/Eldiagram between
A and C, taken about A represent?
1 [CSE-Prelims, CE : 2010]
El = Constant Answers
Fig. 6.19
1. (b) 2. (c) 3. (d) 4. (d) 5. (d)
6. (d) 7. (c) 8. Deflection

Explanations
1. (b) 3. (d)

El = Mdx _ WL3
dx xi Case-1:
Yc •48E1
f" Mdx _ 5 wL4
(L2' — L1') = = change of slope Case-2:
xl El Yc 3• 84 E/
wL = W
2. (c)
, 5 WL3
- + ML
xL Yc —•384 x EI
YOc 2E1 El
Yc 8
3 ML2 Yc' — 5
— 2 E/
Strength of Materials Deflection of Beams 107

4. (d) 7. (c)
Mo
Moment of area of diagram between A and C
taken about A. x --t)

Fig. 6.23
L
2 2 y
2
EI = Px + Mo
dx

dy Px 2
El — —+Mox+Ci
dx 2

a
y = 0 at x = L
dx
Fig. 6.21
PL2 A A r,
0 = — ivio...+L.,1
1 WL L(L) WL3 2
Moment about C= - — x- - =
2 4 2 6 96E/
PL2
1 WL L( L) WL3 C1 = ---MoL
Moment about A= x— x- - = 2
2 4 2 3 48
dy Px 2 A A PC AA I

5. (d) El — = + ivi o x ivi oL


dx 2 2
WL3
P63 2 p 2
L.
81 = 3E1
ElY = T x-M°Lx+C2
I, = (2b)(2d)3 16bd3 y= 0,x=0, C2 =0
= 16/
12 12 = y = 0 at x = L
82 = 81 = 6.25% P L3 M L2 P L3
0= Moe
6 2 2 -
6. (d)
PL3 M01-2
3 2
Mo
P = 1.5—

Fig. 6.22
8. (Deflection)
RBL3 Deflection at the centre C of the beam = moment
A-
3E/
3EIA of area of — between A and C taken about A.
RB = E/
M
07
CHAPTER
Torsion

Q.7.1 Design two solid circular shafts to transmit 200 HP each without exceeding a shear stress of
70 MPa at 20 rpm and other at 20,000 rpm. Give your inference about the final results from the
view point of economy. Do you have any other suggestion to improve the economy further?
[CSE-Mains, 2013, CE: 10 Marks]

Solution:
HP = 200
Shaft-1
RPM = 20
2.TE x 20
= 2.0944 rad/sec
60
200 x746
Torque, Ti _ 2.094 = 71251.194 Nm = 71251.194 x 103 Nmm

ic 3 it
71251194 = — xdi X T = — xdi3 x70
16 16
d13 = 5183990.724 mm3
d1 = 172.98 mm, shaft diameter
Shaft-2
RPM = 20,000
27c x 20000
= 2094.4 rad/sec
60
200 x 746
Torque, T2 - 2094.4 = 71.251 Nm = 71.25 x 103 Nmm

71251 = 16 cll x
16
4
X 70, 4 = 5183.0
d2 = 17.3 mm, shaft diameter
Shaft 1 at 20 rpm
Shaft diameter is too large. There is hardly any machine which runs at 20 rpm for power transmission.
Shaft 2 at 20,000 rpm
Shaft diameter is reduced to one tenth from 173 mm to 17.3 mm. There is saving in material by 99%. But
at high speed of rotation, large centrifugal forces are developed if there is any eccentric mass on shaft,
which will produce large deflection and slope in the shaft, rigidity is drastically reduced. Even to instal
any gear, pulley for power transmission, keyways cannot be made on a shaft of 17.3 mm.
Strength of Materials Torsion 1 109

For ordinary power transmission in simple machine speeds are 200-500 rpm, say at 400 rpm, shaft
diameter will be approx. 47 mm. To install any gear, pulley on the shaft, keyways can be easily cut.

Q.7.2 Find the diameter of a solid cylindrical shaft subjected to 100 rpm and transmitting 350 kW power,
when is the shear stress not to exceed 90 N/mm2.
What percent saving in weight would be obtained if this shaft is replaced by a hollow one, whose
internal diameter equals to 0.65 of the external diameter, the length, the material and the maximum
shear stress being the same.
[IFS 2012, CE : 15 Marks]

Solution:
Power = 350 kW = 350 x 1000 Nm/s
N, speed = 100 rpm
2rc x 100
Cylinder velocity, = 10.472 rad/sec
- 60
350 x 1000
Torque transmitted, T 10.472 = 33422.54 Nm = 33422.54 x 103 Nm

ti = 90 N/mm2, maximum shear stress


3
T= d
16

33.422 x 106 = X d 3 x 90
16
d3 = 1.891298 x 106 mm3
d = 123.66 mm, shaft diameter
(b) For same power transmission, same shear stress, polar modulus should be the same

rcd3
Zps for solid -
16

4 2
[ 4
Zph, for hollow = — D -(0.650 ]- —[0.82149]E)3
32 D 16
d3
D3 - =1,2173d3
0.82149
D = 1.06774 d
Inter dia. = 0.694 d
Saving in material depends on area of cross-section of solid and hollow shaft
itd 2
Area of solid shaft, As -
4

Area of hollow shaft, Ah = [(1.06774d)2 - (0.694d)2 ] = Ed2 [1.14 - 0.4816]


4 4

Ed2 x0.658
4
Ah
AS = 0.658
Weight of hollow shaft is 0.658 x weight of solid shaft
Saving in material = 34.2%.
110 10- IAS & IFS (Objective & Conventional) Previous Solved Questions MADE EASY

Q.7.3 Compare the weights of equal lengths of hollow and solid shaft to transmit a given torque for the
same maximum shear stress if the inside diameter is 2/3 of the outside diameter.

[IFS 2012, ME : 10 Marks]

Solution:
Say, d = Diameter of solid shaft
D = External diameter of hollow shaft

2D
= Internal diameter of hollow shaft
3
T = torque
= Maximum shear stress
Solid shaft
16T
= 3 ...(1)
70
Hollow shaft
32T 32T 81
= x
0.5D (D4 16 D.4) 704 65
n
81 )
16T 81
= x ...(ii)
TCD3 65
From eq. (i) and eq. (ii)
16T 16T 81
x
7cd3 tD3 65
381
u x
65
D = 1.0761 d ...(iii)
Since length is same, material is the same (not given in problem), weight of shaft depends on area of
cross-section.

Ah = .0761d)2 -(; x 1.0761d)2 ]

= E[1.1580 - 0.51466]c/2 = -7-c10.643341d2


4 4
Ah W
= = 0.64334
AS Ws
Weight of hollow shaft is only 64.334% weight of solid shaft.

0.7.4 A stepped shaft ABC, is 0.8 m long. For a length AB = 0.4 m, shaft diameter is 40 mm and the
length BC = 0.4 m, shaft diameter is 20 mm. Shaft is fixed at both the ends A and C. At the section
B, a torque T is applied which causes a maximum shear stress of 100 MPa in stepped shaft.
Determine the magnitude of torque T.
Strength of Materials Torsion 41 111

Fixed
A

Fig. 7.1
[CSE-Mains, 2001, ME : 20 Marks]
Solution:
Torque Tapplied at section B is shared by shaft AB and BC, as shown
± T2 = T ...(i)
eAB = eBc (since eAc = 0)
T1 x0.4 T2 x0.4
GJ1 GJ2
J1 = 16 J2 as di = 40 mm, d2 = 20 mm (d1 = 2d2 )
T1 T2
16J2 J2
= 16T2
So, T1 = 0.94117 T
T2 = 0.05883 T
Assume that max. shear stress occurs in AB

3 —TC X 100 x 403 = 1256637 Nmm = 0.94117 T


T = — T x 40
1 16 = 16
Torque, T = 1335186 Nmm = 1.335 kNm

Then, T2 = 78540.35 Nmm = 16 x r' x 20 3


= 50 N/mm2 in portion BC. (Less than permissible hence, OK)
Hence, T = 1.335 kNm

Q.7.5 A shaft of 12 cm external diameter and 8 cm internal diameter is subjected to a bending moment
of 300 kgf-m, twisting moment of 100 kgf-m and a direct thrust of 10,000 kgf. Determine the
maximum principal stress and diameter in which it acts with reference to the axes of the shaft at
the end points P and Q of diameter PQ as shown.
[CSE-Mains, 1987, ME : 30 Marks]
Solution:
D = 12cm
d = 8 cm

Area of cross-section = 5- (D2 - d2 ) = 'It-(144 - 64) = 20 TG cm2


4 4
= 207c cm2 (Comp.)
10,000
Direct stress, ad = = 159.155 kgf/cm2
2rc
O
Bending stress (sagging moment) 6 cm
Q

(D4
(D -d 4 )=
4 -d ) -1(12 4 -84 Fig. 7.2
64 )
112 ► IAS & IFS (Objective & Conventional) Previous Solved Questions MADE EASY

= 1 -(20736 - 4096) = 260n cm4


64
My + 300 x 100 kgf-cmx 6 cm
ae = +
I - 2607c cm4
= ±220.37 kgf/cm2
Shear stress

"C = x6

J = Polar moment of inertia = 21= 520n cm4


100x100 x6 2
— = 36.73 kg/cm
520n
Point P (normal stress = 22037 + 59.155 = 379.525 N/mm2 (comp.) and shear stress = 36.73 kgf/cm2)
Principal stresses
2
379.525 379.125
/31, p2 + (36.73)2 379.525
2 2 ) + ab

= -189.76 ± J36009.8 +1349.1 36.73

= -189.76 ± 193.285
ti
= -383.04 kgf/cm2, + 3.525 kgf/cm2 36.73
Point Q (Normal stress = 220.37 - 159.155 = 61.215 kgf/cm2)(tensile) and sehar stress = 36.73 kgf/cm2)
Principal stress

61.215 \I(61.215)2+ (36.73)2


Pi, P2 = 2 ±

= 30.605 ± V936.82 +1349.1


= 30.605 ± 47.81
= +78.416 kgf/cm2, - 17.205 kgf/cm2 36.73 kgf/cm2

Q.7.6 A brass tube fits closely over a steel shaft of 100 mm diameter. Find the thickness of the brass
tube which would ensure that the torque applied to the assembly is shared equally by the two
materials. Find the maximum shear stress in each material and the angle of twist in a length of
3 m. The torque applied is equal to 20 x 104 kgf-cm. Assume modulus of rigidity of steel = 8 x 105
kgf-cm2, modulus of rigidity of brass = 4 x 105 kgf/cm2.
[CSE-Mains, 1990, ME : 20 Marks]

Solution:
GS = 2 GB
Say outer radius of bras tube = R2 cm
es e9 Brass
(Since its is parallel combination)
TSLS TB LB R

GSJS GB JB

But, Ts = TB as given
Ls = TB Fig. 7.3
Strength of Materials Torsion 4 113

GsJs = GBJE3

GO JD
LIS -= JE1 X Gs =2
or JI3 = 2Js
Js' polar moment of inertia of steel shaft

Js = EX
2
54 = 312.57c cm4

II (F? - 5 4 ) = 625 x TC
JB = 2 x 312.5 p= 625x 7C CM 4 = 2
-

R24 - 625 = 1250


R24 = 1875
R2 = 6.58 cm
Thickness of brass tube = 6.58 - 5 = 1.58 cm
Steel shaft

Torque, Ts = 10x104 kg-cm= —


It x c x d3
16

10x104 = EXTXr 3 =EXts X 53


2 2
- 10 x104 x 2
Shear stress, Ts = 509.3 kgf/cm2
Icx125
Brass shaft
TB = 10 X 104 kg-cm
4
_ 7c R -R/ x T B
2 R2

n[6.58 4 -5 41 TC [1875 -625] 625ic


10 x 104 = x TB - 2 TB = XTB
2 6.58 6.58 6.58
Maximum shear stress in brass tube
= 10 x104 x6.58
TB = 335.116 kg/cm2
625n
TsL 10x104 x300
0 = = - 0.0382 radian = 2.99°
GsJs 8x105 x 312.5n

Q.7.7 A shaft circular in section and of length L is subjected to

a variable torque given by kx 2 , where x is the distance


L
measured from one end of the shaft and k is a constant.
Find the angle of twist for the shaft by use of Castigliano's
theorem. Torsional rigidity of the shaft is GJ, see fig. 7.4.
Fig. 7.4
[CSE-Mains, 1994, ME : 20 Marks]
Solution:
Take length dx along the length
Strain energy due to twisting
114 ► IAS & IFS (Objective & Conventional) Previous Solved Questions MADE EASY,

LT2dx
u= IJO
2GJ
au = 2Tdx 1
9 f L Tdx But T= k x2
ar Jo 2a/ GJ 0 L

1 L kx2 kL3 kL2


dx -
GJ f0 L 3GJL 3GJ

Q.7.8 Obtain the angle of twist of one end relative to the other end of a tapered shaft having radii ri and
r2 at its ends and of length L. The shaft is subjected to equal and opposite torque Tat it ends.
Modulus of rigidity of shaft is G.
[CSE-Mains, 1995, ME : 30 Marks]

Solution: T
T
r1 = radius at one end
r2 = radius at other end
I
= radius at distance x (from r1 )
I
= + kx where k- ro
L
x
L( `4 dx
Polar moment of inertia, = r4 = 7 kx) 4
x 2 x 2 Torque T

2Tdx Fig. 7.5 [ Tdx]


angular fracture length, dx, dO = Here, dO =
G1C(ri + kx)4 GJ

0
L 2Tdx A = 2T L
S (ri + kx)-4 dx
° TtG(r1 + kx)- nG °
L
2T [(ri + kx) 3 2T 1
= =
TcG -3k 37ckG[ ( ri + k x\3 1
o 1 0

2T [ 1 11 2T [ 1 1 ]
= =+
ankG (ri + kL)3 r13 3TckG r13 q

2T [r? - r1
3nkG ri3r?

2T L (r2 - ri )(ri2 + ri r2 + r22 )


_ x
anG (r2 -r1) r13r2

=
2TL [ rig + rir2 + d1
anG 3 3
ri r2 ]
Q.7.9 A solid circular uniformly tapered shaft of length L with a small angle of taper is subjected to a
torque T. The diameter at the two ends of the shaft are d and 1.2d. Determine the error introduced
if the angular twist for a given length is determined on the basis of the uniform mean diameter of
the shaft.
[CSE-Mains, 1996, ME : 20 Marks]
Strength of Materials Torsion 115

Solution:
Tapered shaft 1.2d to d (di = d, d2 = 1.2d)

3271 +d1d2 d2 +1.2d2 +1.44d2 ]


0- x
3nG d1 d2 d3 x(1.2d)3

32TL [ 3.64 d
x 7.152 7±
3itG 1.728 GJ
0', on the basis of mean diameter 1.1d
32TL TL
0' = rc(1.1 A G = 6.9571GJd -

7.152 - 6.9531
% Error _ x 100 = 2.78%
7.152
Q.7.10 For the shaft loaded as shown in figure 7.6, calculate the maximum shear stress induced and the
angle of twist for cross-section at A value of modulus of rigidity is G.
M 4M C
t
2t
3d
t
O(A B
2L 0-14 2L L

Fig. 7.6

[CSE-Mains, 1994, ME : 20 Marks]

Solution:
Polar moment of inertia

nd 4 it(2d)4 7C (3d)4
J J,- = 16,11, J, = = 81J1
1 32 - 32 - 32
4M - T2

JZ
A M-
2L

Fig. 7.7

Let torque at 0 is Ti and at D is T2 as shown,


Therefore in portion AB M- T1 = 4 M- T2
T2 - T1 = 3 M
°Ao OBA + °DB = 0
x 2L (114-7-1)2L + T2 L L
=0
Ji J2 1 J2 J3
116 P. lAS & IFS (Objective & Conventional) Previous Solved Questions MADE EASY

T1 x2L (M-Ti )x2L IL L )


+T2 + = 0
J1 16,11 16J1 81J1

on- Ti)2L T2 x L T2 X L
Lx2Ti + + = 0
16 16 81
M x2L Tix2L T2L T2L
Lx2Ti - + + + = 0
16 16 16 81
17 2ML (81+16)T2L
x 2LTi 4. = 0
16 16 16x81
17 M 97T2
T1 + 0
8 8 16 x 81 =
17., 97T2 M
/1 + ...(ii)
8 16 x 81 8
97 1 8 M 8
T1+- x- x-T2 = 8 —
16 81 17 x1
M
Ti + 0.03522 T2
17
Ti + 0.03522 T2 0.0588 M
Also T2 - Ti 3M From eq. (ii)
or T2 2.9547 M
7-1 -0.0453 M
Ti x 2L = -0.0453M x 2L
e AO GJ
0.0453M x 2Lx 32 = 0.9228ML
Gxrcd 4 Gd 4
16T2 _ 16 x 2.9547M 1.881M
tin portion BC
n(2d)3 8icd3 d3

16T2 = 0.557 M
ti in portion CD d3
TC(3d)3
1.881M
Hence, tmax is in portion on BC - d3
Q.7.11 A cantilever ABC, 200 mm long and 20 mm diameter is fixed at end A as shown in the below figure
7.8. A horizontal axial load of 10 kN, a vertical load of 1 kN acts at end C. Torque T applied at
section C, produces a maximum shear stress of intensity 50 MPa in cantilever. Determine principal
stresses at point B of cantilever.

10 kN

H 100 mm 100 mm-01


Fig. 7.8
[CSE-Mains, 2001, ME : 20 Marks]
Strength of Materials Torsion 41 117

Solution:
Axial compressive force = 10,000 N

Area, A= — 7C X 202 = 100 nmm2


4
as = compressive stress, uniform
10,000
31.83 MPa
1007c
(-ve sign indicate compressive nature)
BM at B = 1 kN x 100 = 1000,00 Nmm
ird 3 IC 3
Z = = x 20 = 0.7854 x 103 mm3
32 32
1000,00
Bending stress, CS, = 127.324 MPa (at point B)
" - 0.7854 x 10°
(Tensile at upper fibre)
Net direct stress at B, (upper fibre) aB = 127.324-31.830 = 95.494 MPa
Shear stress due to torque, ti = 50 MPa

\ (GB)2
Principal stress at B, !Du p2 =
2 - 2

= 47.747 ± V47.7472 + 502


= 47.747+ V2279.8 + 2500 = 47.747 ± 69.136
pi = 116.883 MPa
1)2 = -21.389 MPa

Q.7.12 A solid circular shaft is encased in a hollow copper shaft so as to make a compound shaft. The
diameter of steel shaft is 8 cm and outside diameter of the copper shaft in 11 cm. The compound
shaft is 2 m long and is subjected to a torque of 8 kNm. Determine maximum shear stress in steel
and copper.
Gsteel = 2 Gcopper = 80 kN/rne
[CSE-Mains, 2007, ME : 20 Marks]

Solution:
Solid steel shaft
d = 8 cm
rcd 4 Tcx8 4
J - 128ncm4
S — 32 - 32
Hollow copper shaft
d= 8, D = 11 cm

JCL/ = (114 - 84 ) = (14641- 4096) = 329.53n cm4


32 32
Gs = Gcu x 2
Angle of twist will be equal in both shafts
°s = esu
TsLs T„L„
JsGs J„G„
118 0. lAS & IFS (Objective & Conventional) Previous Solved Questions

Ls = Lcu

Ts Js x Gs 128ir 2
x = 0.778
Tcu Jcu Gcu 329.1341 1

Ts = 0.778 Tcu
Also, Ts +Tcu = 8 kNm
1.778 Tcu = 8
Tc„ = 4.50 kNm
Ts = 3.50 kNm

7C 3.5 x106
Stress in solid steel shaft x , = 43.51 N/mm2
s - 16 128 x10- Tc
Maximum shear stress in solid steel shaft
55 = 329.5371 x 104
tcu
Copper shaft Ta, = JCL, X X CU
55
- 329.537c x104 Tcu
4.5 x 106
55
tai = 23.90 N/mm2, Maximum shear stress in hollow copper shaft.

Q.7.13 A mild steel shaft of 200 mm diameter is to be replaced by a hollow shaft of alloy steel for which
the allowable shear stress is 25% greater. If the power to be transmitted is to be increased by
20% and speed of rotation increased by 5%, determine the maximum internal diameter of the
hollow shaft taking its external diameter to be limited to 200 mm.
[CSE-Mains, 1998, ME : 30 Marks]

Solution:

3
Solid shaft Tc —(200) X
= 16

P= coT ...(ii) =T

Hollow shaft PH = 1.2 P


(4-/ = 1.05 w
1.2P
T=
1.05w

P 1 .05 it [200 4 - d 4 ] x 1.25T it


or — = x = (200)3 ti [Since Ts = TH]
w 16 1.2 200 16

1.05 (2004 -d4 )


or x1.25 - 2003
1.2 200
1.09375[2004 - c/4] = 2004
(1.09375-1)2004 = 1.09375 d4
(0.09375)2004 = 1.09375 d4
0.09375 4
_ x 200
1.09375
d = 0.541 x 200 = 108.2 mm, internal diameter of hollow shaft.
Strength of Materials Torsion 1 119

Q.7.14 The pulley A exerts a torque on shaft B as shown in figure 7.9. The total vertical tension on both
sides of the belt on each pulley is 400 kg-f. Diameter of the shaft is 6 cm. If the tensile and shear
stress intensities are not to exceed 3200 kgf/cm2 and 1600 kgf/cm2 respectively, what is the
maximum power that can be transmitted by the shaft when running at 150 rpm. Shaft may be
assumed to the simply supported at the bearings C and D.

400 kgf Rc 400 kgf RD = 66.667 kgf

Fig. 7.9
[CSE-Mains, 1989, ME : 30 Marks]
Solution:
Reaction
Moment about C
RD x 0.9 + 400 x 0.3 = 400 x 0.45
60
0 . 9 = 66.667 kgf
RD = —

Rc = 733.33 kgf
BM diagram
MA
Mc = 400 x 0.3 = - 120 kgf-m (Hogging)
MB = +66.67 x 0.45 = 30 kfg-m
Mmax = 120 kgf-m = 120,00 kg-cm
crmax = 3200 kgf/cm2
Amax = 1600 kgf/cm2

Me = Equivalent BM = 32 X d 3 X a

ic 3
= --X 6 x 3200 = 67858.4 kgf/cm
32

Te = Equivalent TM = d 3 xTmax
16 x
It 3
= — x 6 x 1600 = 67858.4 kgf-cm=VT2 + Mmax 2
16
(Putting the value of Mmax)
= 46047626629 = 144000000 + T 2
T2 = 4460762629
T = 66789 kgf-cm = 667.89 kf-m

Mi4T2 I- M 2
Me = = 67858.4
2
120 N. IAS & IFS (Objective & Conventional) Previous Solved Questions MADE EASY

2 x 67858.4 = 120,00 + VM2 +T2

123716.8 = Vm2 + T2 = V12000 2 + T2


or T = 123133.4 kgf-cm = 1231.33 kgf-m
Thus value of Twill for greater than 667.89 kgf-m (hence permissible value for shear stress will reach first
than permissib;e value of tensile stress)
2ic x 150
= = 15.708 rad/sec
60
Power = 667.89 x 15.708 kgf-m/s = 10491.2 kfg-m/s
= 140 HP, as 1 HP = 75 kfg-m

Objective Questions
Q.1 Two steel shafts, one solid of diameter D and Q.3 A stepped solid circular shaft shown in the figure
other hollow of outside diameter D and inside 7.11 is built in at its ends and is subjected to a
diameter DI2 are twisted to the same angle of torque To at the shoulder section. The ratio of
twist per unit length. Ratio of maximum shear reactive torque Ti and T2 at ends is (J1 and J2
stress in solid shaft to that in the hollow shaft is are polarmoment of inertia)
T,
4
(a) 4 (b) 8

16
(d) 1
(c) 15
[CSE-Prelims, ME : 1998]

Q.2 A load perpendicular to the plane of the handle F 1" L2 -1.1

is applied at the free end as shown in the figure Fig. 7.11


7.10. The value of (shear force), bending moment
J 2L2 J2L1
(BM) and torque (T) have been determined (a) (b) J1L2
jiLi
respectively on 400 N, 340 Nm and 100 Nm by
a student. Among the values these are J1L2 J1L1
(d)
(c) J2L1 J21-2
[CSE-Prelims, ME : 2001]
Q.4 Steel shaft and brass shaft of same length and
diameter are connected by a flange coupling.
The assembly is rigidity held at its ends and is
twisted by a torque through the coupling.
400 N Modulus of rigidity of steel is twice that of
brass. If torque of the steel shaft is 500 Nm,
then the value of the torque in brass shaft will be
(a) 250 Nm (b) 354 Nm
Fig. 7.10
(c) 500 Nm (d) 708 Nm
(a) SF, BM and torque are correct
[CSE-Prelims, ME : 2001]
(b) SF and BM are correct
(c) BM and torque are correct Q.5 A shaft is subjected to a bending moment of
(d) SF and torque are correct M = 400 Nm and a torque = 300 Nm. The
[CSE-Prelims, ME : 1994] equivalent bending moment is
Strength of Materials Torsion 4 121

(a) 900 Nm (b) 700 Nm


(c) 500 Nm (d) 450 Nm
[CSE-Prelims, ME : 2002]

Q.6 A hollow shaft of length L is fixed at its both


ends. It is subjected to torque Tat a distance Fig. 7.12
of L/3 from one end. What is the reaction torque (a) T (b) T/3
at the other end of the shaft? (c) T/2 (d) 2T/3
2T
(a) —3-- (b) 2 Q. 10 A circular shaft of length 'L' a uniform cross-
sectional area 'A' and modulus of rigidity '6' is
(c) 3 (d) 4 subjected to a twisting moment that produces
maximum shear stress in the shaft. Strain
[CSE-Prelims, ME : 2007]
energy in the shaft is given by the expression
Q.7 Consider the following statements, in connection T2AL/kG, where k is equal to
with a metallic rod of a circular section being (a) 2 (b) 4
subjected to equal and opposite torque Twithin (c) 8 (d) 16
elastic limit: [CSE-Prelims, CE : 2001]
1. The transverse section of the rod does not
Q. 11 When subjected to a torque, a circular shaft
experience warping.
undergoes a twist of 1° in a length of 1200 mm,
2. The diameter of rod does not alter.
and the maximum shear stress induced is.
3. Angle of relative twist between two sections
80 N/mm2. The modulus of rigidity of the material
is proportional to the lengths between these
of the shaft is 0.8 x 105 N/mm2. What is the
sections.
radius of the shaft?
4. A surface element of the rod is under pure
(a) 90/7r mm (b) 108/Tc mm
shear state of stress.
(c) 180/Tt mm (d) 216/7cmm
Which of these statements are correct?
[CSE-Prelims, CE : 2008]
(a) 1, 2, 3 and 4 (b) 1, 2 and 3 only
(c) 2 and 3 only (d) 2, 3 and 4 only Q. 12 Power is transmitted through a shaft, rotating at
[CSE-Prelims, ME : 2009] 2.5 Hz (150 rpm). The mean torque on the shaft
is 20 x 103 Nm. What magnitude of power in
Q.8 Which one of the following is true for torsional
kW is transmitted by the shaft?
shear stress at the axis of a circular shaft?
(a) 501c (b) 120 TC
(a) Minimum (b) Maximum
(c) 100 Tc (d) 1507c
(c) Negative (d) Zero
[CSE-Prelims, CE : 2009]
Q.9 A circular shaft of length 'L' is held at two ends
Answers
without rotation. A twisting moment 'T is applied
at a distance l'/3 from left support as shown in 1. (d) 2. (d) 3. (c) 4. (a) 5. (d)
the given figure. The twisting moment in the 6 . (c) 7. (a) 8. (d) 9. (d) 10. (b)
portion AB will be 11. (d) 12. (c)

Explanations
1. (d)
Ti . Ji = TcD4 x 32 16
Ti G6 = T22
_. T 04 04)= 15
= T2 J2 32
J1 L J2
It ( 16
12 2 0. lAS & IFS (Objective & Conventional) Previous Solved Questions MADE EASY

15
T T,
2 16 '
3
Ti = D T
16

• ( -15 D4 )x_
T2 = 2 xi' Fig. 7.14
32 06 ) D

15 7C n • (15 D3 ), , Ts Ls LB
TO LB
TB
t Because, —
16 x 16 —3 :7 16 16) GBJ B

= e, Ratio = 1 Ts _Gs
TB GB
2. (d)
SF = 400 N 5. (d)
BM = 100 Nm m = 400 Nm
T= 300 Nm

400+,4002 + 3002
Mc = 2
= 450 Nm
400 N
6. (c)

T= 100 Nm

80 Nm
400 N

400 N
T= 100 Nm

Fig. 7.15
Fig. 7.13
°BA = °BC
3. (c)

0 T1 L1 = T2 L 2 TA L TB X 2L
x —
GJ1 GL/2 GJ 3 GJ x 3

TA = 2TB
T1 = J 1 L 2
T2 J2 L1 TA + TB = T

TB = T/3
4. (a)
7. (a)
T =G
Ts s =2 All statements 1, 2, 3 and 4 are correct.
TB GB
8. (d)
Ts = 2TB
At the axis of the bar, torsional shear stress is
TB = 0.5 Ts = 250 Nm
zero.
Strength of Materials Torsion 4 12 3

9. (d) 11. (d)


L = 1200 mm
2T
TAB = 7C
0=
180
T
Bc = 3 ti = 80 N/mm2
G = 0.8 x 105 N/mm2
Because
T GO
TAB x L TBc 2L
R L
GJ 3 — GJ 3
TAB = 2 TBc T1._ 80 x 1200
= x180
TAB 4- TBC = T R= GO 0.8 x105 xic
2T = 216/ir mm
TAB =
12. (c)
co = 2.5 x 27c = 57r rad/sec
10. (b)
T= 20 x 103 Nm
T2AL Power = Tco Nm/s
Strain energy, U =
4G = 100 x 7C X 103 W
= 100 7c kW

11•IM•
08
CHAPTER
Springs

Q.8.1 A close coiled helical spring is of 80 mm mean coil diameter. The spring extends by 37.55 mm
when loaded axially by a weight of 500 N. There is angular rotation of 45° when the spring is
subjected to an axial couple of 20.0 Nm. Determine the Poisson's ratio of the material of the
spring.
[CSE-Mains, 1996, ME : 20 Marks]

Solution:
Axial load, W = 500N
Extended load = 37.75 mm
500
Stiffness, k- = 13.265 N/mm
37 .75

Gd 4
_
8nD3
Axial couple, M = 20 x 103 Nmm
ML _ 7C
0 = Angular rotation =
El - 4
L = 27EnR =TcnD

nd4
/
64
D = 80 mm

G x d4
13.245 -
8n x 803

13.245 x 8n x 803 n
G= = x 54251520
d4 d4
th ML
1. El

ML 20x103 xrcx nx80x4x64


E= =
(t)./ it xlcd4
n
= —, x130379724.14
cr
Strength of Materials Springs • 12 5

G= 54251520
d" x
E 130379729.4
= 2.4032 = 2(1+v)
G - 54251520
v = 0.2016, Poisson's ratio

Q.8.2 A close coiled spring has coil diameter to wire diameter ratio of 6. Spring deflects 30 mm under a
load of 500 N and maximum shear stress is not to exceed 350 MPa. Find diameter and length of
wire required. Modulus of rigidity of wire material = 80 GPa.

[CSE-Mains, 2012, ME : 10 Marks]

Solution:
G = 80,000 N/mm2

k = Stiffness = 300 = 16.6667 N/mm

Gd 4
8nD3
Where, D = coil diameter
d = number of coil

80000 (d)3
16.6667 = xdx
8n D

1000 d
d xi 1 )3 =
10000x —
n 6 216 x n

d
— = 16.6667 x 216 = 0.36
n 10000
D 3
Torque 500x — - —
7c x d X "C
2 16
3
7E X d x 350
250 D = —
16

250 xr) = 6 d2 x350


d 116

16 1
d2 = 250 x 6 x x 350
— =21.82696
It
d = 4.67 mm, wire diameter

n- d = 13 , number of turns
0.36
Length of the wire required, L = icDn = It x 6 x 4.67 x 13= 1144.36 mm

Q.8.3 Design a suitable helical spring for a balance which is used to measure 0 - 100 kg over a scale
of 80 mm. Spring is to be enclosed in a casing of 25 mm diameter. Approximate number of turns
is 30. Also calculate the maximum shear stress induced, G = 0.85 x 105 N/mm2.
[IFS 2011, ME : 10 Marks]
126 0. 1AS & IFS (Objective & Conventional) Previous Solved Questions MADE, EA

Solution:
Load = 0-100x9.8=0-980N
Range = 80 mm
980
Stiffness of spring, k = — = 12.25 N/mm
80

Gd 4
k -
8nD3
Where, G = Shear modulus = 85000 N/mm2
d = wire diameter
n = number of coils = 30
Casing diameter = 25 mm
So, mean coil diameter D = 25 - d
Putting the values in k expression

85000 x d4
12.25 -
8 x30 xD3

D3 = 28.91156 d4
D = 3.069 C/4/3
25 - d = 3.069 d4/3
25 = d + 3.069 d4/3
Say, d= 4 mm RHS = 4 + 19.486 = 23.486
d = 4.1 mm RHS = 4.1 + 20.138 = 24.138

d = 4.2 mm RHS = 4.2 + 20.795 = 24.995

So, wire diameter, d= 4.2 mm


Coil diameter, D= 25 - 4.2 = 20.8 mm

980 = d3T
16

980 x 20.8 x 16 = 700 MPa


2xnx 4.23

Q.8.4 (i) For a closed coil helical spring under compression, illustrate the stress distribution across
wire diameter.
(ii) Differentiate between compound and composite helical spring.
(iii) The spring load against which a valve is opened is provided by an inner helical spring
arranged within and concentric with an outer helical spring. Free length of inner spring is
6 mm larger than that of outer helical spring. Outer spring has 12 coils of mean diameter
25 mm, wire diameter 3 mm and initial compression 5 mm when the valve is closed. Find the
stiffness of the outer spring, if the greatest force required to open the valve of 10 mm is
150 N.
If the radial clearance between the springs is 1.6 mm. Find the wire diameter of the inner spring,
if it has 10 coils. For both springs, G = 82000 N/mm2.
[CSE-Mains, 2002, ME : 30 Marks]
Strength of Materials Springs 12 7

Solution:
16WR
(i) On wire diameter, Ts = torsional shear stress =
703
R = mean coil radius
4W
Td = Direct shear stress — icd2

At inner coil radius, resultant shear stress


'Cr = 'Cs + "Cd
At outer coil radius, resultant shear stress,
Tr = is — Td Fig. 8.1 Shear stress distributions
(ii) Compound spring
Springs in parallel are called compound spring
keq = k1 + k2
Composite spring
kik2
kequivalent = k1 + k2

Where k1 and k2 are stiffnesses of individual springs

(iii) Initial compression in outer spring = 5 mm


Initial compression in inner spring = 5 + 6 = 11 mm
(As the free length of the inner spring is 6 mm more than the free length of the outer spring)
Say k1 = Stiffness of inner spring in N/mm
k2 = Stiffness of center spring in N/mm
Initial load on valve = 11k1 + 5k2
Outer spring
n= 12, R = 12.5, d = 3 mm
Gd4 82000 x 34
k2 = = 4.428 N/mm
64nR3 64 x12 x 12.53
The valve is opened by 10 mm, additional force required to open the value
F0 = 10 ki + 10 k2
Ft = Total force = 10 + 10 k2 + 11 + 5 k2
= 21 k1 + 15 k2 = 150 N
k2 = 4.428 N/mm
15 k2 = 66.42
21 k1 = 150 — 66.41 = 83.58 N
83.58
k1 — = 3.98 N/mm
21
Gd4 = 82000 x

16n1F3 16 x10 xl:q
Radial clearance between the springs = 1.6 mm
Outer spring, R2 = 12.5 mm
Radial clearance = 1.6 mm
Outer coil radius of inner spring = 12.5 — 1.6 = 10.9 mm
mean coil radius of inner spring -= (10.9 — 0.5d1) mm
128 •• IAS & IFS (Objective & Conventional) Previous Solved Questions MADE EASY

82000 x dj
3.98 —
160 x (10.9 — 0.5d)3

82000 d4
(10.9 — 0.5d1)3 — x 1 = 128.770
160 3.98
10.9 — 0.5d1 = 5.049 d11.33
5.049 d11.33 + 0.5 di = 10.9
say d1 = 2 mm LHS = 13.72
d1 = 1.8 mm LHS = 11.95
d1 = 1.7 mm LHS = 11.095
d1 = 1.68 mm LHS = 10.923
Wire diameter of inner coil spring is 1.68 mm

Objective Questions

Q.1 Two identical springs, each of stiffness k are


1 2 1 2
assembled as shown in the figure 7.2. (a) — kx2 - - kX1 (b) 2 k(xi — x2 )2
2 2
Combined stiffness of assembly is
1 (x1 X2
2, + xi)2
(c) — (d) k
2
[CSE-Prelims, ME : 1999]

Q.4 A close coiled helical spring has wire diameter


10 mm and spring index 5. If the spring contains
k2
10 turns, then the length of the spring wire
would be
(a) 100 mm (b) 157 mm
Fig. 7.2 (c) 500 m (d) 1570 mm

(a) k2 (b) 2k Q.5 Match List-I (Type of spring) with List-II


(c) k (d) k/2 (Application) and select the correct answer
[CSE-Prelims, ME : 1998] List-I
A. Leaf/helical spring
Q.2 Two close coiled springs are subjected to same
B. Spiral springs
axial force. If the second spring has 4 times
C. Beleville springs
the coil diameter, double the wire diameter and
List-II
double the number of coils of the first spring,
1. Automobile/Railway coaching
then the ratio of deflection of the second spring
2. Shearing machine
to that of the first will be
3. Watches
(a) 8 (b) 2
Codes:
(c) 1/2 (d) 1/16
A B C
[CSE-Prelims, ME :1998]
(a) 1 2 3
0.3 A spring of stiffness k is extended from a (b) 1 3 2
displacement x1 to a displacement x2 . Work (c) 3 1 2
done by the spring is (d) 2 3 1
[CSE-Prelims, ME : 2000]
Strength of Materials Springs 129

Q.6 The equivalent spring stiffness for the system Q.9 A close-coiled helical spring has 100 mm mean
shown in figure 8.3. CS is the spring stiffness diameter and is made of 20 turns of 10 mm
of each of the three springs. diameter steel wire. The spring carries an axial
load of 100 N. Modulus of rigidity is 84 GPa.
The shearing stress developed in the spring in
N/mm2 is
(a) 120/it (b) 160//n
(c) 100/it (d) 80/7c
[CSE-Prelims, CE : 2003]

Q.10 For a laminated spring, 1= length, n = number


of plates, width of each plate = b, thickness of
Fig. 8.3 each plate = t, central point load = W What is
(a) S/2 (b) S/3 the expression for the maximum bending stress?
(c) 2S/3 (d) S
3WI 3W1
[CSE-Prelims, ME : 2001] (a) f = , (b) f =
4nkt 2nb2t
Q.7 If the number of turns in a spring are halved, its
2W1 3W1
stiffness is (c) f (d) f =
3nb2t 2nbt 2
(a) halved
[CSE-Prelims, CE : 2007]
(b) doubled
(c) increased from turns Q.11 A close coiled helical spring is subjected to an
(d) not clayed axial moment M, producing an angle of rotation
[CSE-Prelims, ME : 2007] of 90° at free end with respect to the fixed end.
Strain energy absorbed in spring is 1007t Nmm,
Q.8 A closed coiled helical tension spring having
20 coils is cut in two parts such that part A has what is M
8 coils and part B has 12 coils. What is the (a) 100 Nm (b) 200 Nmm
ratio of stiffness of the spring A to the stiffness (c) 300 Nmm (d) 400 Nmm
of the spring B Answers
(a) 1 (b) 1/2
1. (b) 2. (a) 3. (a) 4. (d) 5. (b)
(c) 3/2 (d) 3/4
6. (c) 7. (b) 8. (c) 9. (d) 10. (d)
[CSE-Prelims, ME : 2006]
11. (d)

Explanations
1. (b)
Same deflection 8 for both, k = k1 + k2 = 2k 2. (a)
Gd 4
k,
64nR3
W
k1 =
W = 88
k2 1
Fig. 8.4
130 0- IAS & IFS (Objective & Conventional) Previous Solved Questions MADE EASY

Gx16d4 7. (b)
k, = Doubled
64 x 2n x 64R3
Gd 4 16 Gd 4
=x
64nR3 x 128 64 Ran
Gd 4 [11 n is in denominator.
64nR3 L8
8. (c)
3. (a) A = 8 coil
B =12 coils
1
kA oc 8

r‘B c'c 12
k A 12 , 3
-4'i
= — = 1.0 =
[4 X2
kB 8
Fig. 8.5
9. (d)
D= 100 mm
— kx2 2 --kxi2 = Work done
2 2 R 50 mm
d= 10 mm
4. (d) n = 20
L= irDn.nx5dn W= 100N
= nx5x10x10 G = 84 GPa
= 1570 mm
16WR
5. (b) rcd3
A. Leaf/helical spring 16 x100 x 50 80
B. Spiral Spring = N/mm2
Tcx103
C. Belleville spring
1. Automobiles/Railway coach 10. (d)
3. Watches L = Length
2. Shearing machine n = number of beam
W = Load
6. (c)
t = thickness
b = width
WL
n6 x
4 = 6 x'
3 WL
f_ x ,
2 nbt`
11. (d)
1 7r
2Mx2 = 10076
Fig. 8.5
M= 400 Nmm
2S x S _ 2 s
S8
`je 2S+ S 3
09
CHAPTER
Struts and Columns

Q.9.1 The critical buckling load of a cast iron hollow cylindrical column 3 m in length, when hinged at
both the ends is equal to PkN. When the column is fixed at both the ends, its critical load increases
to (P+ 300) kN. If the ratio of external diameter to internal diameter is 1.25, E= 100 GPa, determine
external diameter of column.
[CSE-Mains, 2012, ME : 12 Marks]

Solution:
Critical load, when both ends hinged = P
Critical load when both ends fixed = 4P = P + 300 kN (Since effective length is half)
So P = 100 kN
E = 100000 N/mm2
L = 3 m = 3000 mm

n2EI
P- 12

ic 2 X100000 XI
100000 -
30002

30002
_ 2 — 911890.653 mm4
Tc

-t--(D4
1 -d4 )
64
d= 0.8D

I
[ 4 - 0 4096D4 ] = 0.02898D4
= 64 D
0.02898 D4 = 911890.653
D4 = 3146.49 x 104 mm4
D = 74.89 mm, external diameter of CI column.

Q.9.2 A hollow column, 400 mm external diameter and 300 mm internal diameter, is hinged at both
ends. If the length of the column is 5 m, E = 0.75 x 105 N /mm2, factor of safety 5. Rankine's
constants 1/1600 and crushing stress 587 N/mm2, find the safe load, the column can carry
without buckling. Use Euler and Rankine, formulae.
[CSE-Mains, 2005, ME : 30 Marks]
13 2 P. IAS & IFS (Objective & Conventional) Previous Solved Questions MADE EASY

Solution:
E = 75 x 103 N/mm2
D = 40 cm
d = 30 m
700
A = 4 (1600 — 900) = -iL = 175 5c cm2

/ = 7 (402 — 302 ) = 8.59 x 104 cm4 = 8.59 x 108 mm4


64
1= 5m
n2E1 5c2 x 75000 x 8.59 x108
P = Euler buckling load — 2 =
50002
= 25433970.54 N = 25433.97 kN
FOS = 5
Pe safe = 5086.8 kN
_ I _ 8.59 x108 = 1.5624 x 104 mm2
— A — 1755tx100
12 50002
=1600
k2 k2
.A 587x1757cx100
PR— /2 = 1
1+ax--g— 1+ x 1600
k2 1600
= 161360.0 x 100 N = 16136 kN
PR safe = 3227.2 kN
Safe load without buckling = 3227.2 kN

Q.9.3 A vertical column 6 m length is fixed at the base and a clockwise moment of 1.4 kNm is applied at
the top of the column, a horizontal force of P is applied to the column at a height of 3 m above the
base so as to give a CCW moment.
Determine the value of force P so that horizontal deflection at the top of the column and at the
point of application of P shall be equal (i) when the deflections are on the same side (ii) when the
deflections are on the opposite sides of the vertical line through the foot of the column

[CSE-Mains, 1993, ME : 20 Marks]

Solution:
Take a section yy at distance x from A
Mx = 1.4 — P(x — 3)
d2y
El , = 1.4 — P(x — 3)
Az'
dy
El = 1.4x --(x— 3)2 +ci
dx 2
dy
0, at x = 6
dx =

0 = 1.4 x 6- 2 (3)2 +ci


= 8.4 — 4.5 P+
Constant, c1 = 4.5 P— 8.4 Fig. 9.1
Strength of Materials Struts and Columns 4 133

dy
El- = 1.4x - -(x - 3)2 + (4.5P - 8.4)
dx 2

El y = 0.7x 2 - f(x - 3)2 + (4.5P - 8.4)x+ C2

y = 0, at x = 6, from top

= 0.7 x 36-i x 27 +(4.5 P-8.4)x 6 +C2

0 = 25.2 - 4.5 P+ 27 P- 50.4 + C2


= 22.5 P- 25.2 + C2
C2 = 25.2 - 22.5 P
25.2 - 22.5P
Deflection at, x = 0, yA =
El
At x = 3 m, Elyc = 0.7 x3 2 -0 + (4.5 P- 8.4) x 3 + 25.2 - 22.5 P
= 6.3+ 13.5 P- 24.6 + 25.2 - 22.5 P
= - 9P+ 6.9
6.9 - 9P
Y,
Deflections on same side

25.2 - 22.5P 6.9 - 9P


El El
18.3P = 13.5P
P = 1.335 kN
Deflections on opposite sides
25.2 - 22.5P 9P - 6.9
El El
31.5P = 32.1
= 1.019 kN (For deflection on opposite side)

Q.9.4 A cast iron column of circular section 20.0 cm external diameter and 2.0 cm thickness and of
height 4.0 meters is required to carry 15.0 kN at an eccentricity of 2.5 cm. If both the ends are
fixed, find the extreme stresses In the column section. E. 100 GPa.
[CSE-Mains, 2011, CE : 15 Marks]
Solution:
P= 15 kN, D= 20 cm, d= 16 cm

A, area of cross-section, A = -71(202 -162 ) = = 113.1 cm2


4
15000
ad , direct shear - 132.626 N/cm2 = -1.32626 N/mm2 (Compressive)
113.1
eccentricity, e = 2.5 cm = 25 mm
4 .1p
= P-+-
Pe
sec -
MaX A Z 2 E/-

le = equivalent length = = 2 m (fixed ends)

I= (D4- = (204 -164 )


64 64
134 ► lAS & IFS (Objective & Conventional) Previous Solved Questions MADE EASY

= it-[-I 60000 - 65536] = 4637 cm4 = 4637 x 104mm4


64
E I = 100000 x 4637 x 10+4 Nmm2 = 4637 x 109 N/mm2
I 15000 = 1 I 15 _ 1
_
1/EPI \ 4637 x 109 1000 4637
\ 1000 x 17.582

le X, 1
\IP = 1000x 1 = 0.05688 rad = 3.258°
2 El 1000 x 17.582 17.582
sec 3.258° = 1.0016
Z= = 4637 x 104 mm4 = 4637 x 100 mm3
100

p 15000 x 25
-e sec3.258° = x 1.0016 = +0.810 N/mm3
463700
Amax = 1.32626 + 0.810 = 2.13626 N/mm2 (comp.)
Amin = 1.32626-0.810 = 0.51626 N/mm2 (comp.)

Q.9.5 A built up column consists of three ISWB 450 CD


0.794 kN per m connected effectively to act as one
column (below figure).
Determine the safe load carrying capacity of built
up section, if the unsupported length of column is -x
450 mm
4.25 m. Properties of ISWB 450 10 0.794 kN/m.
Area = 101.15 cm2
I = 35057.6 cm4
/YY= 17067.7 cm4
tW = Thickness of web = 9.2 mm
f = 250 MPa, yield stress.
Slenderness Ratio 10 20 30 40 50 60 70
Permissible stress in
axial compression in MPa
fr = 25 MPa. 550 148 145 139 132 122 112

[CSE-Mains, 2013, CE : 10 Marks]


Solution:
Area of cross section = 3 x 101.15 cm2 = 303.45 cm2
I = 2 x 35057.6 + 1706.7 = 71821.9 cm4
/wPassing through G
= 1 + 2Iyy + 2 x 101.15 (22.5 + 0.46)2
= 35057.6 + 2 x 176.7 + 202.3 x 527.1616
= 35057.6 + 3413.4 + 106644.8 = 145115.8 cm4
< I yy

,, 71821.9
K- - - 236.684 cm2
303.45
k = 15.38 cm
L = 4.25 m = 425 cm
L
- = 27.63
k
Strength of Materials Struts and Columns 4 13 5

From the table


30 - 27.63
f = 145 + x 3 = 145 + 0.711 = 145.71 N/mm2
10
Safe load = 145.71 x 303.45 x 100 N
= 4421570 N
= 4421.57 kN
= 4.42157 MN

Objective Questions

Q.1 Which one of the following statements is Q.3 A short vertical column having a square cross
correct? section is subjected to an axial compressive
(a) Eulers formula holds good only for short force, centre of pressure of which passes
columns through the point R, as shown in figure.
(b) A short column is one which has the ratio of Maximum compressive stress occurs at point.
its length to least radius of gyration greater
than 100
(c) column with both ends fixed has minimum
A
equivalent length or effective length
(d) The equivalent length of column with one
end fixed and other end hinged is half of
its actual length
[CSE-Prelims, ME : 2000]

Q.2 A strut's cross-sectional area A is subjected to


(a) S (b) Q
a load Pat point S(h, k) as shown in the figure.
(c) R (d) P
Stress at the point Q(x, y) is
[CSE-Prelims, ME : 2002]
Q.4 Which one of the pairs is not correctly matched
(a) Slenderness ratio 1. Ratio of length of
the column/least
Q(x, y)
radius of gyration.
S(h, k)
x (b) Buckling Factor 2. Ratio of maximum
load to the
permissible axial
load on the
y column.
(c) Short Column 3. A column for which
P + Phy Pkx slenderness ratio
(a) — +
A Ix I y < 32.
P Phx Pky (d) Strut 4. A member of a
(b) structure in any
A Iy
position and
P _Phx Pky
(c) carrying an axial
A Ix I y
compressive load.
(d) P + Phx Pky [CSE-Prelims, ME : 2003]

A ly I x
Q.5 What is the cause of failure of a short MS strut
[CSE-Prelims, ME : 2000]
under an axial load?
13 6 0. lAS & IFS (Objective & Conventional) Previous Solved Questions MADE EASY

(a) Fracture stress (b) Shear stress (a) 2 (b) 1/2


(c) Buckling (d) Yielding (c) 4 (d) 1/4
[CSE-Prelims, ME : 2007] [CSE-Prelims, CE : 2007]

Q.6 Which one of the following columns has effective Q.11 In an axially loaded compressive member with
length twice the value of actual length? a circular cross-section of radius r, what is the
(a) Hinged-Hinged column radius of the core section which is proof against
(b) Fixed-Fixed column tensile stress?
(c) Fixed-Hinged column (a) r /2 (b) r /3
(d) Fixed-Free column (c) r /4 (d) r /6
[CSE-Prelims, ME : 2009] (CSE-Prelims, CE 2009]

Q.7 Critical Euler buckling load for a long column of Q.12 A Short column of external diameter D and
diameter D was evaluated as P. If the diameter internal diameter 01 is subjected to an eccentric
of the section is reduced to D/2, what is the load P at an eccentricity of e thereby causing
load carrying capacity of the modified column? tensile stress at the extreme fiber. What is the
(a) P/2 (b) P/4 magnitude of eccentricity?
(c) P/8 (d) P/16 D2 +02
D2 +
[CSE-Prelims, ME : 2009] (a) (b)
8nD 8D
Q.8 The axial load which just produces the condition D2 — D1 D2 —D-f
of elastic instability in a column is (c) (d)
870 80
(a) Rankine load (b) Euler load [CSE-Prelims, CE : 2010]
(c) Yield load (d) Crushing load
[CSE-Prelims, CE : 2002] Q.13 A column of length 4 m, an area of cross section
2000 mm2, moments of inertia, /xx = 720 cm4,
Q.9 If the stress on the cross-section of a circular lye = 80 cm4, is subjected to a buckling load.
short column of diameter D is to be wholly Both the ends of the column are fixed. What is
compressive, the load should be applied within the slenderness ratio of the column
a concentric circle of diameter (a) 200 (b) 120
(a) D/2 (b) D/8 (c) 100 (d) 80
(c) D/4 (d) D/6
[CSE-Prelims, CE : 2003] Answers

Q.10 Column C1 has both the ends hinged while the 1. (c) 2. (b) 3. (a) 4. (b) 5. (d)
column C2 has one end hinged and other end 6. (d) 7, (d) 8. (c) 9. (0) 10. (b)
fixed. What is the ratio of the critical load for C1 11. (c) 12. (b) 13, (c)
to that of C2 according to the Euler's formula?

Explanations

1. (p) 2. (D)
Only statement (c) is correct. All compressive in I-quadrant
In column if the both the ends fixed, has minimum
P Ph x_Pyk
equivalent length or effective length i.e. 2 . A 1), 1,,

e =h
Strength of Materials Struts and Columns 4 13 7

10. (b)

n2E1
C1,both ends hinged P1 =
(x, y)Q
(h, k)S
---
C2,one end fixed, other hinged P2 =

=
P = 0.5
C2 P2

e=k 1 1 . (c)
P Px Column of circular section of radius r
----xe --
PY-xe (Compressive)
A Iy x x Y
radius of cone =
4
3. (a)
p P(QR) 12. (b)
°max = + ab =A+ x (QS)
I YY External diameter D,

2 2
4. (b) 7C (D - D )
Internal diameter 01, A= -
4
(b) is not correctly matched
= 1`— (D4 —D41 )
5. (d) 64
ms strut short in length, fails by yielding
4P
a
6. (d) c n(D 2 - Of)
for a fixed, free column, effective length is twice
P.e x x 64
the actual length. ab —
2 x n(D4 -D4)
7. (d)
32 Pe D _ 32 Pe D
m(D4 -D11) n(D2+EX)(02 -DD
D is reduced to -
2
32 Pe D 4P
n(D2+ DD(D2 - n(D 2 - Di
)
I is reduced to —
16
D2 +
e-
P 8D
Pis reduced to iF
6
13. (b)

8. (c) Imin = 80 cm4


Yield load produces elastic instability in a column A = 20 cm2
= 180 = 2 cm
9. (c) kmin
,\ 20
D , core of section.
Concentric circle of diameter _ L
4 le = - (Fixed ends) = 200 cm
2
le =
200 =100
kmin 2

II • III •
10
CHAPTER
Theories of Failure

Q.10.1 Explain what do you understand by theories of failure. Compare any three failure theories
graphically for an element subjected to two mutually perpendicular direct stresses.
[CSE-Mains, 1989, ME : 20 Marks]

Solution:
A body may be subjected to various combinations of M (bending moment), T (twisting moment) and
F(axial force) and may fail. At the critical section of the body principal stresses p1 > p2 > p3 can be
worked out. Then three principal stresses individually or in combination are compared with the uniaxial
tensile stress for a bar, under which the bar fails. For a brittle material it is no ultimate tensile strength but
for ductile material it is aYP' yield point stress.
Generally for any plane stress problem, two principal stresses p1 > p2 and
p3 = 0 are determined as shown. There are five theories of failure but we let
us take following 3 theories of failure
1. Maximum principal stress theory
2. Maximum shear stress theory
3. Strain energy theory
P2
P2
YP
YP

0J 0

P1
—= x
aYP

(a) (b) (c)

Fig. 10.1

Maximum principal stress theory (Rankine's theory)


P11 Gyp
P2 5' Gyp

p1 < ±1

YP

P2 < as shown in figure (a).


YP
e.A
Strength of Materials Theories of Failure 4 13 9

Maximum shear stress theory (Tresca theory)

P1 P2 YP
2 - 2

1P1 - P21 5- cryp


If p1 and p2 both are positive

P2
Pi or —
Then t=—
i
2 2

If p1 > 0 and p2 < 0, then


(P1 P2) < cryp
Fig. (b) shows graphical representation of maximum shear stress theory.
Strain energy theory
[p12 p22 - 2vp1p2] 0. 2
yp
2 2
P1 + P2
or 2 2
2v P1P22 < 1
a YP a
YP aYP

or (x2 + y2 - 2vxy) 5_ 1, ellipse

P1 P2
where, x- =
a YP a YP

Fig. (c) shows graphical representation of strain energy theory.

Q.10.2 What is failure theory? Discuss its importance. Enumerate various failure theories and mention
their fields of application.
[CSE-Mains, 1997, ME : 15 Marks]

Solution:
There are various theories of failure depending upon various types of stresses on a body and different
types of strain energies absorbed by a body. These theories of failure are based on three dimensional
response of any component due to external load, and at a particular section of interest, these principal
stresses pi> p2 > p3 are determined. Analysis is based on the parameters of a simple tensile test on the
material. In case of ductile material, yield point stress Gyp and in case of brittle material but are determined.
Principal stresses or strain energies are compared with stresses in simple tensile test and strain energies
in simple tensile test.
There are 5 theories of failure
1. Maximum principal stress theory
< used for brittle materials
2. Maximum shear stress theory
G
P1 P3 YP
, used for ductile materials
2 2
3. Maximum principal strain theory

(pi - vp2 - vp3


< YP theory is not acceptable to designers
) E

or [P1 - v(P2 P3)] ayp


140 0. lAS & IFS (Objective & Conventional) Previous Solved Questions MADE EASY

4. Maximum strain energy theory


2
2 3 1 a via
[Pi + P2 + P3 - 2 v(PiP2 P2P3 + P3P1)] - '-
2E 2E

v, Poisson's ratio
This theory is also not acceptable to designers
5. Shear strain energy theory
[(P1 - P2)2 + (P2 - P3)2 + (P3 - 2ayp2
For ductile materials, the theory is in good agreement with the experimental results and is used for the
design of shafts.

Q.10.3 Explain the salient features of maximum distortion energy theory of failure and discuss how it
compares with maximum strain energy theory and maximum shear stress theory.

Solution:
Maximum distortion energy theory
It principal stresses are p1 > p2 > p3 at a point of a body, then as per distortion energy theory
[(P1 - P2)2 + (P2 - P3)2 + (P3 - P1)2] aryl!
Where aYP = yield point stress of the material in simple tensile test.
This theory is used for ductile materials and specially in the design of shafts.
Maximum strain energy theory
[P12 + p22 + p32 2V(P1 P2 + P2P3 P3P9] 5 ayp2
where, v = Poisson's ratio
However this theory is not used by designers.
Maximum shear stress theory of p1 > p2 > p3

Pi -P3 < a YP
2 - 2
This theory is used for the design of components of ductile materials. This theory provides more factor of
safety and a conservative design of a component is obtained.

Q.10.4 State and deduce the strain energy of distortion theory of elastic failure and compare it with
maximum shear stress theory with respect to the field of application and suitability for optimization.

[CSE-Mains, 1998, ME : 15 Marks]

Solution:
u = total strain energy per unit volume

= —1 [Pi +14 + - 2 v(P 1P2 P2P3 P3P1)]


2E
= uv + vs
= Volumetric strain energy + Shear strain energy

= due to volumetric stress, here pv = Pi + P2 + P3 = Pm


3

3,2
uv -2v(3p,i.1 )] = t'm (1 2v)
2E 2E
Strength of Materials Theories of Failure 4I 141

= —X
3 (Pi + P2 +P3 )2 (1— 2v)
2E

u = u— uv, taking (1 + v) = G , taking v = 0.5, volumetric strain energy becomes zero.


2

us = (1+ v) 3E + p2 + /313 — (PiP2 P2P3 P3P1)]

1 [ 2 + P32 /
= — Pi+ P2 kP1P2 + P2P3 + P3P1)]
6G
2
a_ 0 2 a yp
1 [4
0 D,2
2p2 ,p3 — 0 — 2p2p3.— 2p3pi]
= 12G ' 6G
[(P1 — P2 )2 + (P2 + P3)2 + (P3 2ayp2
Maximum shear stress theory is used for ductile material and gives the maximum size of the component.
Distortion or shear strain energy is also used for ductile material, but gives slightly lower value of dimensions
of a component, as diameter of a shaft. Using this theory, shaft diameter can be optimized.

Q.10.5 (a) What is the drawback of maximum principal stress theory?


(b) A body is under action of two principal stresses of +40 N/mm2 and —70 N/mm2 and the third
principal stress is zero. If the elastic limit in simple tension as well as in compression is
200 N/mm2, find the factor of safety based on the elastic limit according to
(i) maximum shear stress theory
(ii) maximum strain energy theory
(iii) maximum shear strain energy theory
Take v = 0.3
[CSE-Mains, 2008, ME : 20 Marks]

Solution:
(a) Maximum principal stress theory is used for brittle materials. It is not advisable to use this theory
for ductile materials
If principal stresses p1 > 0, P2 >
Then this theory coincides with maximum shear stress theory
If p1 > 0, p2 < 0, then
Pi + p2 GYP
"Cmax
2 — 2xFOS
av„
FOS —, (Factor of safety is reduced)
+ P2)
(b) p1 = +40 N/mm2
p2 = —70 N/mm2
cse = 200 N/mm2
(i) Maximum shear stress theory

40 — (-70) 200
2 2 xFOS

200
FOS =1.82
= 110
14 2 0. IAS & IFS (Objective & Conventional) Previous Solved Questions MAINEEA

(ii) Maximum strain energy theory


2
63q3
[(p12 + p22-2v(p1p2)] =
FOS

2
G
yp
[702 + 402 - 2 x 0.3(-70)(40)] = 1—
FOS

(4900 + 1600 + 1680) = GY


FOS)

a „
90.44 -
FOS

Factor of safety, FOS - 200 = 2.211


90.44
(iii) Maximum shear strain energy theory
)2
( a
[ p 2 p 2 p p ]

1 2 1 2
FOYPS

( )2
a
YP
[(-70)2 + (40)2 + 70 x 40]
FOS

2
YP
(4900 + 1600 + 2800) <
FOS 2

6 YP
96.43
FOS

200
FOS - = 2.074
96.43

Q.10.6 A round member is subjected to a direct tensile load of 20 kN and shear load of 12 kN. The yield
stress in tension is 25 kN/cm2, and Poisson's ratio 0.3. Determine the diameter of the member,
using a factor of safety of 2 according to
(i) Maximum principal stress theory
(ii) Maximum shear stress theory
(iii) Maximum distortion energy theory

Solution:
20 kN
Direct stress, 61 =
A
12 kN
Shear stress, T -
A

10 kN + (10 kN)2 (12k12 25.


62kN
Principal stress, p1 = A A ) A A

-5.62 kN 15.62 kN
P2 = A ' tmax - A
Strength of Materials Theories of Failure -4I 143

Gyp 25 kN/cm2
=

FOS = 2
Gyp all = 12.5 kN/m2
(i) Maximum principal stress theory
25.62 kN
- 12.5 kN/cm2
A

2
A = 2.0496 cm2 = 4 d

d= 1.6 cm = 16 mm
(ii) Maximum shear stress theory
15.62 12.5 kN
A 2 cm 2
A = 2.4992 cm2 F
d= 1.78 cm = 17.8 mm
(iii) Maximum distortion energy theory
(pi 2 2_ p p ) a 2

p2 i 2 yp

2 2
F25.621 +1-5.621 + [25.62 5.62]
- (12.5)2
L A 1_ A L A2
156.25 A2 = 656.3844 + 31.5844 + 143.9844
= 831.9532
A= 2.3075
d= 1.714 cm = 17.14 mm

Q.10.7 A hollow shaft 30 mm inner diameter and 50 mm outer diameter is subjected to a twisting moment
of 800 Nm and an axial compressive force of 40 kN. Determine the factor of safety according to
theories of feature based on normal stress theory, shear stress theory and distortion energy
theory. The tensile and compressive yield strength of material is 280 N/mm2 and Poisson's ratio is 0.3.
[CSE-Mains, 2006, ME : 20 Marks]

Solution:
Shaft, D. 50 mm, d = 30 mm
A II (502 3U
-2) _ 400 7t mm2
Area,
4

504 3-4)
U = 53.40 x104 mm4
32 (
Torque, T= 800 Nm = 800 x 1000 Nmm
T
J 25
T 800 x 103x 25
Shear stress, ti = — x 25 =
53.4 x 104 = 33.45
= 37.45 N/mm2
P 40,000
a, axial compressive stress - =
A 400/1
= -31.83 N/mm2 31.83
144 ► IAS & IFS (Objective & Conventional) Previous Solved Questions MAD EAU

Principal stresses

31.83 (31.83)2
+37.452
P1' P2 = 2 ± 2

= -15.915 ± V253.28 +1402.5

= -15.915 ±,1655.7825
= -15.985 ± 40.691
p1 = +27.776 N/mm2 (tensile)
p2 = -56.606 N/mm2 (comp.)
Itmaxi = ±40.691 N/mm2
yp = 280 N/mm2
Maximum normal stress theory

4:3 YP
P2 — FOS
280
FOS - = 4.946
56.606
Maximum shear stress theory
CT YP

max 2 x FOS
280
40.691 -
2 x FOS
140
FOS - = 3.44
40.691
Distortion energy theory
a )2
[(27.776)2 + (-56.606)2 - (27.776X-56.606)] = YP
IFOS

p
(771.506 + 3204.24 + 1572.29) =
FOS

yp
or 15548.036 - FOS

280
FOS - = 3.759
74.485

Q.10.8 A hollow circular steel shaft is subjected to a torque of 800 Nm and a bending moment of 1200 Nm.
The internal diameter of the shaft is 60% of the external diameter. Determine the external diameter
of the shaft according to
(i) maximum principal stress theory
(ii) maximum shear stress theory
(iii) shear strain energy theory
Take factor of safety as 2 and the yield strength of the material as 27 kN/cm2.
Strength of Materials Theories of Failure 4 145

Solution:
aYP = 27000 N/cm2 = 270 N/mm2
T= 800 Nm
M = 1200 Nm
FOS = 2
ayp = allowable = 135 N/mm2
External diameter = D, internal diameter = 0.6 D

1 = 6 [D4 —(0.604 ]= 0.042726D4


4
J = 21= 0.08545 D4

M D 1200x103 xD 14043 x 103


ab, maximum bending stress =
x 2 — 0.042776 x 2D4 D3

T D 800 x 10 3 D 1 4681x 10 3
T, maximum shear stress = x = x x =
J 2 0.08545 2 D4 D3
Principal stresses

ab (5b2 'T
2
2 2

7021.5 8438.8 15460.3 3


x 10
D3 D3 D3
7021.5 8438.8 —1417.3 103
= , X
P2 — D3 D'
(I) Maximum principal stress theory

103° x 15460.3 < 135


D3
D3 = 114.5153 x 103
D = 48.55 mm
(ii) Maximum shear stress theory
8438.8 x 103 _ 135
D3 —2
D3 = 125.02 x 103
D = 50 mm
(iii) Shear strain energy theory
(p 2 + p 2 p p )

1 2 1 2

2 2
115460.3 x10- 1-141,3 x101 (15460.3 x1417.3 x106
) = 1352
D3 D3 D3 x D3 )
1352 x D6 = 106[239020.8761 + 2008739.3 + 21911883.2]
1352 x D6 = 106[262941498.6]
06 = 106 x 14427.517
D = 49.34 mm
146 0. lAS & IFS (Objective & Conventional) Previous Solved Questions MADE EASY

Q.10.9 A hollow shaft of outside diameter 50 mm and inside diameter 20 mm is subjected to a torque of
T Nm and a bending moment of 0.5 T Nm. If the tensile yield stress of the shaft material is
250 N/mm2, what is maximum permissible value of Tto avoid failure according to Tresca's failure
theory? Take a factor of safety of 2.0 for a given application.
[CSE-Mains, 2011, ME : 15 Marks]
Solution:
aYP= 250 N/mm2
FOS = 2
Allowable, a = 125 N/mm2

Allowable shear stress, ti = 2 = 62.5 N/mm2

Shaft, D= 50 mm
d= 20 mm

it (D4 -d 4 ) it (50 4 - 20 4 )
Torque, T- x x 62.5 - 16 x 62.5
16 D x 50
71 609
_ x x 62.5 x 104 Nmm = 149.47 x 104 Nmm = 1494.7 Nm
16 50
Torque, T = T Nm
BM, M' = 0.5 T Nm

Tv equivalent torque = VT2 +(0.5T)3 = 1.118 T Nm = 1494.7 Nm

T = 1336.9 Nm

Q.10.10 The load on a rod consists of an axial pull of 10 kN along with a transverse shear force of 5 kN.
Determine the diameter of the rod by using the following theories of failure
(i) Strain energy theory
(ii) Shear strain energy theory
Elastic limit in tension is 270 N/mm2 and a factor of safety of 3 is to be used.
Poisson's ratio = 0.3
[CSE-Mains, 2011, CE: 15 Marks]
Solution:
10,000
6-
A
5000
A
A = area of cross-section
Principal stresses are
2 2.414 x 5000 12070
a 11(1 2 5000 + x 5000
= — + 2 +i =

A A
2 A A

= 5000 .4 x 5000 0.414 x 5000 2070


± T2 =
P2 = 2J A A A A
2
(i) Strain energy theory
ae = 270 N/mm2
Strength of Materials Theories of Failure 147

FOS = 3
270
aAllowable = = 90 Nimm2
3
Poisson's ratio = 0.3
[p12 + p22 - 2v pi p2 ] 5. al

12070)2 +( 2070)2 2 x 0.3 x 12070 x 2070] =902


) +
A A ) A2
[(

[14.5684900 + 4284900 - 14990940] = 8100A2


A2 = 20365.523
2
—d
A = 142.708 mm2 = 4
Rod diameter, d = 13.48 mm
(ii) Shear strain energy theory
[P12- P1P2 + ,022] = aa2

12070)2 112070 X 2070) ( 2070)2 ] = 902


+ +
A ) A2 A
[1

1
--2 [145684900 + 24984900 + 4284900] = 902
A
A2 = 18823.23

A = 137.2 mm2 = 1-t-d2


4

Rod diameter, d = 13.22 mm

Objective Questions
Q.1 A certain steel has proportionality limit of Q.3 Which of the following is the most appropriate
300 N/mm2 in simple tension. Under three theory of failure for mild steel?
dimensional stress system, the principal (a) Maximum principal stress theory
stresses are 150 N/mm2 (Tensile), 75 N/mm2 (b) Maximum principal strain theory
(Tensile) and 30 N/mm2 (Compressive), p. = 0.3. (c) Maximum shear stress theory
The factor of safety according to maximum shear (d) Maximum strain energy theory
stress theory would be [CSE, Prelims-CE : 2009]
(a) 1.4 (b) 1.5
(c) 1.3 (d) 1.66 Q.4 A shaft is subjected to a torque and an axial
[CSE-Prelims, CE : 2001] compressive force. Shear stress due to torque
is 30 MPa and compressive stress due to axial
0.2 Which one of the following theories of failure is force is 80 MPa. If yield strength of material is
widely used in the design of a machine element 255 MPa, what is factor of safety as per the
made of a ductile material? maximum principal stress theory?
(a) Maximum normal stress theory (a) 3.19 (b) 3.01
(b) Maximum strain theory (c) 2.83 (d) 3.64
(c) Strain energy theory
Q.5 The principal stresses developed at a point are
(d) Maximum shear stress theory
+80, -80 MPa, 0.0. Using shear strain energy
[CSE-Prelims, CE : 2003]
148 0 1AS & IFS (Objective & Conventional) Previous Solved Questions MADE EASY

theory factor of safety is 2. What is yield strength Q.7 A thick cylinder of an internal diameter 100 mm
of the material in MPa. and external diameter 200 mm is subjected to
(a) 150 MPa internal pressure p. Yield strength of the material
(b) 80fd MPa
is 240 MPa. Using maximum shear stress theory
(c) 240 MPa (d) 277 MPa
FOS is 2. What is the maximum value of internal
Q.6 The principal stresses at a point are 70, 60 and pressure p?
-80 MPa. If is the yield point stress of the
Gyp (a) 120 N/mm2 (b) 90 N/mm2
material, using maximum principal stress theory (c) 72 N/mm2 (d) 60 N/mm2
factor of safety is 3.6. What is FOS if the
Answers
maximum principal strain theory is used.
Poisson's ratio of material is 1/3 1. (d) 2. (d) 3. (c) 4. (c) 5. (d)
(a) 3.29 (b) 3.9 6. (a) 7. (b)
(c) 4.5 (d) 5.5

Explanations
1. (d) 5. (d)
a = 300 N/mm2 Shear strain energy theory
Principal stress are p1 = 80, p2 = -80, p3 = 0
+150, +75, -30 N/mm2
Gyp
Poisson's ratio = 0.3 (P12 - P1P2 + P22) = FOS
150+30)
-Maximum shear stress = 90
2
802 + 802 + 802= 3 x 802 =
ae 2
= i8 = 150
2 ayp = 138.56 x 2 = 277 MPa
150 6. (a)
FOS - =1.66
90 Maximum principal stress = 70
2. (d) FOS = 3.6
For ductile material ayp 70 x 3.6 = 252 MPa
=

(d) Maximum shear stress theory is widely used. Maximum principal strain
1 1
3. (c) 70 - 3(60)+ 3(80)
76.66
Mild steel — a ductile material =
E E
(c) Maximum shear stress theory
252
4. (c) FOS = = 3.287
76.66
7. (b)
R22 + Ri2
5
80 ac ax = P x 2
R2 - Ri2 = 3P

5
3 P+P
- -1-A
= p
tmax 2 3
pm = -40 -,402 +302 = -90 N/mm2
4
a = 255 -p = 120
3
255 p = 90 N/mm2
FOS = — = 2.83
90
11
CHAPTER
Strain Energy Methods

Q.11.1 The L-shaped bar shown in figure 11.1 is of uniform cross-section 60 mm x 120 mm. Calculate
the total strain energy. Take E = 2 x 105 MPa, G = 0.8 x 105 MPa.

b= 60 mm
d= 120 mm

Fig. 11.1
[IFS 2011, CE : 10 Marks]

Solution:
Total strain energy
U= UB + Us + UA
= Strain energy due to bending of BC + Strain energy due to shear of BC +
Strain energy due to axial load W
w20 3w2L w2L _
1+ 2
6EI 5Gbd 2AE
Where, W = 10,000 N, L1 = 2000 mm, L2 = 1000 MM, E = 2 x 105 MPa, G = 0.8 x 105 MPa
bd3 60 x1203
/- = 8.64 x106 mm4
12 12
b = 60 mm, d = 120 mm
A=bxd= 60 x 120 = 7200 mm2
Putting the values
(10000)2 x(2000)3 + (10000)2 x 0.6 x 2000(10000)2 x1000
Strain energy, U- +
6 x2 x105 x8.64 x106 0.8x105 x60 x120 2 x7200 x 2 x105
= 77160.5 + 208.33 + 357.143 Nmm
= 77725.967 Nmm = 77.725 Nm
150 N. lAS & IFS (Objective & Conventional) Previous Solved Questions MADE EASY

Q.11.2 The cross-sectional area of each member of the truss


shown in figure 11.2 is A = 400 mm2 and E= 200 GPa.
Determine the vertical displacement of joint C if a 4 kN
force is applied.

Solution: 4 kN
Reactions
12
RBV= 8 = 1.5 kN T

RAV 1.5 kN T
4 kN Fig. 11.2
RAH
Force in members
2 2
FAB - = — = -2.5 kN
cosO 0.8
2
FcA
+ = +2.5 kN(tensile)
cos() 4 kN
FAB +2 kN (tensile)
Unit load diagram (Fig. 11.3)

1 kN 1.5 kN

2 kN

0.5 kN

0.666 kN

0.5 kN 0.5 kN

Fig. 11.3 Members n N nNL


CB -0.833 -2.5 kN 10.4125
FCA FCB -0.833 CA -0.833 +2.5 kN -10.4125
FAB = +0.666 AB +0.666 +2 kN 10.656
1nNL = 10.656 kNm Z nNL = 10.656 kNm
A = 400 mm2
= 400 x 10-6 m2
= 4 x 10-4 m2
E = 200 kN/mm2 = 200 x 106 kN/m2
AE = 4 x 10-4 x 200 x 106 = 800 x 100 kN
= 80000 kN
EnNL 10.656
Vertical deflection at C, 0.1332 x10-3 m
AE 80000 -
= 0.1332 mm

Q.11.3 A shaft is supported by two anti-friction bearings with loads of 140 Newton each acting at points
B and F as shown in Fig.11.4. Portion of the shaft between B and C has diameter of 2D as
Strength of Materials Strain Energy Methods 4 151

compared to a diameter D for the portion of the = —P


shaft between A and B and between C and F. Using
Castigliano's theorem, determine the deflection of
the shaft at points B and F.
[CSE-Mains, 1992, ME : 20 Marks]

Solution:

Let us take length L, with AB = BC = CF=


RA =P
Moments about A
1.-150 cm-44-150 cm-44-150 cm
PL L/3 L/3 L/3
= Re xL
3
Fig. 11.4
2PL 2L
Re x —
3 3
Rc =
RA = +PT
BMD (Fig. 10.5) E
MA = 0
PL
PL 3
MB +-3
B C F
2PL PL PL Fig. 10.5
MC - 3 - 3 3

MF =

704
1AB = CF
64

7c(204 1670 4
18c =
64 64
18c = 16 /AB

f L/3 (Px)2 L/3 P2x2 p 2x3 L/3 P 2 L3


Strain energy, dx xdx _
UAB — UCF = JO 2EI AB 2EIAB 6EIAB 0 -162E/ AB

2
(11)
UBc x3 = P2L3
'
2EIBc 27 x 2 x E I Eic

2P2L3 P2L3 P2L3 P2L3


U— + = +
162E/AB 54 E/Bc E I AB 81 54 x16 E BAB

P2L3 1 1 11,667P2L3
DAB 1_81 864 j DAB x 864

704
/AB -
64

(:a
15 2 0. IAS & IFS (Objective & Conventional) Previous Solved Questions MADE EASY

11.667P2L3 64 746.667P2L3 237.67P2L3 0.275P2L3


U— x =
E x 704 864 ED47c x 864 ED4 x864 ED4
Say deflection at B and F is 8, then
au 0.275 x 2PL3
8— =
ap ED4
8 — 0.550 PL3
ED4
P= 140N
L= 4500 mm, D in mm

0.550 x140 x 45003 7.02 x1012


8— mm
ED4 ED4

Q.11.4 A steel tube having outside and inside diameters of 10 cm and 6 cm


respectively is bent into the form of a quadrant of 2 m radius as shown 1000N = P
in fig. 10.6 One end is rigidity attached to a horizontal base plate to
which a tangent to that end is perpendicular, and the free end supports
a load of 1000 N. Determine the vertical and horizontal deflection of
the free end under this load using Castigliano's theorem. E. 2 x 1011
N/mm2.
[CSE-Mains, 1993, ME : 20 Marks]

Solution:
Vertical deflection Ht--- 2 m
Take a small element Rd 0 at an angle 0 from vertical Fig. 10.6
Bending moment at small element = P.R sin()
fir/2 p 2R 2 sin
2
ORA
rt/2(M9)2Rdo
U, strain energy = So
2E/ — 2EI

au
= sv , vertical deflection
ap
PR 3 ic/2 PR3 (1— cos 20) d0
= sine Ode =
fola EI E, 2
n12 1 PR3 PR3 cos 20
= d0
fo 2 E1 X de n/2
So E/ x 2

7EPR3 PrcR 3
. 0=
4EI 4EI

= 0 0 4 _ 6 4) = 427.256 cm4
64
E= 2 x 107 N/cm2
= 854.513 x 107 Ncm2

1000 x icx 2003


- 4x 854.513x10',= 0.1838 cm
Strength of Materials Strain Energy Methods 41 153

Horizontal deflection
Put frictitious horizontal load, H = 0 at A
M0 = P(R sine) + H(R - R dose)
l 2
U 171/2
J. (M ) Rd()
2E1e
au cir/2 1
OH = .10 2E1[PRsine][R-Rcos0] x Rd()
aH
/r/2 2
§-1 = So 2EI [PR3][sin0-sin0cose]d0

3 r it/2 PR 3 f rE/2 sin20


= PR sin0d0 c/0
El Jo EI Jo 2
n/2 nI2
_ PR 3
cos0
PR 3 cos20
El + EI 4 0
0
PR3 PR [cosic cos01
=+ +
El El 4 4 j
PR3 PR3 [ 1 1] PR3
. +— - - =
El El 4 4 2E1
2
§4 = it- x8 =0.117 cm

Q.11.5 Using the Castigliano's theorem, calculate the vertical deflection 8 at the middle of a simply
supported beam which carries a uniformly distributed load of intensity w over the full span. The
flexural rigidity El of the beam is constant and only strain energy of bending is to be considered.
[CSE-Mains, 1991, ME : 20 Marks]

Solution:
Let us apply a fractious load P. 0 and the centre of the beam reactions,
wL P 0
RA = Rg = —+ — P=
2 2 l'x —1-I w

wL P wx 2
BM at section x, MX=
Tx+ V 2
Beam is symmetrically loaded about center
P
U2 (Mx) 2 dx
Strain energy, U= 2
.10 2E1 Flg. 10.7

2 j am
u2 2m x x—x dx
2E/ 0 DP

[ wL P wx 2 ]x
= .dx
El 0 2 x+ 2 x 2 2
But P = 0
2 ulwLx 2 wx3 d
8= i
El o 4 4 )"
154 0. lAS & IFS (Objective & Conventional) Previous Solved Questions MADE EASY

L/2
2 wLx3 wx4 2 wL4 wL4
EI 12 16 0 El 96 256

2 x 8-3
= xwL4 = 5wL4
El 768 384E/

Q.11.6 A circular rod of diameter d is bent at right angle. It is fixed


at one end and a load W is applied at the other end as shown
in the fig. 10.8. Determine the deflection under the load W, if
E and G are elastic and shear modulii of the material.
[CSE-Mains, 1996, ME : 20 Marks]
Solution:
w 2b3
Strain energy due to bending in BC -
6E/

2a
Strain energy due to twisting in member AB - (Wb)
2GJ
W2a3
Strain energy due to bending in member AB -
6E1
IA/ 4,3 2,3
vv Li a + vv
m/a
Total strain energy, U = "
6E1 2GJ 6EI
au 8 - Wb3 Wb2a Wa3
+
aw 3E1 GJ 3E1
J= 21
Wb3 Wa3 Wb2a
8- + +
3E/ 3E/ 2GI

ird 4
I -

8-
64
W [ b3 a3 b 2al 64W
+ +
I 3E 3E 2G
=
704
r 3E
+ a3 b 2a
2G ]

Q.11.7 (a) State the theorem of Castigliano's.


(b) Using the above theorem, find the horizontal displacement along the load line of the frame
shown in fig. 10.7. Considering the deflection due to bending only. The moment of inertia
is the same for all sections.
B
Pa)
B ' P 10-P P44 '
Pa c)
P8 Pe
1-4-- b a

D
Fig.10.7
[CSE-Mains, 1996, ME : 30 Marks]
Strength of Materials Strain Energy Methods 4 15 5

Solution:
Castigliano's Theorem
If a body is acted upon by forces F1 , F2, F3 .. F and U is the strain energy stored in the body due to these
forces (causing, axial stresses, bending moments or twisting moments etc. then partial derivative of the
strain energy with respect to force F., gives the displacement of the body in the direction of
au
aF; 8i, deflection along force F;

Free body diagram of the figure is given


P 2a3
Member CD
Uco 6 E1
Member BC
p2 a 2 x b
UBC 2E1
Member AB
P2a3
UAB 6E/

P2a3 P 2 a 2 b
Total strain energy,
3E./ 2E1
au
= 8 = deflection along P
ap P

2Pa3 Pa 2 b Pa2
_ [2a + 3b]
3E1 EI 3E1

Q.11.8 Determine the vertical deflection of the joint C of the frame shown in fig. 10.10. Area of cross-
section of each member is 850 mm2. E = 200 kN/mm2.
5 kN

10 kN

RDH = 1 kN

Unit load at C
3
RAV =15kN RDV = 10 kN

Fig. 10.10
[IFS 2013, CE : 10 Marks]
Solution:
Reaction: Taking moments about D
10 x 3 + 5 x 3 = RAvx3
RAC = 15 kN
156 1. lAS & IFS (Objective & Conventional) Previous Solved Questions MADE EASY

RDH = 10 kN
Roy = 10kN J.
Forces in the members
FED = -10 kN (comp.)
FDD = 0
FAB = -15 kN (comp.)

FBD = +1 = +14.14 kN(Tensile)


Unit load at C
FDD = -1 (comp.)
FBD = o

FAB o
FBD =0

Members N n L NnL
AB —15 0 3 0
BC —10 0 3 0
CD 0 1 3 0
BD +14.14 kN 0 3'I2 0
E nNL = 0
Vertical deflection at joint C is zero

Vertical deflection at point C is zero.

Q.11.9 The steel truss shown in fig. 11.11 is anchored at 6 kN 6 kN


A and supported on roller at B. If the truss is so
designed that under the given loading all tension V
members are stressed to 100 N/mm2, and all
E
compression members to 80 N/mm2, find the cq
vertical deflection of the point C. E= 2 x 105 N/mm2. 6 kN
Find also the lateral displacement of the end B. [4— 2.4 m —44-- 2.4 m —44— 2.4 m
[CSE-Mains, 1998, CE : 40 Marks]
Fig .11.11
Solution:
There are 9 members in truss.
Zero force member is FD
Compression member AE, EF, FB, ED, FC
Tension member AD, DC, CB
Draw unit load diagram, unit load at Cas shown below:
= tan-1 0.75
= 36.87°
sine = 0.6
cos() = 0.8 0.444 F 0.888
For unit vertical load at C,
Reactions at A and B 0-) co.
0

RA = D 0.888
-1
2.4 m —.4.-- 2.4 m 2.4 m
2
RB = 5
Fig. 10.12
Strength of Materials Strain Energy Methods 41 15 7

Joint B Joint A
0.888 0.444

Joint E
FED = 0

FEF = 0.444

Joint F Joint C

0.444 (Net)

0.444
0.888

Members Type of force N/A n nN/A L(mm) n(N/A) L

AE C -80 -0.444 +35.52 2400 85248

EF C -80 -0.444 +35.52 2400 85248

FB C -80 -0.888 +71.04 2400 170496

AD T +100 +0.555 +55.5 3000 166500

ED C -80 0 0 1800 0

DF 0 0 -0.555 0 3000 0

FC C -80 +0.333 -26.64 1800 -47952

CB T +100 +1.111 +111.1 3000 333300

DC T +100 +0.888 +88.8 2400 213120

En L = 340992 + 499500-47952 + 213120 =1006160


A

NL 1006160
Zn-- = = 5.03 mm
AE 2 x 105

MUM
12
CHAPTER
Miscellaneous Questions

Q.12.1 Determine the bar forces in the members CB, BE and EF


10 kN 10 kN
of the truss as shown.
[IFS 2013, CE : 15 Marks]

Solution:
Force in various members can be determined by method of
sections.
(i) Let us break the frame by section x-x as shown
Taking moments about B (force in BF passes through
point B so no moment caused by it)
[4-4 m -+- 4 m 4m
10 x 4 + 20 x 4 = FEF X= FEF x distance B P)
vL Fig. 12.1
120 = FEF x 2.828
FEF= —42.443 (Comp.), Foo = +20 kN (Tensile), FOE = —10 kN (comp.)

10 kN
(ii) Cut off by yy-section
Taking moments about E
C 20
FcB x 4 = 20 x 4 — 10 x 4 = 40
FCB 10 kN (tension)
Taking cut along section z-z as shown
y
10 kN 10 kN

m —444-- 4 m

Member BC, BE, BFare cut


FCB 10 kN (tensile)
Taking moments about point F
20 x 8 — 10 x 4 — 20 x 8 + FBE x 4 = 0
FBE = 10 kN (comp.)
Strength of Materials Miscelaneous Questions 4 15 9

FBE 10 kN (comp.)
FCB +10 kN (tension)
FEF -42.443 kN (Comp.)

Objective Questions
Q.1 Figure 12.2 shows the loading pattern on a (a) 1 (b) 2
truss. The force in the member AC is (c) 3 (d) 4
2t 4t 4t 4t 2t [CSE-Prelims, CE : 2001]

Q.4 A pin-jointed lower truss is loaded as shown in


the below figure. The force induced in the
member DFis

8t 8t A
Fig. 12.2
(a) Zero
(b) 2t
(c) 8t
(d) Statically indeterminate
[CSE-Prelims, ME : 2001]

Q.2 What is the vertical deflection of joint 'N' of the


pin-jointed frame as shown in the figure below?
4m

Fig. 12.5
(a) 1.5 kN (Tension) (b) 4.5 kN (Tension)
(c) 1.5 kN (Comp.) (d) 4.5 kN (Comp.)
[CSE-Prelims, CE : 2002]

0.5 For the truss shown in the figure, which one of


FIg. 12.3 the following members has zero force induced
(a) PL/6AE (b) PL/3AE in it?
(c) PL/AE (d) 2PL/AE
20 kN
[CSE-Prelims, CE : 2008]

Q.3 The degree of static indeterminacy of the pin-


jointed plane frame shown in figure is
w2

Fig. 12.6
(a) BC (b) AD
(c) DE (d) BD
[CSE-Prelims, CE : 2003]
Fig. 12.4
160 ► IAS & IFS (Objective & Conventional) Previous Solved Questions MADE EASY

Q.6 The force induced in the vertical member CDof (a) 0 (b) 40 kN
the symmetrical plane truss shown in the (c) 80 kN (d) 100 kN
figure is [CSE-Prelims, CE : 2006]
50 kN
Q.9 In the plane truss shown above, how many
members have zero force?

50 kN
Fig. 12.7 A A

(a) 50 kN (tension) (b) 100 kN (tension)


(c) 50 kN (Comp.) (d) Zero
[CSE-Prelims, CE : 2004] Fig. 12.10
(a) 3 (b) 5
Q.7 In a truss work as shown in the figure given (c) 7 (d) 9
below, what is the force induced in the member [CSE-Prelims, CE : 2006]
DE?
50 kN Q.10 What is the force in the member EH for a pin-
jointed tower truss as shown in the figure?

2 kN A
T
3m

2 kN

3m
Fig. 12.8
2 kN —110.E
(a) 50 kN (tensile) (b) Zero
(c) 50 kN (Comp.) (d) 25 kN (Comp.) 3m
[CSE-Prelims, CE : 2005]
A
M
.4
I
Q.8 Due to horizontal pull of 60 kN at C, what is the
force induced in the member AB? ~-4 m H
C
60 kN

Fig. 12.11
(a) 6.0 kN (Tension) (b) 6.0 kN (Comp.)
(c) 7.5 kN (Comp.) (d) None of the above
[CSE-Prelims, CE : 2008]
Answers

1. (a) 2. (c) 3. (d) 4. (c) 5. (a)


3m
6. (a) 7. (b) 8. (d) 9. (d) 10. (c)
Fig. 12.9
Strength of Materials Miscelaneous Questions 161

Explanations
1. (a) FDF = 4.5 kN (T)
2t
FFE = 4 kN (T)
At joint F

8t

8t
Zero force member at joint A.

2. (c) FDF = 5 kN
5 cos() = 4 kN
Balance FEF
5 kN sine = 3 kN
3 + FDF = 4.5 kN
FDF = 1.5 kN (Comp.)

5. (a)
In members BC and CD, no force.
Vertical displacement of N FAB =
1 [P L 1 P L 1 FED = 0
x x + xx Fa), = 0
A E A AE
6. (a)
3. (d)
At joint D
Members = 21 = m
Fop = 50 kN (T) Tension
Joints = 11 = j
Ds = m + Re - 2j 7. (b)
= 21 +(2 x 2 +1)- 2 x 11 = 4
Static indeterminacy 2. 50 kN
4. (c)
12 + 6
RE- 4 = 4.5 kN

4 kN
At the joint E,
= 4.5 kN FDE -= 0
4.5 kN
16 2 0. lAS & IFS (Objective & Conventional) Previous Solved Questions MADE EASY

8. (d) FEH cos° = 6


At joint C
6
F = = 7.5 kN(Comp.)
84 0.8

Q.12.2 Analyse the continuous beam as shown in


fig. 12.8 by moment distribution method.
Draw the bending moment diagram.
Assume El = constant.

20 kN 20 kN/m
60 kN

4 A'
sin° = - = 0.8
5
cos° = 0.6 2m 3m 4m

60 60 -40 IN/m
F - = = 100 kN
Bc cos() 0.6 +24 kN/m
FEic = FAB = 100 kN -20.488
FBD = 0

9. (d)
Fig.12.8

[CSE-Mains, 2012, CE : 15 Marks]

Solution:
Mp = 0
Mc = -40 kNm

144_2 3 x 42
ME = = + 8 = +6 kNm
8

None zero force members. 24x 4 24x3x(


a1.71 = x 2+1)
2 2 3 +2
10. (c) = 32 + 108 = 140 kNm2

a1.771' ( Origin at B)
24x3 24 x2 2
x 2+ x3+-
2 2 3
11
72 + 24-) = 160 kNm2
3
a272 (origin at B or C)

= 2
-x 6x 4x 2 = 32 kNm2
9 kN 9 kN 3
sin° = 0.6 Considering span AA' (imaginary) of span
cos0 = 0.8 zero.
Clapeyron's theorem for spans A'A, AB
18+12+6
RH - = 9 kN
4 M A' X01-2MA(5-1-0)+5MB
Strength of Materials Miscelaneous Questions 41 163

6 x ai.71 5MA + 2.5MB -96


= 0 (origin at B) 15.5% +40
5
MB +2.58 kNm
10MA + 5MB= -5 x 160 —192 -CO 5MA = -56- 18MB = -56 - 18 x 2.58
-56 - 46.44 = -102.44
10MA + 5MB = -192
MA -20.488 kNm
Or 5MA + 2.5MB = -96
MB +2.58 kNm
Spans AB and BC
MC -40 kNm
5MA + 2MB(4 + 5)+ 4Mc

6 xaqi 6 xa 2-
x2
(origin at A)
5 4
zE
-6 x 140 6 0
x 32
5 4
= -168-48 =-216
A
5MA + 18MB - 4 x 40 = -216 +2.58
5MA + 18MB = -56
, P2 points of contraflexure

BM Diagram

Fixed and Continuous Beams

Q.1 A fixed beam as shown in the figure 12.15 has (a) 0.5 (b) 1
a span L and uniform flexural rigidity El. It is (c) 1.5 (d) 2
subjected to a concentrated clockwise moment [CSE-Prelims, CE : 2006]
at the centre. The deflection at the centre of the
Q.3 A beam shown in fig. 12.16 of span 10 m and
beam is
having El = 10000 kNm2 is subjected to a
rotation of 0.001 radian at end B. What is the
fixed end moment at A?

14e-- L/2 )-I4 L/2 —H

Fig. 12.15
10m
ML2
(a) , upwards
8 EI Fig. 12.16
(b) ML2, downwards (a) 1.5 kNm (b) 2.0 kNm
(c) Zero (c) 3.0 kNm (d) 4.0 kNm

ML2
, downwards
(d) 384E/ Q.4 A heavy weight W attached to a rod can slide
[CSE-Prelims, CE : 2002] in a grooved support as shown in fig. 12.17.
What is the equilibrium sliding distance A?
Q.2 A fixed beam and a simply supported beam
w
having same span and develop same maximum
bending moment due to uniformly distributed
load on entire span, what is the ratio of uniformly
distributed load on fixed beam to that on simply L---4
supported beam Fig. 12.17
1 64 ► lAS & IFS (Objective & Conventional) Previous Solved Questions MADE EASY

clockwise. What is the distance of the point of


WL3 WL3
(b) contraflexure from the left support?
(a) 3E/ 12 EI
3L
WL3 WL3 (b) TC)
(c) 24E1 (d) 48E1 2L
(d)
[CSE-Prelims, CE : 2010]
[CSE-Prelims, CE : 2006]
Q.5 What is the bending moment at A for the beam
shown in fig. 12.18 Q.7 What is the fixed end moment for the beam
shown in fig. 12.19

E= 200 GPa E= 100 GPa

L -I- L

Fig. 12.18
PL 3PL
(a) -3-- (b) Fig. 12.19
2
2M0
PL 2PL (a) M0 (b)
(c) (d) 3
3
Mo
[CSE-Prelims, CE : 2010] (c) 3 (d)
60
Q.6 A horizontal fixed beam AB of length L has Answers
uniform flexural rigidity El. During loading the
1. (c) 2. (c) 3. (b) 4. (c) 5. (d)
right support A rotates through an angle 0
6. (a) 7. (b)

Explanations

1. (c)

W2 L2 AA
L/2 L/2 —al — 8 ""max

Zero deflection at centre.


1-2 w21-2
2. (c) 12 - 8

— = 1.5
W2

Hi— L/2 L/2


5. (b)

Wi
M -L2
24

wi 10m
MA = M1 = 12 = Mmax
Strength of Materials Miscelaneous Questions 4 16 5

Say equivalent load is W Pi + P2 = P

WL2 2P
0= =0.001 P1= 3
° 2E1
2P
Wx102
- 0.001 MA _ 3
2 x 10000
W 0.2 kN 6. (a)
MA = 0.2 x 10 = 2 kNm

4. (c)

4E/0
P L
2E10
L
He— L/3 I-1-4 2L/3
A fixed beam of length 2L, central load W,
L
Point of contraflexure lies at - from left support.
W (203 WL3 3
Ac - 192E/ 24E/
7. (b)

5. (d)

C
L -14 L

L-3
81 M 21 2/

P2 1-3 ►
['*-. 4 2 4
82 = 3E21
M0L ML 1 ML 1 ML 1
= 82 x x + x
21 - 4 21 4 21 2 1
P1 E
1 I= 2 ML ML _ 3 MI_
P2 E2 +
- 4/ 21 - 4 I
P1 = 2P2
M= -Mo
3
P P2

t B

C P2
FBD
13
CHAPTER
Rotational Stresses

Q.13.1 A circular disc 50 cm outside diameter has a central hole and rotates at a uniform speed about
an axis through its center. The diameter of the hole is such that the maximum stress due to
rotation is 85% of that in the thin ring whose mean diameter is also 50 cm. If both are of the same
material and rotate at the same speed, determine the diameter of the central hole and speed of
the disc for the datas given
Allowable stress = 900 kg/cm2
Specific weight = 7.8 gm/cm3
Poisson's ratio = 0.3
[CSE-Mains, 2006, ME : 30 Marks]

Solution:
In a thin disc, oc max occurs at inner radius, R1
If R2 is outer radius = 250 mm
,,„2
C max =
" [ki (2Fq + k 2R1

3+v .3
k1 - = = 0.4125
8 8
1+ 3v 1.9
k2 - = = 0.2375
8 8
2
PC° [0.4125(2 x 2502 +Rn- 0.2375Rn

x 2502 (02r2p1
= 0.85 xp [stress in ring =
g
0.85 x 2502 = 0.825 x 2502 + 0.175 R12
0.025 x 2502 = 0.175 R12
R1 = 94.5 mm
Allowable stress = 900 kg/cm2 = 900 x 9.81/cm2 = 88.29 N/mm2

2 x 2502
7.8 x10-6x 0.85 x (0 88.29 N/mm2
9810
(02 = 2090195.84
= 1445.175 rad/sec
N = 13806 rpm

(:4
Strength of Materials Rotational Stresses 1 16 7

Q.13.2 A steel rotor disc of uniform thickness 50 mm has an outside diameter 800 mm and a central hole
of diameter 150 mm. There are 200 blades each of weight 2N at an effective radius of 420 mm
pitched evenly around the outer periphery of the disc. Determine the maximum rotational speed
such that maximum shearing stress in the disc does not exceed 375 MN/m2. Take (p) of steel as
7470 kg/m3. Following basic relations for radial stress (ar) and Hoop stress (a0) at radius of
rotating disc at w rad/sec can be used with usual notations

po) 2r 2 pco 2 r 2
= A — —(3+v) , Go = A+ 2 (i + 3V)
8 8
[CSE-Mains, 1995, ME : 30 Marks]

Solution:
t= 50 mm, R2 = 400 mm, R1 = 75 mm, timax= 375 N/mm2 and p = 7470 kg/m3
Number of blades = 200
Effective radius = 420 mm
Weight of each blade = 2N
Centrifugal force on the periphery due to blades
200 x 2 400 x co2 x 420
x co2 xr =17.1254t w2N
9810
Resulting area = it x 800 x 50 = 12.566 x 104 mm2
Radial stress at the periphery of the disc
17.1254(1)2
=1.363 x10-4 w2Nimm2
12.566x104
Radial stress at the inner radius 75 mm is zero

pco 2r 2
6r = A——(3+v)
8
Taking v = 0.3, Poisson's ratio for steel
2 2
or = 0 = A E3 (3 + 0.3)Pc° r
752 8
p = 7470 kg/m3 = 7470 x 10-9 kg/mm3 = 7.47 x 10-6 kg/mm3
3.3
8 = 0.4125

B 0.4125x 7.47 x10-6 x(02 x752


0 = A 75“r, 1
B
=A 1.7332 x10-2 co2
5625

A B — 1.7332 x 10-20 Nimm2


5625
Radial stress at periphery of the disc = 1.363 x 10-4 (02 Nmm2
B 3+0.3 7.47 x10-6 w2 x 4202
1.363 co2 x 10-4 = A —
4202 8 1

1.363 co2 x 10-4 = A 0.543 w2


176400
168 ► IAS & IFS (Objective & Conventional) Previous Solved Questions MADE 'EASY'

B
0 0.5431363 = A
176400

1.7332 x 10-2 w2 = A B ...from equations (i)


5625
B x 170775
0.525800)2 =
5625 176400 5625 x 176400
0.525800)2 x 5625 x 176400
— — 3055 0)2
170775
B
0.5431363a)2 +
A
176400
= 0.5431363 0 + 0.017318 0)2
= 0.560450
Maximum Hoop stress occurs at inner radius 75 mm.
(1+ 3v)
ao. = A+ 2 x 30)2r 2
8

2 305542 1.9 6 x 032 x 752


0.560450) + 75,, 8 x 7.4 x 10-
= 0.56045 0)2 + 0.5431 0)2 — 0.998 x 10-2 (02

GA
-max
'Cmax = 375 mN/m2
2

Goma, = 750 N/mm2 = 1.09337 0

w = 685.827 rad/sec
27EN
60
N = 6550 rpm

Q.13.3 (a) What do you understand by rotating disc of uniform strength?


(b) A turbine disc is required to have a uniform stress of 150 MPa at a speed of 3200 rpm. The
disc is to be 30 mm thick at the centre. What will be its thickness at a radius of 40 mm?
Assume density of disc material = 7800 kg/m3.
[IFS 2012, ME : 10 Marks]

Solution:
(a) Rotors of a steam or gas turbine, on the periphery of which
turbine blades are attached are designed as disc of uniform
strength, in which the circumferential and radial stresses,
developed due to centrifugal forces on account of high angular
speed ware equal and constant and independent of radius.
Both radial and circumferential stresses are maximum at the
centre of the disc, while radial stress becomes zero at outer
radius but circumferential stress reduces to a minimum at the
outer radius. So as, the stresses increase towards the centre
of the disc, thickness of the disc also increases to maintain
constant stress ar = Go = a as shown in figure. Relation between Fig. 13.1. Disc of uniform strength
Strength of Materials Rotational Stresses 4 169

pa 2 r 2
2ga
thickness t at any radius r and thickness to at the centre is t = to.e
Where,
r= radius, at which thickness is t
co = angular velocity of rotation of disc in rad/sec
p = density of the material
g= acceleration due to gravity
a = constant stress
(b) r = 40 mm
p = 7800 kg/m3
7800 x 9.8
= 0.07644 x10-3 N/mm3
109
27c x 3200
— = 335.10 rad/sec
60
a = 150 N/mm2
g = 9810 min/sect
p(.02r2 0.07644 x 10-3 x 335.12 x 402
= 0.004666
2ga 2 x 9810+150
pt 2r2
296
e = e-0.004666 = 0.99534
t = to x 0.99534 = 30 x 0.99534 = 29.86 mm
Note that radius given is too small
Let us take 40 cm radius or 400 mm radius

pco 2r 2 0.07644 x10-3 x 335.12 x 4002


= 0.4666
2ga — 2 x9810 x150
43-0.4666
= 0.6270
t = to x 0.6270 = 30 x 0.6270 = 18.8 mm
14
CHAPTER
Unsymmetrical Bending

Q.14.1 A cantilever beam of span 3 m is subjected to a vertical load of 1.0 kN at free end. The cross-
section of the beam consists of equal angle 100 mm x 100 mm x 12 mm with one of its legs placed
vertically. Find the magnitude and direction of the resultant deflection. Given /uu = 329.3 cm4.
= 84.7 cm4, E= 2 x 105 N/mm2, centroidal distance = 29.2 mm.
[IFS 2013, CE : 15 Marks]

Solution:
/„ = 329.3 cm4, /w = 84.7 cm4
Position of neutral axis
/ 0 =45°
tam = tan() 100 ‘,
x
I„
0 = 45° and tang = 1

329.3
tang — = 3.888
84.7 29.2-1
r 00
a = 75.57°
Fig. 14.1
S = deflection at free end

2
wL3
sin2 0(iruu + cos2 0
3E1„

W = 1 kN
/„ = 329.3 x 10-8 m4
E = 200 kN/mm2 = 200 x 106 kN/m2
3EI„= 3 x 200 x 106 x 329.3 x 10-8
= 1975.8 kNm2
L = 3 m, = 45°

2
sin2 01W + cos2 0 = \10.5 x1329'312
l'w ) 84.3 +0.5
Fig. 14.2
= V7.55765 + 0.5 = 2.8386

3
S— x 2.8386 = 0.0388 m = 38.8 mm
1975 .8
Resultant deflection in a direction perpendicular to neutral axis.
Strength of Materials Unsymmetrical Bending 4 171

Q.14.2 Stress concentration factor is not considered harmful for ductile materials
in static loading but for brittle materials, it has damaging effect in both
static and dynamic loading. Justify the above statement giving
illustrations.
[CSE-Mains, 2011, ME : 15 Marks]

Solution:
In a brittle material, due to stress concentration, if ama, = SCF x aav is more
than nut, ultimate stress, and the crack is developed in sample then it is
carried through and specimen breaks. Figure shows a flat sample with a central
hole, amax = 3 (Yaw a SCF = 3, crack may develop at inner edge of the hole and
carried through axes aa. But in a ductile material, the plastic deformation at
the tip of the fine crack blunts the edge of crack and further propagation of
crack becomes difficult in a region where SCF is less. In fatigue load, brittle Fig. 14.3
material should never be used. Sometimes in the presence of a notch in a
machine member, ductile material behaves as a brittle material and machine
member breaks.

Q.14.3 A cast iron sample when tested in compression fails along


approximately 45° plane from its axis while when tested in torsion
also fails along a 45° (approx) helical plane from its axis. Explain
the reason for such failure and mention about the dominating stress
causing failure.
[CSE-Mains, 2011, ME : 10 Marks] Shear failure

Solution:
Cast iron is weak in tension and shear, but strong in compression. Its
compressive strength is about three times its tensile strength.
When CI sample is tested in compression, ac is the axial compressive
stress, but shear stress "E is maximum on planes at ±45° to the axes of
Fig. 14.4
the sample. So sample breaks under shear on inclined plane at approx
45° to the axis as shown.

- ti
\p 2
Failure under torsion

Fig. 14.5

When a specimen is subjected to twisting moment, angular twist 0 and shear stress "C act on the sample.
Shear stress is maximum at outer radius and gradually reduces to zero at the centre. Principal stress
p1 = +T acts on a plane about 45° to the axis of the sample. Due to angular twist, fractured surface
becomes helical.

Q.14.4 How ductility of structural steel in evaluated from static tension? What is the effect of gauge
length on the value of ductility?
[CSE-Mains, 2011, ME : 10 Marks]
17 2 0- IAS & IFS (Objective & Conventional) Previous Solved Questions MADE EASY

Solution:
A standard sample of structural steel is tested under tension using Universal testing machine. Original
diameter and original gauge length of the sample are noted down before applying tensile force. Tensile
force on specimen is slowly and gradually increased till the specimen breaks into two pieces. Final
gauge length is measured using an internal callipers.
Find gauge length = L'
Initial gauge length = L

L' —L
% elongation — L x 100

Ductility is measured by percentage elongation of the specimen


d A

4
L
Gage length

Fig. 14.5
As per Barba's law
Change in length SL = L'— L
= bL+c,174
Where b and care Barba's constants
L = gauge length, A= area of cross-section
As the gauge length increases, bL increases but cJ remains constant. Therefore as the gauge length
of the specimen increases, percentage elongation also increases.

Q.14.5 (a) What is fatigue? Define fatigue limit and fatigue strength.
(b) Explain how various factor of design effect the fatigue strength.
(c) What is cumulative fatigue damage? Explain Miner's hypothesis and bring out the drawbacks
of this hypothesis.
[CSE-Mains, 2008, ME : 5 + 5 + 10 = 20 Marks]

Solution:
Behaviour of any material under fluctuating loads or stress cycle is
termed as fatigue. For different stress levels, fatigue tests are
performed on samples and cycles required for fracture of sample
at a particular stress level S1, noted down say it is N1. Then S1 is
s'
known as fatigue strength of material at stress cycles N1.
If a graph is plotted between S,. and then the stress Se at which S. I
_ SB
the graph becomes asymptotic is known as stress limit as shown
in the figure.
Various factors affecting the fatigue life are (i) surface roughness Fig. 14.5
(ii) size factor (iii) stress concentration factor (iv) factor of safety
(v) order of reliability.
Depending upon the surface roughness fatigue life of member is reduced. As the size increases life of
the member decreases. As the stress concentration factor increases life of member decreases under
fatigue loading. Finally depending upon the reliability factor more reliability factor less is the life.
Strength of Materials Unsymmetrical Bending 41 17 3

Cumulative fatigue Damage: There Is cumulative effect of stress levels and number of cycles put in at
that level, that forms a fraction of total life upto failure
k n.
N
1.1 i=C
ni = number of cycles accumulated at stress S,
N= total number of cycles upto failure at stress level S,

+ n2 n, nk
N1 N3 ... Nk = 1

or + C2 ... Ci + Ck =
C is the fraction of life consumed by exposure to the cycles at different stress levels.
This is known as Miner's rule or hypothesis.
Drawbacks: Miner's rule becomes invalid if any stress level is more than Cry p (yield point stress) or less
than Se (endurance limit stress).

11.11111111
15
CHAPTER
General Objective Type
Questions

Directions: The following 4 questions, items consist Reason (R): Twisting occurs hence the line of
of two statements one labelled as 'Assertion A' and action of the load does not pass through the
other labelled as 'Reason R'. You are to examine these web of the beam.
two statements carefully and decide if Assertion A and
Q.3 Assertion (A): A thin cylindrical shell is
Reason R are individually true and so weather the
subjected to internal fluid pressure that induces
Reason R is a correct explanation of the Assertion.
2-D stress state in the material along longitudinal
Select your answers to these items using the codes
and circumferential directions.
given below and mark your answer sheet accordingly.
Reason (R): The circumferential stress in the
(a) Both A and R are true and R is the correct
cylindrical shell is two times the magnitude of
explanation of A
the longitudinal stress.
(b) Both A and R are true but R is not the correct
explanation of A Q.4 Assertion (A): Brittle material such as gray cast
(c) A is true but R is false iron cannot be extruded by hydrostatic pressure.
(d) A is false but R is true Reason (R): In hydrostatic extrusion billet is
uniformly compressed from all sides by the
Q.1 Assertion (A): I, T are channel sections are liquid.
preferred for beams. [CSE-Prelims, ME : 2000]
Reason (R): A beam cross-section should be
such that the greatest possible amount of area Q.5 Match List-I with List-II and select the correct
is as far away from the neutral axis as possible. answer.
List-I
Q.2 A channel cross-section of the beam shown in A. Proof stress
the figure 15.1 carries a uniformly distributed B. Endurance limit
load. C. Leaf spring
D. Modulus of rigidity
w N/m
List-I I
1. Torsion test
w N/m 2. Tensile test
CG
• 3. Fatigue test
4. Beam of uniform strength
Codes:
A
Fig. 15.1
(a) 2 3 4 1
Assertion (A): The line of action of the load (b) 2 3 1 4
passes through the centroid of the cross- (c) 3 2 4 1
section. The beam twists besides bending. (d) 3 2 1 4
Strength of Materials General Objective Type Questions 1 17 5

Directions: The following 4 (four) items consist of the C. Double shear stress
statement, one labelled as Assertion 'A' and other D. Maximum shear stress
labelled as Reason 'R'. You are to examine these two
statements carefully and decide if the Assertion 'A' 1. Neutral axis of beam
and Reason 'R' are individually true and of so whether 2. Closed coil helical spring under axial load
the Reason is a correct explanation of the Assertion. 3. Cylindrical shell subject to fluid pressure
Select your answers to these items using the codes 4. Rivets of double strap butt joint
given below and mark your answer sheet accordingly. Codes:
(a) Both A and R are true and R is the correct A B C D
explanation of A (a) 3 1 4 2
(b) Both A and R are true but R is not the correct (b) 4 2 3 1
explanation of A (c) 3 2 4 1
(c) A is true but R is false (d) 4 1 3 2
(d) A is false but R is true [CSE-Prelims, ME : 2007]
Q.6 Assertion (A): When an isotropic, linearly elastic 0.11 Match List-I (Type of thread) with List-II (Use)
material is loaded biaxially, the direction of and select the correct answer using the code
principal stresses are different from those of given below the lists:
principal strains. List-I
Reason (R): For an isotropic linearly elastic A. Square thread
material, Hooke's law gives only two B. Acme thread
independent material properties. C. Buttress thread
0.7 Assertion (A): In theory of torsion, shearing D. Trapezoidal thread
strain increases radially away from the List-II
longitudinal axis of the bar. 1. Used in vice
Reason (R): Plane transverse sections before 2. Used in lead screw
loading remain plane after the torque is applied. 3. Used in screw jack
4. Used in power transmission devices in
Q.8 Assertion (A): For a thin cylinder under internal machine tool
pressure, a minimum three strain gauges are Codes:
needed to know the stress state completely at
A BCD
any point on the shell.
(a) 2 3 4 1
Reason (R): If the principal stress directions
(b) 2 3 1 4
are not known, the minimum number of strain
(c) 3 2 1 4
gauges needed is three in a biaxial field.
(d) 3 2 4 1
Q.9 Assertion (A): Buckling of long column causes [CSE-Prelims, ME : 2007]
plastic deformation.
0.12 Match List-I (Natural of failure) with List-II
Reason (R): In a buckled column, the stresses
(Nature of member) and select the correct
do not exceed the yield stress.
answer using the code given below the lists:
[CSE-Prelims, ME : 2001]
List-I
Q.10 Match List-I (stress induced) with List-II A. Structural member fails in axial compression
(situation/location) and select the correct answer B. A member fails along a 45° helical plane
using the code given below the lists: subjected to torsion
List-I C. A structural member bends and collapse
A. Membrane stress under axial compression load
B. Torsional shear stress D. A member fails in double shear for a joint
176 ► lAS & IFS (Objective & Conventional) Previous Solved Questions MADE EASY

List-II Q.16 Which one of the following is true regarding the


1. Knucide joint fatigue life of a set of identical ball bearings?
2. Long column (a) Directly proportional to load
3. Strut (b) Inversely proportional to load
4. Cast iron round bar subjected to torsion (c) Inversely proportional to the square of load
Codes: (d) Inversely proportional to the cube of load
A BCD [CSE-Prellms, ME : 2009]
(a) 3 1 2 4 Q.17 Match List-I with List-II and select the correct
(b) 2 4 3 1 answer:
(c) 3 4 2 1 List-I
(d) 2 1 3 4 A. Partial derivative of strain energy w.r.t. load
[CSE-Prelims, ME : 2006] B. Derivative of deflection
C. Derivative of slope
Q.13 What type of fracture occurs when a brittle
D. Derivative of moment
material is under torsion?
List-II
(a) Cup and cone
1. Equation for shear force
(b) Granular transverse
2. Equation for slope
(c) Granular helicoidal
3. Equation for bending moment
(d) Smooth transverse
4. Deflection under the load
[CSE-Prelims, ME : 2006] Codes:
Q.14 Which one of the following is not correctly A BCD
matched? (a) 4 1 3 2
(a) Torsional rigidity : Torque per unit angle of (b) 3 2 4 1
twist (c) 4 2 3 1
(d) 3 1 4 2
(b) Modulus of rigidity : Strain energy per unit
[CSE-Prelims, ME : 2003]
volume
(c) Creeping effect : Loss of mechanical energy Directions: The following item consists of two
[CSE-Prelims, ME : 2008] statements, one labelled as the 'Assertion (A)' and the
other as 'Reason (R)'. You are to examine these two
Directions: The following items consist of the
statements carefully and select the correct answers to
statement, one labelled as the 'Assertion (A)' and the
these items using the codes given below:
other as 'Reason (R)'. You are to examine these two
(a) Both A and R are true and R is the correct
statements carefully and select the correct answers to
explanation of A
these items using the codes given below:
(b) Both A and R are true but R is not the correct
(a) Both A and R are true and R is the correct explanation of A
explanation of A (c) A is true but R is false
(b) Both A and R are true but R is not the correct (d) A is false but R is true
explanation of A
(c) A is true but R is false Q.18 Assertion (A): To a cantilever beam of circular
cross-section, if a moment is applied with its
(d) A is false but R is true
axis perpendicular to the axis of beam, no shear
Q.15 Assertion (A): Plane state of stress does not stress will be induced in the beam.
necessarily result in a plane state of strain. Reason (R): To the above beam, if moment is
Reason (R): A uniaxial state of stress in general applied with its axis along the axis of the beam,
produces a 3-dimensional strain state. no bending stress will be induced in the beam.
[CSE-Prelims, ME : 2008] [CSE-Prelims, CE : 2004]
General Objective Type Questions 41 177

Q.19 Match List-I with List-II and select the correct List-II
answer using the code given below the lists: 1. T-1
List-I 2. M LT-2
A. Beam 3. mL.2-r2
B. Column 4. ML-1T-2
C. Circular section of diameter d of a shaft Codes:
D. Close-coiled helical spring A BCD
List-II (a) 1 3 2 4
1. Member subjected to twisting (b) 4 3 2 1
2. Member used to store strain energy (c) 1 2 3 4
3. Member subjected to buckling (d) 4 2 3 1
4. Member subjected to bending [CSE-Prelims, CE : 2010]
Codes:
Q.23 Match List-I with List-I I and select the correct
A BCD
answer using the codes given below the lists:
(a) 4 3 1 2
List-I
(b) 1 2 4 3
A. Moment of inertia about diametral axis
(c) 4 2 1 3
B. Moment of inertia about an axis tangent to
(d) 1 3 4 2
the perimeter
[CSE-Prelims, CE : 2009]
C. Moment of inertia about an axis through
Q.20 Given that F +(3.12 ) where F denotes centroidal axis
force and t time; how is r3 described D. Polar moment of inertia
dimensionally? List-I I
(a) MLT-3 (b) MLT-2
(c) LT-4 (d) MLT-4 ico4 17704
1. 2.
[CSE-Prelims, CE : 2009] 32 64

Q.21 What is the unit vector of the resultant of the 5704 704
3. 4.
following two forces? 64 64
= 21+31+4k; F2 = 41+31+2k Codes:
A BCD
j (a) 4 3 2 1
(a) 61 + 61+ 6k (b) + + (b) 1 3 2 4
(c) 4 2 3 1
(c) -21+ 2k (d) 21- 2k (d) 1 2 3 4
[CSE-Prelims, CE : 2009] [CSE-Prelims, CE : 2010]
Q.22 Match List-I with List-II and select the correct Answers
answer using the codes given below the lists:
List-I 1. (a) 2. (c) 3. (b) 4. (b) 5. (a)
A. Modulus of elasticity 6. (d) 7. (b) 8. (d) 9. (c) 10. (c)
B. Work 11. (c) 12. (c) 13. (c) 14. (b) 15. (b)
C. Force 16. (d) 17. (c) 18. (b) 19. (a) 20. (d)
D. Frequency
21. (b) 22. (b) 23. (a)
178 N. IAS & IFS (Objective & Conventional) Previous Solved Questions MADE EASY

Explanations
1. (a) 11. (c)
Both A and R are true, R is the correct explanation A. Square thread 3. Used is screw jack
of A. B. Acme thread 2. Used is lead screw
C. Buttress thread 1. Used in vice
2. (c)
D. Trapezoidal thread 4. Used in power
A is true but R is false,
transmission
Line of action of the load must pass through the
devices in machine
shear centre.
tool
3. (b)
12. (c)
Both, A and R are true but R is not the correct
A. Structural member falls in axial compression
explanation of A.
B. A member falls along a 45° helical plane
4. (b) subjected to torsion
Both A and R are true but R is not the correct C. A structural member bends and collapse under
explanation of A. axial compression load
D. A member falls in double shear for a joint
5. (a)
3. Strut
A. Proof stress 2. Tensile test
4. Cast iron round bar subjected to torsion
B. Endurance limit 3. Fatigue test
2. Long column
C. Leaf spring 4. Beam of uniform
1. Knuckle joint
strength
D. Modulus of rigidity 1. Torsion test 13. (c)
Granular helicoid - Type of fracture in brittle
6. (d)
material under torsion.
A is false, R is true.
14. (b)
7. (b)
Modulus of rigidity strain energy per unit volume.
Both A and R are true but R is not the correct
explanation of A. 15. (b)
Both A and R are individually correct but R is not
8. (d)
the correct explanation of A.
A is false but R is true.
16. (d)
9. (c)
Fatigue life of a ball bearing is inversely
A is true but R is false.
proportional to cube of load.
10. (c)
17. (c)
A. Membrane stress 3. Cylindrical shell
subjected to fluid dU
4. Deflection
pressure A. dW
dW
B. Torsional shear stress 2. Close coiled dy
helical spring B. 2. Slope
dx
under axial load
C. Double shear stress 4. Rivets of double d2y
C. 3. Equation of BM
strap butt joint dx 2
D. Maximum shear stress 1. Neutral axis of dM
beam D. 1. Shear force
dx
Strength of Materials General Objective Type Questions 41 17 9

18. (b) 22. (b)


Both A and R are individually true but R is not the A. Modulus of elasticity 4. ML-1T-2
correct explanation of A. B. Work 3. mL2T-2
C. Force 2. MLT-2
19. (a)
D. Frequency 1. T-1
A. Beam 4. Bending
B. Column 3. Buckling 23. (a)
C. Shaft 1. Twisting
itD4
D. Spring 2. Strain energy storage A. MI about diametral axis 4.
64
20. (d)
MLT-2, = MLT-4 5704
B. AN about an axis 3.
64
21. (b)
tangent to perimeter
+ F2 = 61+61+6k
17704
6i+61+6k C. Micentroidal axis 2.
Unit vector — 64
-11)E3

61+61+6k TcD4
D. Polar moment of inertia 1.
6i 32

= - + - + -
I k
MINIM

You might also like